Young Christian: Volume 2, 1912

Table of Contents

1. Inspiration of the Scriptures: Examples of the Divine Authority of the O. T. Scriptures, Part 1
2. Unscriptural Marriage: Or, the History of Ellen, Part 1
3. Scripture Study: Why are there Four Gospels?
4. Truths for Young Christians: The Wilderness, Part 2
5. From Egypt to Shiloh: Part 1, God is Love
6. The Coming and Reign of Our Lord Jesus Christ: The First Resurrection, Part 3
7. Correspondence: Who were the "Sons of God" in Gen. 6:2,4?
8. Inspiration of the Scriptures: Examples of the Divine Authority of the O. T. Scriptures, Part 2
9. Unscriptural Marriage: Or, the History of Ellen, Part 2
10. Scripture Study: Matthew 1
11. Truths for Young Christians: Waiting, Part 1
12. From Egypt to Shiloh: Part 2, Shiloh
13. The Coming and Reign of Our Lord Jesus Christ: The Apostasy, Part 1
14. Correspondence: Matt. 12:31 and 1 John 5:16-17; Luke 7:36-50
15. Inspiration of the Scriptures: Some Samples of its Alleged Inaccuracies, Part 1
16. Surrender
17. Unscriptural Marriage: Or, the History of Ellen, Part 3
18. Scripture Study: Matthew 2:1-2
19. Truths for Young Christians: Waiting, Part 2
20. Burning With Pure Oil
21. From Egypt to Shiloh: Part 3, A Picture of the Church
22. The Coming and Reign of Our Lord Jesus Christ: The Apostasy, Part 2
23. Correspondence: Accomplishment of Salvation; Saints in Heaven; 1 John 1:6-7
24. Inspiration of the Scriptures: Some Samples of its Alleged Inaccuracies, Part 2
25. Unscriptural Marriage: Or, the History of Ellen, Part 4
26. Scripture Study: Matthew 3
27. Truths for Young Christians: Walking, Part 1
28. I Was on the Wrong Track
29. From Egypt to Shiloh: Part 4, The Revival of Shiloh
30. The Coming and Reign of Our Lord Jesus Christ: The Apostasy, Part 3
31. Correspondence: Pearls before Swine; Disobedient Healed; John 10:16
32. Inspiration of the Scriptures: Some Samples of its Alleged Inaccuracies, Part 3
33. Unscriptural Marriage: Or,the History of Ellen, Part 5
34. Scripture Study: Matthew 4
35. Truths for Young Christians: Walking, Part 2
36. Is Conversion a Reality?
37. From Egypt to Shiloh: Part 5, Hannah
38. The Coming and Reign of Our Lord Jesus Christ: The Apostasy, Part 4
39. Correspondence: Healing; Ps. 23:4; Birth, Conversion, Chastening; Matt. 25:32
40. Inspiration of the Scriptures: Some Samples of its Alleged Inaccuracies, Part 4
41. To Christians Contemplating Marriage
42. Scripture Study: Matthew 5:1-16
43. Truths for Young Christians: Walking, Part 3
44. Fragment: Evil and Good
45. The Power and Value of the Name of the Lord Jesus
46. The Father's House
47. From Egypt to Shiloh: Part 6, The Song of Shiloh
48. The Coming and Reign of Our Lord Jesus Christ: The Apostasy, Part 5
49. Correspondence: Meaning of "Baptized for the Dead"
50. Inspiration of the Scriptures: Some Samples of its Alleged Inaccuracies, Part 5
51. Our Man's Rejected
52. Wholeheartedness for Christ
53. Scripture Study: Matthew 5:17-48
54. Truths for Young Christians: Working for Christ, Part 1
55. From Egypt to Shiloh: Part 7, The Spirit of Samuel
56. The Coming and Reign of Our Lord Jesus Christ: The Apostasy, Part 6
57. Correspondence: Rom. 13:14; Acts 2:16-18; 1 Tim. 5:8
58. Inspiration of the Scriptures: Some Samples of its Alleged Inaccuracies, Part 6
59. The Lost Crown
60. Scripture Study: Matthew 6
61. Truths for Young Christians: Working for Christ, Part 2
62. The Breaking of Bread: Acts 20:7
63. The Coming and Reign of Our Lord Jesus Christ: The Apostasy, Part 7
64. Correspondence: Explanation of Ex. 33:11, 20; Acts 1:11
65. Inspiration of the Scriptures: The Silent Interval of Four Hundred Years
66. The Mighty Hand of God
67. Scripture Study: Matthew 7
68. Love Your Enemies
69. Truths for Young Christians: Selfishness
70. Christ the Center: Or, Why Christians Should Meet in His Name Alone, Part 1
71. The Coming and Reign of Our Lord Jesus Christ: The Apostasy, Part 8
72. Correspondence: Burdens in Gal. 6:2, 5; Luke 11:24-26; Matt. 16:17
73. Inspiration of the Scriptures: The New Testament, Part 1
74. The Fruit of Faithfulness
75. Scripture Study: Matthew 8
76. Truths for Young Christians: Pride
77. Be Thou in the Fear of the Lord All the Day Long: Proverbs 23:1
78. Christ the Center: Or, Why Christians Should Meet in His Name Alone, Part 2
79. The Coming and Reign of Our Lord Jesus Christ: The Appearing, Part 1
80. The Rest-Stone
81. Correspondence: Isa. 28:23-29; 1 John 3:6; 1 Cor. 15:58
82. Inspiration of the Scriptures: The New Testament, Part 2
83. A Plea for "Sound" Reading
84. Scripture Study: Matthew 9
85. Truths for Young Christians: Envy
86. Christ the Center: Or, Why Christians Should Meet in His Name Alone, Part 3
87. The Lord Is My Portion
88. The Coming and Reign of Our Lord Jesus Christ: The Appearing, Part 2
89. Truthfulness
90. Correspondence: 2 Cor. 12:16; Matt. 12:40; Heb. 5:7
91. Inspiration of the Scriptures: The New Testament, Part 3
92. A Plea for "Sound" Reading
93. Scripture Study: Matthew 10
94. Truths for Young Christians: The Sin that Envy Leads to
95. Christ the Center: Or, Why Christians Should Meet in His Name Alone, Part 4
96. An Appeal in These Last Moments
97. The Harvest Is Great
98. The Coming and Reign of Our Lord Jesus Christ: The Appearing, Part 3
99. Correspondence: Psa. 138:2; Acts 2:16-17; Rom. 11:26; The Lord's Table

Inspiration of the Scriptures: Examples of the Divine Authority of the O. T. Scriptures, Part 1

EXAMPLES OF THE DIVINE AUTHORITY OF THE OLD TESTAMENT SCRIPTURES.
With regard to the divine authority of the ancient Scriptures, nothing is more plainly marked than God’s displeasure at dishonor to His word, and the blessing that has accompanied obedience. The notice of a few instances will suffice to confirm this.
It was by one man’s disobedience that sin entered into the world, with all its consequences of sorrow, death and judgment. Later on, when God’s judgment of fire and brimstone was about to overtake the wicked cities of Sodom and Gomorrah, and Lot with his wife and daughters were rescued from it, they were commanded to “look not behind;” but one of the party thought it wiser to follow her own reasoning than the word of the messenger of God from heaven, and disobeyed. We are told that then God’s hand overtook her in judgment; for Lot’s wife “looked back.... and she became a pillar of salt” a standing monument of God’s displeasure (Gen. 19:17, 26). How awful it is to go contrary to God’s Word!
The children of Israel who came out of Egypt to the number of six hundred thousand men, never entered the land, except Caleb and Joshua, because they did not believe God; and long after they had had the benefit of God’s ordinances and prophets for a series of years, they rebelled against Him, refused His word, despised His prophets, and became worse than the heathen; so that both Israel and Judah were given into captivity. How true it is that God cannot look on disobedience to His word with any allowance!
Again, at the famine of Samaria, Jehovah’s servant, Elisha, prophesied. His testimony was, “Thus saith Jehovah, Tomorrow about this time shall a measure of fine flour be sold for a shekel, and two measures of barley for a shekel in the gate of Samaria.” But a man of high standing there unbelievingly replied, “If Jehovah would make windows in heaven might this thing be?” This was despising Jehovah’s word. So the prophet said unto him, “Behold, thou shalt see it with thine eyes, but shalt not eat thereof.” And so the words of Jehovah were fulfilled; for the next day there was plenty in the famished city, and this great, but unbelieving man, who despised God’s word, saw the plentiful supply of food, but instead of tasting it, “the people trod upon him in the gate, and he died” (2 Kings 7). Another appalling instance of the terribleness of refusing, through unbelief, God’s word!
Take another case, not that of an unbeliever, but of the failure of a man of faith David, a man after God’s own heart. He attempted to bring up the ark from Kirjath-jearim to Jerusalem; but in so doing God struck Uzzah dead on his touching the ark when the oxen stumbled, and David was deeply distressed. And why was this marked displeasure of the Lord? He afterward found out it was because he had acted according to his own reasonings, and that of others, instead of referring to the written Word and acting on it in faith. Oxen, cart and all were the mere inventions of David and his captains; so that when he knew this he was not surprised at the “breach.” From the book of Numbers he learned that, None ought to carry the ark but the Levites. He now takes the written Word as his guide, acts on it, and we know with what blessed result. O that all God’s people would search the Scriptures to know His mind, and seek grace and strength from Him to carry it out!
Before closing this part of our subject, let us briefly glance at King Jehoiakim in Jeremiah 36. The prophet had written in a roll what Jehovah had told him about the king and people, and it was read to the king, when he took a knife, cut the roll into pieces, and burnt it in the fire. What followed? We are told that the prophet wrote “another roll,” and added to it; and, as a mark of God’s displeasure, said, “Thus saith Jehovah of Jehoiakim king of Judah, ‘He shall have none to sit upon the throne of David; and his dead body shall be cast out in the day to the heat, and in the night to the frost’” (Jer. 36:21-32). How true are the words of Jehovah, “They that despise Me shall be lightly esteemed.”
Let us now briefly turn to a few examples of such as honored God in receiving His written Word, and acting on it, in days too, much like the present, when the traditions and opinions of men have set aside, or corrupted almost everything that God has made known for present guidance and blessing.
Passing by some who had evidently prospered in keeping God’s statutes and commandments, “as it is written in the law of Moses,” we find it recorded that King Jehoshaphat sought to the Lord God of his fathers, and walked in His commandments. He sent also teachers who taught in Judah, and had the book of the law of Jehovah with them. Present blessing was with him so connected with obedience to the written Word, that in the battle field he said, “Hear me, O Judah, and ye inhabitants of Jerusalem, believe in Jehovah your God, so shall ye be established; believe His prophets, so shall ye prosper.” We know how remarkably God’s blessing was with him (2 Chron. 17:4, 9; 20:20,30).
After this, in a time of abounding evil, we find that Jehoiada, after breaking down the house of Baal with its altars and images, appointed the offices of the house of Jehovah, as it is written in the law of Moses, with rejoicing and singing, as it was ordained by David (2 Chron. 23:17-18).
(Continued and to be continued.)

Unscriptural Marriage: Or, the History of Ellen, Part 1

Ellen was the only surviving child of a father who died while she was very young; and of a pious mother who brought her up in the nurture and admonition of the Lord, and who, being herself an accomplished lady, with a limited income, had educated her with great care. At the age of eighteen she began to notice that her mother dwelt much on the subject of the world to come. Sometimes a foreboding entered Ellen’s mind, which proved but too true; for her mother felt that heart and flesh were failing. As yet she knew not how to break the subject to her darling child; but the pallid cheek and the decaying form soon told the sad tale to the alarmed girl, although for a while she supported her hope by depending on the efficacy of medical skill.
Now her mother would press the things of God on her attention with tenfold earnestness, and, while describing the condition of one truly converted, and the breathings of a soul adopted into the family of God, would ask with a faltering tongue, “Oh, my Ellen, have you found the work of the Spirit of God upon your heart? Are you a new creature in Christ Jesus?” Ellen would weep, and say she hoped she was; and the exhausted parent would weep and hope too. But Ellen, when alone, and pondering on those things, felt there was some difference which she could not account for; the deep interest which her mother had always manifested in the Word of God, struck her mind as forming a. strange contrast to her own formal perusal of its sacred pages. Prayer also, which called forth so frequently all the emotions of her mother’s heart, with her was rather repeating prayers than praying; still, she thought herself a Christian, for her conduct was outwardly blameless. Occasionally she would ask herself, “Can all this difference be owing to the diversity of years?” But these inward debates were soon absorbed in the alarm of the rapid advance of disease in her beloved mother. The Bible and Christ were her all. Even her dear child Ellen, in whom she had seemed bound to earth, was given up in faith to the Lord, and resigned to His guardian care; for she knew her end was near.
One morning she called gently to Ellen, who was alone with her, then hastily took her pocket Bible from its place on the pillow, gazed for one moment on it as a most precious treasure, then sweetly smiling on her pale and agitated child, as if to compose her alarm, and suddenly holding to her the Word of God, just uttered, “Here, Ellen, take, take this, faith for sight,” and then fell asleep in Jesus.
The first perception that brought Ellen’s thoughts to her own situation was the chill of the cold hand, which had been clasped in her own, while the bequeathed Bible was held in the other. She pressed the precious volume to her heart, had only strength to call the nurse, and then fell down insensible. On being restored from the swoon, tears came to her relief, and then the anguish of her bereaved condition began to open upon her.
Her guardian, although a professor of religion, was really devoted to gaiety, and Ellen wanted more than she found in all around her. There was no heart to feel as she did, or to share her distress. She was not only a mourner for the dead, hut unhappy in herself, and knew not where to find a remedy.
At length she remembered how her mother, in all her sorrow, had retired to her Bible, and there found consolation; and she devoted a portion of every morning and evening to the study of its sacred pages. Still, as she came to portions marked by her dear parent’s hand, as having been found precious to her soul, Ellen wondered that she found none of that sweet consolation which she knew to have often rejoiced her mother’s heart. In fact, many of its passages appeared to have some meaning which she could not grasp, to make some appeal to the soul which she could not feel; but while musing on the strange interest which her mother found in the holy pages, and at her own strange unconcern, the thought occurred, “Is not this very apathy a part of the burden which the Lord invites me to cast on Him? Must there not be some further opening of the understanding and affections of the heart, wrought by God, which I have not known?” This consideration induced the afflicted girl earnestly to pray to God that what she knew not He would deign to teach her; that His light might be upon His word, and His power upon her soul.
The Lord was now gradually drawing her to Himself, yet it was not till after many weeks that she found peace. How differently did the Scriptures now affect her mind! They were full of interest, because there was the promised work of the Holy Spirit within corresponding with the Word from without; all its disclosures were suitable to her condition; she wanted what they presented. Hungry and thirsty, she came to the Word of God, and was satisfied.
Nearly two years passed in this serene and happy fellowship with God, during which Ellen well sustained the reproach of the cross. Her views of the danger to which the frivolity and vanity she saw around her, exposed the souls of her former companions, led her to endeavor to manifest that danger to them. The taunts, the sneers, the affected pity with which she was assailed, affected her not.
But a greater danger was near. She thought her mountain stood strong, and would never be moved. The duty of watchfulness over her heart was less cautiously observed, and that, too, at a time when a temptation was about to present itself.
She began to attract the attention of Mr. R—, a young man of most amiable disposition and elegant manners, who, with an elder brother, had just succeeded to a respectable competency by the death of his father. He had been educated for the law, but had now retired, content with a sufficiency for the comforts of life; and was distinguished for considerable attainments in literature possessing at once at acute mind, great natural powers, an excellent temper, an unblemished moral character, and a pleasing person.
Mr. R- closely observed Ellen, and thought that if he could laugh her out of what he considered her too great precision of habit, and enthusiasm of religious feeling, she would form a partner in whom his heart would delight. When first he paid his addresses to her, every friend and acquaintance around congratulated her on her happiness. Objection seemed impossible, and Ellen herself could not but be prepossessed in his favor. She thought that nothing was deficient in him to form an exalted character but true religion, of which she felt conscious he knew nothing. Yet there was such a pliancy and gentleness, such a tendency to all that was good, that she thought, “May I not be the means of winning him over to the things of God?” Still, the Word of God, with which she was well acquainted, clearly forbade the union. “Be ye not unequally yoked together with unbelievers,” (2 Cor. 6:14), was a barrier that her mind could not surmount. She knew that others had violated it, and yet were thought Christians still. She mused and hesitated. Conscience told her she ought to break off the acquaintance; but the conflict had now begun with inclination. She could not pray, as before, with the simple desire to know and follow the will of God, and the inward tumult made the attempt irksome. She relaxed in the endeavor, and though she ceased not to pray, yet it was now with a confusion and formality that sometimes told her all was not well within. The parley with temptation advanced till her own affections became entangled; and what with the assiduity of her admirer, and the urgent pressing of her aunt, and of other connections, the affair was hastily, concluded, and Ellen became the wife of Mr. R- before she well thought what she was doing.
(To be continued)

Scripture Study: Why are there Four Gospels?

Why are there four gospels? To give us four distinct views of our Lord Jesus.
What is the leading thought in each?
We can find the answer in each introduction. Matthew presents Him as Son of David — Heir to the throne of Israel, and Son of Abraham — the One to whom all the promises belong. Matthew 1:1.
Mark presents Him as Servant Son. As the servant He has no genealogy. We do not get His birth recorded, it begins with Him about thirty years of age. Mark 1:9
Luke presents Him as Son of Man, a man amongst men His genealogy is traced to Adam and God. Luke 3:38.
John presents Him as Son of God. God manifest in flesh. John 1:1 goes back to eternity, before time began. “In the beginning was the Word” — eternal existence; “and, the Word was with God” — distinct personality; “and the Word was God” — absolute Deity. John 1:14 is His beginning in time.
Each gospel though giving prominence to Him in its own special character also speaks of Him in all the other characters.
Will it not help us in our study of the Word to observe this? We must not try to harmonize them. God has written them differently to bring distinctions before us.
He has used different writers but He held their pens, so that we might know and feed upon the Lord in these different characters. It is wisdom therefore to mark the differences so that we may learn of Him.
Another thing is right to notice. We must not study the Scriptures as we would men’s books to catch their meaning but rather feed upon the Word, and that with a prayerful meditation, that God, who has given us His Holy Spirit to dwell in us, may by His Spirit teach us of His beloved Son. Let us remember as 2 Peter 3:18 says, that growing in grace and in the knowledge of our Lord Jesus Christ go together. It is not by our studying but by God’s teaching that we are to know the things of God. 1 Corinthians 2:12.
If in our study together what is printed is not plain or in enough detail, questions might be asked by letter, and help be given by those the Lord has taught more fully.
We might take up Matthew’s Gospel in our next paper. Please read carefully and with prayer the first chapters so that our minds may be more familiar with the text. Some complain of having poor memories, but reading the Word often is an immense help.
(To be continued.)

Truths for Young Christians: The Wilderness, Part 2

3. Spiritual Refreshment
The third thing in Exodus 15, is the spiritual refreshment Christ provides for true souls who have known what Marah means in the wilderness. In Elim we find the Good Shepherd leading His flock in the green pastures, and by the still waters. Here is an oasis in a desert. And what oasis does Christ provide for His pilgrims in this world? Truly that of Christian fellowship: these are our Elims. What a blessed time we have when a few of us who are really seeking to follow Christ, can get together beneath the sheltering palm trees, and draw fresh strength from the wells of the water of life. Many a one has called these happy Elims, “foretastes of heaven,” as they have enjoyed the— “Sweet bonds that unite all the children of grace.” Alas! that strife and discord should so often mar what our Lord has provided for our rest and refreshment.
4. Wilderness Food
The fourth thing we notice is in the next chapter, and that is the food for the wilderness. Our bread is the manna that is sent down from heaven. In the deliverance from Egypt, Christ is fed upon as the lamb roast with fire—our Substitute and Saviour; in Canaan we get Him as the old corn of the land—our glorified and exalted Lord, and it is worthy of observation that we never find the Israelites of old loathing either of these two foods. It is the manna—Christ in His humiliation and rejection, that is considered “light food.” It is this “bread from heaven” that is the test for each of our hearts today as to whether we have been so truly won by His love as to esteem a path of rejection with Him, better than all the “leaks and cucumbers” of Egypt.
Surely, too, we may learn an important wilderness lesson from the fact that this precious bread was gathered freshly every morning before the sun was up, so those find now who spend “an hour with Jesus” before the bustle of daily life has begun; that the sweetest and most strengthening food is then gathered and stored. As has been so well said by another, “If I sincerely desire to grow in the divine life — if my one grand object is to be assimilated and devoted to Christ — I shall without doubt seek continually that character of nourishment which is designed by God to promote my spiritual growth. It is plain that a man’s acts are always the truest index of his desires and purposes. Hence, if I find a professing Christian neglecting his Bible, yet finding abundance of time — yea, some of his choicest hours — for the newspaper, I can be at no loss to decide as to the true condition of his soul. I am sure he cannot be spiritual — cannot be feeding upon, living for, or witnessing to, Christ.
5. Streams in the Desert
The fifth point we may observe is the refreshing stream that pours out of the riven rock in accordance with the well known passage in John 7:37, “If any man thirst, let him come unto Me and drink.” Surely, if in the manna we have a picture of the humbled Christ as our food, here we have the indwelling Spirit who is with us throughout our wilderness journey, one of the blessed results and fruits of the death and glorification of Christ (John 7:39). The rock is Christ (1 Cor. 10:4). The waters, doubtless, here as elsewhere, are typical of the Holy Spirit, who is the refreshment and source of power and blessing, not only for ourselves, but others, down here. “Grieve not the Holy Spirit of God, whereby ye are sealed unto the day of redemption.” He is here to testify of Christ, but He can only testify to ready and listening ears. He is here to guide us into all truth, but only those who have willing feet and subject hearts. This water, too, unlike that in Exodus 16, is not for our own refreshment alone, but is to run out from us, so that we ourselves, as filled with the Spirit, are to be as streams in the desert. Thus far we have traced the believer — a song of joy in his mouth — the fellowship of Christ’s sufferings known to his heart — enjoying Christian fellowship — feeding on a humbled Christ — and refreshed by an indwelling Spirit.
6. Wilderness Conflict
Now we come sixthly to Amalek, a picture of the flesh energized by Satan, who is ever walking about, ready to snap up any who are weak and ready to halt. We feel that it is quite impossible in the limits of a short paper to do more than just touch on this most important theme. It will be noticed that the victory in this case (Ex. 17) was obtained by two means; the one, the intercession of Christ on high, and the other, the resistance in the power of the Spirit (Joshua) down here. Now both of these are necessary if we are to overcome our adversary. In Peter’s case, the intercession of Christ that his faith might not fail, was fully answered, but on account of the want of his active resistance against the enemy, he failed. The resistance down here would be valueless, were it not for the uplifted hands on high; at the same time we are to resist the Devil, and the Spirit in us lusts (or fights) against the flesh, that we may not do the things that we would. Christ will not fail in His part, blessed be His name, but how often do we fail in practically resisting the assaults of the enemy.
7. Water for Defiled Feet
The last, or seventh thing we have to notice is the provision made in case of defilement in the wilderness journey. We refer to Numbers 19, which answers in type to 1 John 1:9. This is cleansing by water, not by blood, but it is water which contains and brings home to our hearts the memorials of the death of Christ, (the ashes of the heifer) teaching us that restoration to communion, after getting astray, is not by a fresh application of the blood of Christ, (which is quite an unscriptural thought) but a bringing home to our hearts by the Word of God (the water, see John 13) the power of the death of Christ, which we, in our self-will had forgotten. It is thus that Christ Himself, in His perfect love, washes our feet when defiled with the wilderness journey.
We leave these blessed subjects with these few words, but enough has been said to show something of the perfect provision of our Lord for the wilderness journey, and thus cause our hearts to flow out in fresh praise to Him.
Just think the whole subject over, beloved reader, and you will find that Christ is with us at every step. We meet Him first in Egypt, in the blood of the Lamb; next, in the delivering power of the Red Sea; next, in the power of His cross; then, in His gracious provision for our refreshment; next, as the Manna; then, as the Rock; then, as our Intercessor up on high; and, lastly, in His wondrous love in following us when we go astray, and restoring our souls by the washing of water by the Word. The end of all being to meet His own glorious Self on the cloud when all the journey will be over forever, and we shall praise forevermore the grace that has carried us on eagles’ wings, and at last brought us home to Himself.
(Continued from Volume 1)

From Egypt to Shiloh: Part 1, God is Love

“But go ye now unto My place which was in Shiloh, where I set My name at the first” (Jer. 7:12).
“For where two or three are gathered together to My name, there am I in the midst of them” (Matt. 18:20).
Before we can understand the instruction given to us in these last days, contained in the deeply interesting subject of Shiloh, we must take a brief view of the dealings of God with His people Israel, to whom He appointed Shiloh as the only place where He set His name. Their history, written for our instruction, is a type or figure of each believer’s history, and salvation.
“God is Love.”
If we read Exodus 2:23; 3:1-10, we see the condition of the people in cruel bondage and slavery, an exact picture of our condition under the cruel bondage of sin and Satan. Their cries and groans came up unto God. Do you remember the time when it was thus with you? Think of the slave that can by no means escape from bondage. Such is the condition by nature of every man, whether he knows it or not. And mark, the source of their deliverance was not in themselves. The source of our salvation is the love of God. God is love. He came down to deliver. He sent Moses to deliver. “God so loved the world, that He gave His only begotten Son, that whosoever believeth in Him should not perish, but have everlasting life.”
Yes, God sent Moses— God sent His Son. God is love. They believed Moses. “Then they bowed their heads and worshipped” (Ex. 4:31). This, then, is the first mark of a quickened soul. Have you believed the love of God in sending His Son? This bows your head, and you worship.
The next step is the earnest desire to be gone, to escape from the slavery of Satan and sin. Did they escape through believing the love of God alone? No. Have you? No. Their case became worse as to experience. So has yours. So did mine. They were now put on the principle of more work- to make bricks without straw. They could not, and were beaten because they did not. And you, were you not put under the law of God? They had no straw, and you had no strength. And you found that word true, “Cursed is every one that continueth not in all things which are written in the book of the law to do them.” And you had no strength to do them. Paul describes all this in Romans 7:14-24. Poor, wretched Israel! Moses said, “Neither hast thou delivered thy people at all” (Ex. 5:22-23). And you have believed the love of God, and tried to keep the law of God, but are you delivered from sin? What do you say? “O wretched man that I am! Who shall deliver me?” (Rom. 7:24).
The next or third thing we find in this picture is the promises of God. (Read Ex. 6:1-9.) Do read them. Could God give more precious promises? No. Did the promises deliver? Not in the least; “They hearkened not unto Moses for anguish of spirit, and cruel bondage” (Ex. 6:9) and you have tried to lay hold of the promises. Have they delivered you? No. You say, They do not comfort me. Why? I am such a sinner, such a slave. And this makes you unspeakably miserable.
Now the fourth thing in this picture is the wondrous kindness of God in His providential care of Israel during the plagues of Egypt. From Exodus 8:22, we have the most tender care in sparing His people. But they were all still in cruel slavery.
What is to be done? We too have believed the love of God; we have tried to keep the law of God; we have tried to lay hold of the promises, and to trust the providence of God; and yet no real deliverance from sin — from the cruel slavery of doing what we hate. We are at our wits’ end— we have come to the end. We do not know what to do. Thank God, we have got to the end of ourselves; we can do no more.
All now is of God, we will see what He has done. What do we get in this picture? A Lamb. Every man’s need is met by a lamb. The lamb must now be put forth; the lamb must be killed; blood must be sprinkled; the lamb must be eaten; God must see the blood. God says, “When I see the blood, I will pass over you” (Ex 12:13). Oh, how little redemption is understood. Such is sin in God’s sight; He must send His Son, the Lamb of God. He must be killed, His blood must be shed. And it is what God sees in that blood, it is God’s estimate of that blood, which shelters from judgment. We must have redemption through that blood. There is no other means of pardon and eternal salvation.
Now many reach this point who never go beyond, and are utter strangers to all the teaching of Shiloh. Yea, they are not even delivered from Egypt; they have not yet understood the Red Sea. Until Israel had passed through the waters of the Red Sea, they were sorely troubled about Pharaoh and the host of the Egyptians behind them (Ex. 14). It is so with you, if you have only been brought so far on as being sheltered by the blood. Often you are sorely troubled about past sins, especially sins since you believed the love of God. Does not Satan bring them after you like the armies of Egypt?
A Christian can never really sing in his heart until he knows he is brought out of his old state through death — death written upon him and all his past. Ah, then the Egyptians are all dead on the shore. It is a wonderful thing to reckon ourselves dead with Christ.
But before we reach Shiloh there are two things that must be known — “out of” and “into.” We learn what we are brought out of at the Red Sea. We get the picture of what we are brought into, when we have crossed the Jordan. Now between these lies the wilderness with all its lessons and experiences. But in the wilderness there is not a word about Shiloh. Let it be borne in mind, none can enter into the lessons of Shiloh but those who have not only been brought out of the old creation, typified by Egypt, but also brought into the new creation, into the heavens, as typified by the passage of the Jordan.
It is most needed to learn the lessons of God’s provision for us in the wilderness and by the offerings of Leviticus. How every failure has been met by the one offering of Christ; yea, how all the claims of divine righteousness have been met to the glory of God. He who came to do the will of God, could say, “I have glorified Thee.”
(To be continued)

The Coming and Reign of Our Lord Jesus Christ: The First Resurrection, Part 3

“THE FIRST RESURRECTION.”
Continued from Volume 1.
“Christ,” then, is “the first fruits” (1 Cor. 15:23); and “the first resurrection” embraces not only those who are asleep when He comes for His own in the air, but also the two classes of saints martyred during the awful troubles and great tribulation that will take place between that event and His manifestation in power to reign, the whole of the saints in fact who go into death from Adam to the millennium. These future martyrs will yield up their lives, like many saints of old, that they might obtain a better resurrection (Heb. 11:35.) The rest of the dead, that is, those who die in their sins, as we have already seen, remain in the grave until the close of the millennial kingdom, and the little season the end of the world when they shall come forth to the resurrection of judgment (John 5:29; Rev. 20:5.)
Daniel 12:2, has presented a difficulty to some: “And many of them that sleep in the dust of the earth shall awake, some to everlasting life, and some to shame and everlasting contempt.” If you read carefully the first verse you will see that it is not a question here of literal resurrection, but speaking figuratively of the restoration of the Jews, the deliverance of some out of the time of trouble—great tribulation—judgment being the portion of others. Notice that the words thy people occur twice (Dan. 12:1). This is not the only scripture where the restoration of Israel is spoken of thus. In Ezekiel 37, they are compared to dry bones coming to life, being brought up out of their graves, and placed in their own land (Ezek. 37:1-14). Also in Psalms 68:22: “I will bring My people again from the depths of the sea.”
“But some men will say, How are the dead raised up? and with what body do they come? Thou fool that which thou sowest is not quickened, except it die; and that which thou sowest, thou sowest not that body that shall be, but bare grain, it may chance of wheat, or of some other grain; but God giveth it a body as it hath pleased Him, and to every seed his own body.... So also is the resurrection of the dead. It is sown in corruption; it is raised in incorruption; it is sown in dishonor; it is raised in glory; it is sown in weakness; it is raised in power; it is sown a natural body; it is raised a spiritual body. There is a natural body, and there is a spiritual body” 1 Cor. 15:35-44.
With God all things are possible, and He who by His own mighty power raised from among the dead our Lord Jesus, will surely accomplish this mighty act, and raise all His own who sleep, to be with and like His Son in glory (1 Cor. 6:14.) It matters not by what means or where God’s people fall asleep, whether their bodies shall go into the grave or the ocean’s depth, “We know, that if our earthly house of this tabernacle were dissolved, we have a building of God, an house not made with hands, eternal in the heavens” (2 Cor. 5:1.)
How blessed for the Christian, as he stands by the open tomb, and sees lowered into it a coffin containing the corruptible remains of some dearly loved one who has fallen asleep in Christ, to be enabled to look beyond this scene of sorrow and death, and to know that the departed is “absent from the body and present with the Lord” (2 Cor. 5:8) To know too that at any moment the welcome sound of the Master’s voice may be heard in the air, when every sleeping and every living saint shall be caught up, in a moment conformed to His blessed image; so to be “forever with the Lord.” It is then that “shall be brought to pass the saying that is written, Death is swallowed up in victory. O death, where is thy sting? O grave where is thy victory? The sting of death is sin; and the strength of sin is the law. But thanks be to God, which giveth us the victory through our Lord Jesus Christ” (1 Cor. 15:54, 57.)
How precious to have as the object of our hearts in the glory of God that Blessed One, who could say, “I am the resurrection and the life;” who could weep as a sympathizing man at the grave of Lazarus, but who could display the mighty power of God, for He was and is the Son of God, in raising him from the dead.
A Jewish sect, called the Sadducees, in the days of Christ on earth denied resurrection altogether. Our Lord’s answer to a question put by them is well worthy of note in considering this subject. He saith unto them, “The children of this world marry, and are given in marriage; but they which shall be accounted worthy to obtain that world, and the resurrection from the dead, neither marry, nor are given in marriage. Neither can they die any more; for they are equal unto the angels, and are the children of God, being the children of the resurrection. Now that the dead are raised, even Moses showed at the bush, when he calleth the Lord the God of Abraham, and the God of Isaac, and the God of Jacob. For He is not a God of the dead, but of the living; for all live unto Him” (Luke 20:34-38.)
Paul also in writing to Timothy, warns him to “shun profane and vain babblings; for they will increase unto more ungodliness; and their word will eat as doth a canker; of whom is Hymenaeus and Philetus: who concerning the truth have erred, saying that the resurrection is past already; and overthrow the faith of some” (2 Tim. 2:16-18.)
I cannot close writing on this wonderful subject without asking you my dear reader, If death were to overtake you this day, Are you ready to go? Would you be amongst those who would come forth at the first, the glorious resurrection? Or are you still in your sins, still under the judgment of God? If summoned from this scene in the latter condition, your portion must be to come forth at the resurrection of judgment, and to be cast into the lake of fire. There is only one way of deliverance from such a doom, and that is by faith in the Son of God, who was judged on the cross as the sin-bearer (2 Cor. 5:21.)
How bright the resurrection morn
On all the saints will break!
The Lord Himself will then return,
His ransomed church to take.
We cannot linger o’er the tomb;
The resurrection day
To faith shines bright beyond its gloom,
Christ’s glory to display.

Correspondence: Who were the "Sons of God" in Gen. 6:2,4?

Question 27: Who were the “Sons of God” in Genesis 6:2, 4? W. M.
Answer: There are three classes of angelic beings mentioned in the Scriptures.
1st. Holy and elect angels, Luke 9:26; 1 Timothy 5:21.
2nd. Satan and his host of demons who fell before man’s creation. Ezekiel 28:13-15.
3rd. Those who sinned or left their first estate, 2 Peter 2:4; Jude 6. And are now confined in everlasting chains of darkness awaiting the judgment of the great day.
As the word here translated “Sons of God” is never elsewhere used for men, it is generally accepted that it means this third class of angels.

Inspiration of the Scriptures: Examples of the Divine Authority of the O. T. Scriptures, Part 2

EXAMPLES OF THE DIVINE AUTHORITY OF THE OLD TESTAMENT SCRIPTURES.
In Hezekiah’s reign, there was remarkable blessing on their turning to the authority of the sacred writings.
They soon discovered that they had not kept the solemn and important feast of the Passover, “for a long time in such sort as it is written.” We are told, therefore, that the men of Judah had given them by God one heart to do the commandment of the king and of the princes by the word of Jehovah. Moreover, Hezekiah appointed morning and evening burnt-offerings, and the burnt-offerings for the Sabbaths and for the new moons, and for the set feasts, as it is written in the law of Moses. They kept the Passover and the feast of unleavened bread seven days with great gladness, and then “kept other seven days with gladness,” and offered sacrifices; “so there was great joy in Jerusalem, for since the time of Solomon, the son of David, king of Israel [nearly 300 years], there was not the like in Jerusalem” (2 Chron. 30:2, 5, 15-26).
The wonderful revival in the reign of Josiah, king of Judah, may also be traced to the practical acknowledgment of the divine authority of the Holy Scriptures. It was brought about by Hilkiah, the priest, finding in the house of Jehovah “a book of the law of Jehovah given by Moses. And Hilkiah answered and said to Shaphan the scribe, I have found the book of the law in the house of Jehovah.... And Shaphan read it before the king. And it came to pass when the king heard the words of the law that he rent his clothes.” The reason was that he learned from these writings that they were justly exposed to divine wrath, and the curses written in the book, because of their sins in having forsaken Jehovah their God, and having burned incense to other gods. They, therefore, bowed at once to the authority of the sacred writings, and kept the Passover according to the ordinance, “as it is written in the book of Moses,” and it was accompanied with God’s abundant blessing. They were so exercised by the authority of Scripture about it, that we read that the king’s commandment was, “kill the passover, and sanctify yourselves, and prepare your brethren, that they may do according to the word of Jehovah by the hand of Moses.” We are further told that the evil, and “abominations that were spied in the land of Judah, and in Jerusalem, did Josiah put away, that he might perform the words of the law which were written in the book that Hilkiah the priest found in the house of Jehovah. And like unto him was there no king before him, that turned to Jehovah with all his heart, and with all his soul, and with all his might, according to all the law of Moses, neither after him arose there any like him.” We are told also, “there was no passover like to that kept in Israel from the days of Samuel the prophet” (2 Chron. 34:14-19, 24; 35:6, 12, 18; 2 Kings 23:24, 25).
The return of the Jews from their captivity in Babylon was also strikingly marked by their submission to the authority of the written law of Jehovah. So truly did Ezra recognize the divine authenticity of the Scriptures, that we are told that “Ezra had prepared his heart to seek the law of Jehovah, and to do it, and to teach in Israel statutes and judgments.” We read also, that when they were gathered together as one man in Jerusalem, they “builded the altar of the God of Israel, to offer burnt offerings thereon, as it is written in the law of Moses, the man of God.... they kept also the feast of tabernacles as it is written.” Again, when the temple was finished, they dedicated the house of God with joy, they offered a sin-offering according to the twelve tribes of Israel, “and they set the priests in their divisions, and the Levites in their courses, for the service of God which is at Jerusalem, as it is written in the book of Moses.” They also found it written in the law, that the feast of tabernacles should be kept. “And all the congregation of them that were come again out of the captivity made booths, and sat under the booths; for since the days of Joshua, the son of Nun, unto that day had not the children of Israel done so. And there was very great gladness” (Ezra 3:2, 6:15-18, 7:6, 10. Neh. 8:17).
Do not all these examples warn us most solemnly against disobedience to the written Word, as well as encourage us to bow to its divine authority? Surely, “to obey is better than sacrifice,” and to honor God is always the path of blessing. It is well also to notice, that in Israel’s history, reviving and restoration began in individual exercise before God, and personal cleaving to His Word as demanding willing subjection; and, from individual turning to God in this way collective blessing followed. No doubt it is the same now. Wherever souls turn to God in earnest prayer and supplication, and therefore bow to His Word with the hearty desire to carry it out, there will always be found the marked blessing of God. Nothing less is the path of faith. Every other way is the fruit of unbelief, and cannot please God. Wherever true faith in God is in exercise, there will be constant appeal to what is written for our instruction, and its authority will be always final and conclusive. Surely to such the Scriptures are “the oracles of God.”
“I love the sacred book of God,
No other can its place supply;
It points me to the saints abode,
It gives me wings, and bids me fly.
Sweet book! in thee my eyes discern
The image of my absent Lord;
From thine illumined page I learn
The joys His presence will afford.
In thee I read my title clear
To mansions never to decay:
My Lord! O when wilt Thou appear,
And take Thy prisoner far away?”
(Continued and to be continued).

Unscriptural Marriage: Or, the History of Ellen, Part 2

Ellen R-, as we must now call her, was soon undeceived as to the hopes with which she had quieted conscience before her marriage. Her house which she had fondly thought to preserve like the dwelling of Lazarus, Martha, and Mary, where Jesus oft resorted with His disciples, presented no token of being graced with His presence, or of being honored with His blessing. Family prayer was out of the question with a master who was insensible of its duty, and undesirous of its accompanying influence—who thought this delightful service to belong only to ministers of religion. The day began there as if they had nothing to do with God; so it passed throughout, and so it closed. Ellen sometimes mentioned it with desire to her husband; but every allusion to the holy practice was answered with some general evasion, some playful banter on “nun-like purity she had received in her solitude,” and “which should now be exchanged for conformity with the practices of social life,” When she sought to discourse more seriously on these topics, she found the subject was irksome to her husband, and was obliged to suffer the conversation to be diverted into some other channel. Indeed, on all other points it was so interesting, that, before she was aware, her mind was engaged on matters more congenial with his taste, and the subject dropped. Thus repeatedly foiled in the attempt to engage his thoughts on the things of God, the effort was more seldom renewed, and at length it ceased for a while altogether, in a kind of understood, though not expressed, arrangement, that each was to pursue unmolested the course most agreeable to inclination.
But on Ellen a most seductive influence was being exercised in a thousand forms; all her husband’s friends and connections were as averse to heart religion as himself, although they paid it homage as to outward forms. Many of them were of a literary or scientific character; and their intelligence, with readiness of expression, won her towards their habits, and sometimes induced her compliance with worldly practices, at which, before her marriage, she would have startled. The few pious friends whom she retained found their converse so little acceptable to her husband, and the general conduct of her household so repugnant to their views of propriety, while she herself had so far immerged into the torrent of worldliness, that they fell off one by one. And here, oh, what a sad reverse was experienced to her former joy and peace! She felt, when attempting to retire for prayer, as entering into the presence of One whom she had displeased—whom she was continually displeasing. There was no sacred fervor in prayer, no filial resting on her God and Father’s love and promise; self-examination was a torture, and was less and less attempted, till it was given up altogether. The most fearful doubts preyed upon her mind as to the reality of all she had experienced of the work of divine grace; and these doubts were continually called into exercise, and strengthened by the converse of almost all with whom she mixed. She felt that she had sustained a loss of happiness which worlds could not compensate.
In this condition she continued almost nine years, during which period she had become the mother of four children. Jane, the eldest, now nearly eight years old, had always been delicate in health, and was remarkably quiet, affectionate, and humble in her demeanor. She took little interest in childish play, and appearing always to prefer her mother’s society, she became her almost constant companion when at home. The Scriptures attracted her greatest attention, and she would sometimes ask such affecting questions from them as brought tears into her mother’s eyes. The dear child would notice them, and weep with her, then frequently endeavor to kiss the tears away, and ask their cause. Alas! the unhappy parent could not explain to her beloved child how grievously matters stood between her soul and God; but she would retire to pray if yet she might possibly once more rejoice in the light of God’s countenance, from whom she had departed. Generally, the dear child would offer her hand so affectionately that refusal was impossible; she became the companion of her retirement, and kneeling by her side, would silently unite in her solemn petitions. From such affecting causes Jane was very dear to her, and she cherished the hope that her after life would not witness those wanderings from God, and those bitter sorrows which had been her own portion.
Anne, giddy, playful little Anne, then five years of age, was her father’s darling, and indeed partook much of his natural vivacity, as well as of his likeness. Henry was three years old, a fine, healthy, promising boy; and William, only twelve months, had but lately been weaned, and began to run alone. At this period Ellen was seized with a violent fever, and as it soon assumed infectious and alarming appearances, it was determined to remove the children, who had from the first been kept separated from her, to an adjoining village; but before this could be accomplished, the affectionate little Jane, who could not bear the thought of departing without seeing her mother, left the nursery unobserved, and gently entered her beloved parent’s room. Ellen had just been left in a gentle doze, and the dear child had, after silently watching the panting breath and flushed countenance for a few moments, knelt down by the bedside to pray for her sick mother. In this posture did Ellen behold her daughter, and following the first impulse of nature, would have clasped Jane to her beating heart; but immediately recollecting her own condition, she rang the bell, and could only give the child her blessing, before she, sobbing with anguish, was removed. Jane went with the rest to the village; but that same evening she sickened, and in four days she died.
It was three weeks before Ellen was able to have the sad tidings broken to her; but the shock was less than had been feared, for the near prospect of eternity had greatly altered her state of feeling; she heard it, and bowed submissively to the will of God. Yet, when able to leave her room, the company of her Jane was everywhere missed and her image recalled by every object. On this account, as well as for change of air, an immediate temporary removal was advised to the south of Devon; and as soon as the journey could be borne, her husband took her with the family to lodgings prepared at the lovely watering-place, Torbay.
They were now removed from worldly company, and a continual succession of parties, which for years had been the bane of spirituality of mind. This was the time and opportunity for thought, which she so greatly needed, and while here, the first hope of recovering the peace and happiness, and of walking with God, entered her heart; especially did she date the expectation from a meditation one evening, while seated on a bench provided for invalids on the “Rock Walk” of Waldon Hill.
Ellen almost unconsciously repeated to herself, “Return unto thy rest, O my soul.” Hope fixed on the idea, and from that moment she never abandoned the cheering thought of restoration to communion with God. This change gave a fresh impulse to both body and mind; health again bloomed in her countenance, and she decided to return home.
(Continued and to be continued).

Scripture Study: Matthew 1

Matthew 1:1 is the subject of this book. It is about Jesus Christ, son of David, son of Abraham.
As son of David, He is heir to the throne and kingdom of Israel; as son of Abraham, all the promises are fulfilled in Him. He is the great King foretold in such scriptures as Isaiah 9:6, 7; and “All the promises of God in Him are yea, and in Him Amen, unto the glory of God by us.” 2 Corinthians 1:20.
Matthew 1:2-17 trace His genealogy to Joseph, His reputed father, who was heir to the throne; this gives Him the legal title. It is clearly traced through the time of Israel’s captivity. Verse 17 divides it into three periods of fourteen generations, witnessing God’s perfect ways.
In the history, there are four women mentioned, which show man’s sinful ways, but God’s wonderful grace and miraculous power that has brought a clean thing out of the unclean. Judah’s sin with Thamar is the first (Matt. 1:3); then the grace that brought Rachab the harlot into the royal family (Matt. 1:5); and Ruth the Moabitess also, shows us God’s grace reaching over what the law has said (Deut. 23:3) in giving blessing to the believing stranger.
Then king David’s sin with the wife of Uriah, tells us that while man in his best estate is altogether vanity, God’s grace has provided One to be a Saviour.
The Lord’s genealogy in Luke is traced through Mary, and she is also of the royal line of David, but descended through Nathan David’s son. Joseph comes through Solomon, the king, thus the Lord Jesus is proved legally, and by natural descent the heir to the throne.
Matthew 1:18-25. In Luke we find God speaking to Mary, but here in Matthew all He says is to Joseph as son of David; and this is quite consistent with what we have looked at. Joseph was a pious man, and Mary was espoused to him, but before they came together, she was to be the mother of Jesus. The Holy Ghost’s power had done this.
While Joseph was pondering what he should do, an angel of the Lord graciously explained to him Mary’s condition, and calls her “Mary, thy wife,” and “that which is begotten in her is of the Holy Ghost.” “She shall bring forth a son and thou shalt call His name Jesus, for He shall save His people from their sins.” It was the fulfillment of the prophecy in Isaiah 7:14. “Emmanuel,” that is, “God with us.” What a wonderful person! God come down to earth the deliverer of His people from the state of sin they had fallen into. Joseph believed God, and did as the angel told him to do. This is the simple way of faith; it believes and acts on the word of God, and when her son was born, “he called His name Jesus.”
Israel looked for One to come, a great King to deliver them from their enemies, but this One was to do a greater work, and not only deliver from human enemies, but deliver them from the slavery of sin and Satan, and set them in eternal blessing, but this was not yet unfolded. He who would do this work was come. How happy are they that know His name.
There is no other name than Thine
Jehovah-Jesus! name divine;
On which to rest for sins forgiven,
For peace with God, for hope of heaven.
Name above every name, Thy praise
Shall fill you courts thro’ endless days,
Jehovah-Jesus! name divine,
Rock of Salvation Thou art mine.
(Continued)

Truths for Young Christians: Waiting, Part 1

A careful study of the attitude of the believer towards his absent Lord, as spoken of in various parts of the Word of God, will show that it may well be divided into three distinct aspects. He waits or watches, he walks and works.
We might distinguish between waiting and watching, as it is possible for one to be in a sleepy spiritual state and might be said to be waiting but you could not say watching. But the proper character of waiting involves watching, so we connect them together in this article.
The heart waits and watches for Christ, his feet walk and his hands work; thus his heart, feet and hands are all occupied for the Lord in view of His coming. These distinctions are by no means arbitrary; on the contrary, it will be found that wherever the coming of the Lord is spoken of, it is in connection with some one of these aspects. In Matt. 25, we get the waiting and the working; in Luke 12, principally the waiting; in Thessalonians and Revelation all three; in 1 Corinthians 15, the work; and in 1 John 3, rather the walk.
Waiting Walking and Working
In the book of Revelation, we observe that the Lord says three times over, “Behold, or Surely I come quickly.” The first is in Revelation 3:11 and is manifestly in connection with the Christian’s walk; the second is in Revelation 22:12, and will be seen to be connected with work here and rewards hereafter; the third, as is so well known, is in Revelation 22:20, and is solely in connection with the heart waiting and watching for Christ.
In 1 Thessalonians, again, it is beautiful to observe the closing thoughts of the first three chapters. In 1 Thessalonians 1, we get the Thessalonians waiting for God’s Son from heaven. In the close of 1 Thessalonians 2, we find the Apostle speaking of this same coming in reference to his work amongst them, and the reward which he will get in them in the glory; while in the close of 1 Thessalonians 3, and beginning of 1 Thessalonians 4, we find the return of the Lord Jesus Christ with all His saints is placed in connection with a holy and God-pleasing walk.
Having, then, shown that these are three scriptural distinctions, let us for a moment consider the Christian as waiting and watching for Christ.
It Is the Heart That Waits for Christ
Where this is spoken of in 1 Thessalonians 1, we may notice one thing most particularly, and that is that those who thus earnestly looked out for Christ knew so little about the doctrine of the way and manner of His coming, that it had to be made a subject of a special revelation in the close of 1 Thessalonians 4. But they did not wait till they had that chapter before they looked for Christ. We are thus clearly taught that waiting for Christ is not a matter of intelligence but of heart. And this, beloved reader, shows us where we fail. For in these closing days God has wonderfully opened up His Word to us, so that there are thousands now who know a very great deal more of the Lord’s coming than did these Thessalonians of old; But of how many of all these can it be truly said, They are waiting and watching for Christ?
How We Are to Wait
It is an important and deeply interesting fact that our Lord has detailed the precise attitude in which He wishes us to await His return, so that any uncertainty is not possible. In that wonderful passage in Luke 12, when Jesus seeks to prepare the heart of his disciples for His nearing departure, He also speaks of His return. Those who wait for Him during the long dark night are to be characterized by girded loins, and trimmed lamps, and they themselves are to be like men who wait for their lord. And then follows that wondrously blessed promise that those who are thus girded and watching here, shall there sit down at table while the Lord rises, girds Himself, and serves them!
Now the girded loins, in other words, are the Christian’s walk carefully keeping his garments from the defilement around, and declaring by his tightly girded dress his position as a traveler, and as a pilgrim; while the lights or lamps burning, speak of his work, and his testimony in this world for Christ, both of which we hope to touch on in future papers, so that it is the emphatic “Ye, yourselves, like unto men who wait for their Lord, “that most occupies our thoughts here.
Like Men Who Wait for Their Lord
One thing about this waiting is clear. Although it leads to self-examination and carefulness in walk and ways, it is a waiting with joy, not with fear. While a certain solemnity surely attaches to the thought of that sublime moment when we first behold our Lord, He would have our hearts anticipate it with joy. In order to do this, it is clear we must know something of Christ, for it is certain that it is just in proportion as we know Christ, not truth, that we long to see Him. And this leads us in many ways to walk more worthy of Him; as our hearts get more occupied with Christ, insensibly one thing after another stands revealed in its true light. We distinguish the substance from the shadow, our eyes get cleared from the mists around, our hearts freer for Him, our lives more separate, more devoted, more unworldly; in short, altogether we become like men who wait for their Lord.
We all know how earnestly, if a king or president is about to visit some district, he is waited for. Men rise early, leave their homes and business, travel great distances at considerable expense, push for the best place, or pay well for it, and then stand patiently waiting, hour after hour, And all for what?—to see him pass.
Surely all this is gratifying to such and shows what a place he holds in the hearts of his people. And did we but think how grateful it is to Christ to be waited and watched for, we should not find ourselves put to shame by men of the world. “Blessed,” says the Lord, “are those servants, whom the lord when he cometh shall find watching.”
What a sad tale it would tell a king or president to pass through empty and deserted streets and find none caring to see him. And oh! shall we not regret being found careless and asleep, giving our Lord pain when we might have given Him pleasure at His return?
(Continued and to be continued).

From Egypt to Shiloh: Part 2, Shiloh

If we now read carefully Deuteronomy 12 beginning at Deuteronomy 11:31, we shall see the immense change that would take place, when they had crossed the Jordan, and were in actual possession of the land, and had rest in all that God had given them. All idolatry was to be destroyed. Then God would choose out a place. “But unto the place which the Lord your God shall choose out of all your tribes, to put His name there, even unto His habitation shall ye seek, and thither thou shalt come.” To that place they were to bring all their offerings. There they were to worship and eat, and rejoice before the Lord. All this is solemnly repeated. They were not to do there as they had done in the wilderness. “Every man whatsoever is right in his own eyes, for ye are not yet come to the rest and to the inheritance, which the Lord your God giveth you. But when ye go over Jordan, and dwell in the land which the Lord your God giveth you to inherit, and when He giveth you rest from all your enemies round about, so that ye dwell in safety: then there shall be a place which the Lord your God shall choose to cause His name to dwell there.”
What is the antitype or answer to all this? Can you say, It is true of me; after all my struggles in Egyptian bondage to sin and Satan, God brought me out by the blood of the Lamb? Through Pharaoh’s overthrow I see now, he that had the power of death is destroyed. Can you say, I am dead with Christ? Can you say, God has secured His own glory, and provided for all my needs, by the offering of Christ? Can you say, As to my conscience, the whole question of my sins is settled for eternity: I have eternal redemption; He hath by His one offering perfected me forever, in unchanging continuance?
Now take the epistles. Look at Ephesians 1. Look at the heavenly land, so to speak, our God and our Father hath given us. Here you see the believer clean over Jordan; that is, brought into the land God hath given him in Christ, in the heavenlies — out of Egypt, as you may read in Colossians 1:12-14, but into the heavenlies in Christ, in Ephesians in the one case really across Jordan; dead with Christ, and risen with Him (Col. 2:3), in Ephesians, right up in the heavenlies in the Beloved, in the same favor in the Beloved. Can you say, All this is true of me? Is God so good to you? In His free grace has He given you all this to enjoy in His own love in Christ? Oh, have you taken possession? Have you rest in the unbounded love of God, as thus revealed? Mark, until this is the case, you will be like Israel before they crossed the Jordan: as to all church matters, you will do what seemeth good in your own eyes. You look abroad, and you see many places that men have built, and placed their names, and you will choose for yourself, and not knowing the Lord’s mind. You will do what you think best, in what is called Liberty of Conscience. We will now pass on to Shiloh, Joshua 18:1-10.
The land had now rest from war. Israel were in the land. The Jordan had been crossed, the victory had been won. Surely this points to the resurrection of Christ from the dead. He was risen from the river, from among the dead, but all believers are risen with Him, as all Israel passed over dry shod. It is as risen with Christ that we are in possession of the heavenlies, the true Canaan. The risen Christ is Shiloh.
The Meaning of the Word Shiloh
In Scripture this word Shiloh has two meanings. It is the name of a person in Genesis 49:10. “The scepter shall not depart from Judah, nor a lawgiver from between his feet, until Shiloh come; and unto him shall the gathering of the people be.” The meaning of Shiloh here as applied to a person is “whose it is,” or “whose right it is.” Surely that person is Christ the Lord. It was when He had accomplished the work of redemption, when God had raised Him from the dead, He was the One, the only One, “whose right it is” to gather to Himself. Whether we look at the church of God, or the future gathering of Israel and the millennial nations, or we look up to heaven in Revelation 5, and see one in the midst of the throne, there is only one whose name is Shiloh; to Him, “whose it is,” shall the gathering of the people be. He alone is the Shiloh. He alone is worthy. Worthy is the Lamb. Oh, let us never forget His words—what meaning they have— “For where two or three are gathered together to My name, there am I in the midst of them” (Matt. 18:20). And this brings us to
Shiloh As a Place
The Lord, the true Shiloh, first gathered to Himself, or formed the assembly, by the Holy Ghost at Pentecost (Acts 2). Thus Joshua 18, is a type of Pentecost. The meaning of “Shiloh” as a place is “peaceful tranquility;” the Lord’s own place, the place of peaceful tranquility. What was it but this when He arose from the dead? What were the first words of the risen Shiloh to His disciples assembled together? “Peace be unto you.” And He showed them His hands and His side He had finished the work; He had made peace by His blood-eternal, perfect peace—it is His right to speak peace. Have you heard His voice speaking to you?
Shiloh was the place where Jehovah was pleased to place His name at the first. “And the whole congregation of the children of Israel assembled themselves together at Shiloh and set up the tabernacle of the congregation there.” Joshua 18:1. What a picture of Pentecost! The true One, “whose right it is,” was come, and all believers were gathered unto Him. They were not gathered to Peter, but to the Lord. Peter was the preacher, but Christ, the exalted Lord, was the Shiloh to whom all were gathered.
Thus in the beginning all was done at Shiloh, before the Lord. True there were many who had not as yet received their inheritance, but Joshua was not indifferent: he says, “How long are ye slack to go to possess the land which the Lord God of your fathers hath given you.” Joshua 18:3. Missionaries were sent out, so that all might enjoy their possession. They went out from Shiloh, and returned to Shiloh, and the land was given to them at Shiloh. “Then said Jesus to them again, Peace be unto you: as My Father hath sent Me, even so send I you.” John 20:21 Thus He sent them out as missionaries to make known peace and forgiveness of sins. And it is only as we drink in His precious peace into our own souls, and enjoy in peaceful tranquility the certainty of sins forgiven; yea, that we are accepted in the favor of the beloved One—that we can expect to be used in bringing others into the possession of that present enjoyment of the inheritance which God our Father has given us in Christ. Oh, to go out from His peaceful presence, as He came forth from the Father.
(Continued and to be continued).

The Coming and Reign of Our Lord Jesus Christ: The Apostasy, Part 1

THE APOSTASY.
It has been a widely-spread and widely-received doctrine among those who profess the name of Christ, that the world is getting better, and that the preaching of the present gospel will be used of God in the conversion of the world, and the setting up of a spiritual kingdom of Christ in the hearts of men. Thousands, satisfied with a surface view of things, have accepted what their teachers have taught them, forgetting the exhortation, “Prove all things” (1 Thess. 5:21); and have never really searched the Word of God for themselves.
Some who read the following statement may be at first surprised, but it is nevertheless the truth. There is not a single scripture, from Genesis to the Revelation, that ever hints at the thought that the world would get better during the present hour of Christ’s absence, but several that distinctly declare the very reverse.
It is perfectly true that the kingdom of Christ will be established throughout the world, but not in a spiritual way in the hearts of all (though doubtless great numbers of Gentiles, as well as all Israel, will then be saved—Isa. 11:9; Heb. 8:10-12), but in actual power and glory, all kings falling down before Him, and all nations serving Him. But this will not be brought about through the preaching of the gospel of the grace of God, but by the display of God’s power in judgment.
The object of the preaching of the gospel now is not to convert the world, but for the taking out a people for His name (Acts 15:14). God is not saving people for earthly blessing, but taking out a heavenly people for heavenly blessing (1 Cor. 15:48); not subduing the nations to the faith of Christ, but uniting a company from among Jews and Gentiles to Christ in glory, as His body and His bride, by the Holy Ghost sent down from heaven (1 Cor. 12; Eph. 1:22, 23).
When the Lord descends and His saints are caught up to meet Him in the air, the present testimony of grace, the gospel of the glory of Christ, as the Apostle Paul expressly terms it, “my gospel,” will cease (Rom. 2:16; 2 Tim. 2:8). Then, during the short period to which we have more than once already referred in these papers, which will elapse between the rapture of the saints and their manifestation in glory with Christ when He comes to reign, a fresh testimony will go out, and the gospel of the kingdom be preached as a witness to all nations, and then shall the end (that is, of the age, not world) come (Matt. 24:14).
Now whilst God in grace at the present time is taking out a people, Christians, the world itself grows worse. Revivals of truth have taken place at various periods since Christ was crucified and cast out from this scene, but the world itself, the mass, wallow in sin. Professing Christendom, cloaking itself over with a form of godliness, but denying the power thereof, is sinning against light, and God’s long-suffering grace.
In the Epistles to Timothy we have a solemn testimony as to the closing condition of Christendom (so-called). In the first epistle, 1 Timothy 4:1, we read, “Now the Spirit speaketh expressly, that in the latter times some shall depart from the faith” (not come to it) and “giving heed to seducing spirits and the doctrines of devils.” In 2 Timothy 3:1, 13 “This know also, that in the last days perilous times shall come.... Evil men and seducers shall wax worse and worse” (not better and better), “deceiving and being deceived.” 2 Timothy 4:3, 4 “For the time will come when they will not endure sound doctrine; but after their own lusts shall they heap to themselves teachers having itching ears; and they shall turn away their ears from the truth, and shall be turned into fables.” From 1 John 4:3 we find that the spirit of antichrist was already in the world at that early period; and Paul writing to the Thessalonians, reminds them that “the mystery of iniquity doth already work” (2 Thess. 2:7).
All these evils are at work around us at this moment; daily they gain ground, and men are led away into the vortex of error and deceit. Rationalism, ritualism, Romanism, infidelity (against which in germ the saint is especially warned in Col. 2) are making rapid strides. The Word of God is set aside by tradition, the teaching of the fathers, and the church, and made of none effect; often questioned, sometimes even denied. Men grow bold in their wickedness. Thousands letting slip the only safeguard, “It is written,” are carried away by the terrible tide, to find themselves eventually shut outside the closed door of grace, to come into the awful judgment of God. The predicted apostasy (2 Thess. 2) fast ripens to a head. Men’s hearts, set upon the advancement of commerce, manufactures, civilization, arts, sciences, military and naval power, and human religion, have no room for the true Christ of God. Masses are so engrossed with the affairs of this life, and deluded by Satan, that they pay no heed to the warning voice of the servants, of God who faithfully proclaim His truth, and warn them to flee from the wrath to come. Men, dear reader, are siding for Christ or antichrist.
(Continued and to be continued)

Correspondence: Matt. 12:31 and 1 John 5:16-17; Luke 7:36-50

Question 28: Please explain Matthew 12:31 and 1 John 5:16, 17. J. K.
Answer: Read Mark 3:22-30. Notice the 30th verse “Because they said He hath an unclean spirit.”
The Lord Jesus was casting demons out of men, the Pharisees and Scribes could not deny that, but their wickedness was so great that they deliberately put it down to the power of Satan, thus they blasphemed or spoke injuriously of the Holy Ghost, this showed their malice and enmity against God. And the Lord said “It shall not be forgiven him” (who committed it) “neither in this world, (that is, the age of law that then was) “neither in the world to come” that is, in the Millenial age. (See J. N. Darby’s Translation).
In this present time we can say to every man: “Whosoever believeth on Him shall receive remission of sins.” Acts 10:43. 1 John 5:16, is God’s discipline on His children, there is no special sin mentioned. We might be able to discern that a believer was chastened and pray for his restoration, or, we might feel that his sin was unto death. It is the death of the body that is spoken of (1 Cor. 11:30). Some had died and some were weak and sickly, here it was because they did not discern the Lord’s body in the Lord’s supper. Acts 5:1-11 is another sample.
Question 29: Was the woman in Luke 7:36 to 50, Mary Magdalene? Is Matthew 26; Mark 14; the same case? Has it any connection with that of John 12? F. F. S.
Answer: The woman in Luke 7:37, is unknown except in her character, her name is hidden; there is no evidence I know of in Scripture to say her name was Mary.
Matt. 26:7; Mark 14:3; John 11:2; 12:3; refer to Mary of Bethany and to the one occasion.

Inspiration of the Scriptures: Some Samples of its Alleged Inaccuracies, Part 1

SOME SAMPLES OF ITS ALLEGED INACCURACIES.
Before closing our remarks on “the Old Testament,” it may be well to look at some samples of its alleged inaccuracies.
We say alleged inaccuracies, because to upright souls who wait on God to be taught by His Spirit, what many of the learned of this world think to be contradictory or incorrect, they find to be full of blessing when rightly understood. No doubt errors in copying with the pen accidentally crept in; verbal errors, too, in translation from one language into another; but the preservation of the Scriptures as they are, is of itself a standing miracle, and distinctly marks the guardian care of God. But supposing in our present version there are some few verbal inaccuracies, they by no means touch the great lines of truth as to creation, redemption, and glory in and through our Lord Jesus Christ, which are so prominently and fully set forth. Let us look at some of the supposed discrepancies.
Living Souls
One of the commonest statements made by the opposers of the truth is, that as all other living and moving creatures are said to have living souls as well as man, there is therefore no more proof of a man having an immortal existence than brutes. Now the answer is plain and unquestionable. Brutes have living souls as a part of their creation, concerning which God said, “Let the waters bring forth,” or, “Let the earth bring forth;” whereas God formed man of the dust of the ground, and “breathed into his nostrils the breath of life; and man became a living soul” (Gen. 1:20, 24, 30; 2:7). Thus man was not only unlike every other creature in that he was created in the image of God, but he “became a living soul” by God’s in-breathing. Hence his immortality; hence his existence after death. If he dies in his sins, after death is judgment. His body only is spoken of as mortal.
We must not, however, confound immortality with eternal life. Eternal life is by our being associated with Christ the Saviour by faith; and thus receiving the gift of eternal life. “God hath given to us eternal life, and this life is in His Son. He that hath the Son hath life, and he that hath not the Son of God hath not life” (1 John 5:11, 12).
First and Second Chapters of Genesis
A very old attack of rationalists as to these chapters giving two accounts of the creation, and contradicting each other, has lately been revived, and largely published. The perfection of the two chapters are wholly unperceived by them; so true is it, that “the things of God knoweth no man but the Spirit of God” (1 Cor. 2:11). The truth is, that in the first section of the book of Genesis, which extends to the end of the third verse of the second chapter, we have God’s work and His rest (Gen. 1:1-2:3). God only (Elohim) is spoken of all through. It is not God giving us an account of everything He created, for angels and other heavenly beings are not there included; but it is God giving us as much as He judged best for our profit and blessing. After the general statement in the first verse, Genesis 1:1, the second verse (Gen. 1:2) shows us the chaotic state the earth was in when God began to form the present heaven and earth for man. Between the first and second verses, a considerable time may have elapsed, and vast changes have taken place, so as to account for geological discoveries; for the earth, not the heaven, was without form, and void. The earth, no doubt, when created, must have been perfect. “As for God, His way is perfect.”
In the first chapter it is God making everything for man’s comfort and blessing day after day, on the sixth day forming man, and on the seventh day resting because all was finished. God is mentioned in this section about thirty times; but in the second chapter we have not simply God (Elohim), but all through it is the Lord God (Jehovah Elohim). Why is this? Because it treats of man’s relationship with God. Now relationship is formed, God reveals Himself as Jehovah God.
Unlike the first chapter, it is not here God giving a consecutive account of what He made in six days, and then rested; but this chapter enters into details more in moral order than in a consecutive style, and very especially occupies us with Jehovah Elohim’s thoughts and ways with “the first man.” He is long afterward spoken of in Scripture as “the first man,” and this chapter is authenticated by being quoted from in that he “was made a living soul” (1 Cor. 15:45). Genesis 2 informs us, that “Jehovah God formed man out of the dust of the ground, and breathed into his nostrils the breath of life; and man became a living soul.” He was to till the ground; He might eat of every tree except “the tree of the knowledge of good and evil,” and if he did eat of it, death would be the result. As he was created to have dominion over every living creature on earth, Jehovah Elohim brought them to him to name them; and whatever he called them, that was the name thereof.
(Continued and to be continued)

Surrender

All, the Saviour has surrendered-
All for us, a guilty race,
His, the love that sought and found us,
Brought us to Himself by grace.
He surrendered all,
He surrendered all;
All for us, our blessed Saviour,
He surrendered all.
All, the Saviour has surrendered,
Humbly bowed His blessed head
On the cross—alone—forsaken,
There He suffered in our stead.
Jesus, Thy complete surrender,
In Thy glory makes us share,
And the Holy Spirit guides us
On, to meet Thee in the air.
All, upon the cross surrendered,
There exhausting judgments flame;
All for us for our salvation,
Glory to Thy blessed Name.

Unscriptural Marriage: Or, the History of Ellen, Part 3

However, another awakening to God was yet reserved for her before leaving Torbay. One day Ellen was hurrying homeward, when the darkened sky alarmed her; and being unprepared for rain, she stepped into a small cottage, inhabited by one of the men employed in guarding the bay from smugglers, to borrow a cloak or some additional defense, and was earnestly persuaded by him not to adventure till the shower passed on.
She was more than three miles from home; a storm set in. The whole bay was covered with the white foam of the breaking billowy; the thunderclouds rapidly advanced, and bursting over the sea, spread such a sheet of lightning as for a moment covered the whole expanse. Ellen’s heart sunk within her when she heard the man repeating as he viewed the storm —
“Jesus, lover of my soul,
Let me to Thy bosom fly;
While the billows round me roll,
While the tempest still is high.
Hide me, O my Saviour, hide,
Till the storm of life is past;
Safe into the haven guide,
Oh, receive my soul at last!”
The rain and wind beat upon the cottage, as if about to sweep it into the sea, and a large portion of the cliff near them fell with such a crash that it shook the very place on which it was built; still, the man was calm, and turning to her perceived her terror; when he spoke so forcibly of Christ as a Refuge from every tempest, and of the safety of the soul in Him, even when the elements themselves shall melt with fervent heat, and the heavens being on fire shall be dissolved, that Ellen regained sufficient composure to question her kind host how he attained this delightful confidence. “Through grace, by walking with God,” was his reply; and it was a volume for her (Eph. 2:8, 9, 10; 1 John 2:3-6).
Being faint, she took some refreshment, which he thoughtfully placed before her, and accepted his offer of accompanying her across the sands as soon as the rain subsided; indeed, the assistance was necessary, for the land rivulets, which ran into the bay, were so swollen that she would have found great difficulty in passing them without aid. On the way they met her alarmed husband, who, by inquiry, had traced her route, and was anxiously seeking her. The next day they quitted the coast, and returned home.
Fully determined now to enter no more into that fatal course of worldliness which had so fearfully destroyed her comfort, after spreading the matter earnestly before God in prayer, she seriously unfolded to her husband the whole state of her soul, and earnestly entreated his assistance in conducting their household, and training their children in the ways of God. She found his heart so deeply wounded by affliction that he did not at first attempt to waive the discourse, or to divert her attention from it, but expressed his willingness to unite with her in serving God. The only difference which remained was in what this devotedness consisted; for he could form no idea of that walking with God in the habits of fellowship and communion on which she was now intent. Religion to him appeared nothing more than strict uprightness of conduct with our fellow-creatures, and regular attention to the outward forms of worship. Ellen founded her descriptions on the Scripture, and portrayed a Christian in his likeness to Christ, both in obedience to the requirements of love to God as well as in duty to man. The following scriptures helped her as to her daily walk: Romans 12, Ephesians 4 and 5. In short, she found the scriptures exhibited a Christian as one who, by being quickened together with. Christ (Col. 2:13), is dead to the world, and the world to him; who is renewed in the spirit of his mind, so as to set his affections on things above (Col. 3:2); who is the property and the willing servant of Christ Jesus his Lord, whose will is his guide, whose work is his employ, whose fellowship is his happiness, and whose heaven is his home.
When she spoke of having the heart fixed on heavenly treasures (Matt. 6:19-24), according to the command of Christ, Mr. R- acknowledged that it was not the case with him, but maintained that it was equally unpracticed by others; and, while he admitted the truth of all the Scriptural delineations of a true Christian, he contended that they were utterly above nature, and quite unattainable—rather set before us as marks to be aimed at, than a description of practicable piety. Yet he allowed that neither he nor any of his associates appeared at all to aim at such religion, and, in consequence, he could not but confess a sad deficiency, while he yet maintained that it was not fatal. But Ellen closed all his excuses by quoting 1 John 5:4: “Whatsoever is born of God overcometh the world: and this is the victory that overcometh the world, even our faith.” “All that is in the world, the lust of the flesh, and the lust of the eyes, and the pride of life, is not of the Father, but is of the world” (1 John 2:16). This was conclusive; for he could not but allow that he had always loved these things, and had been so overcome as constantly to be influenced by them. He acknowledged that he had never possessed the faith treated of by the Apostle.
Such conversations became long, and deeply interesting; and it was during a winter’s evening, when he had, as usual, referred the commands of Scripture to the inability of nature, that Ellen entered on the subject of man being born again by the Spirit of God, and took the opportunity to relate to him her early experience of its truth, the sweet intercourse she had then enjoyed with God, the actings of living faith in raising the soul to heavenly things; and the clear distinction between this condition and her previous cold, formal, unproductive assent to divine truth. She declared her firm persuasion that before this change her state was one of unbelief, and of condemnation. Then faithfully, with the most earnest affection and many tears, she besought him no longer to trifle with his soul. He was greatly moved, and starting at the idea, that because destitute, as he acknowledged, of this living faith, he was an unbeliever in the sight of God, he asked her eagerly if she thought that it was by this grace she had turned her back on the world, and entered into such a spiritual state of mind as he had before thought impossible? She declared it was, and deeply lamented the decay of its practice, assuring him that, much as she had appeared to take interest in worldly substitutes for this piety, she had never known what happiness was since she had ceased closely to walk with God, and concluded by referring him to Hebrews 13.
The result of this conversation appeared important. He seemed to be gradually yielding to conviction; yet still, although more faintly, he would question whether the influence of this living faith might not be some enthusiastical impression on her mind, and whether her former morality was not as true religion as these spiritual feelings; but, unable to resist the force of Scripture, he gave way to her simple distinction of the believer, as an accepted, pardoned, blessed child of God, and the unbeliever as condemned, the enemy of Christ, and yet in his sins. The consequence was, that he seemed to desire such a change in himself, and gladly yielded to her design of forming religious acquaintances. Among these was a young couple of decided piety, who manifested the utmost willingness to cultivate mutual friendship; and the first evening of receiving a visit convinced Mr. R-that the highest enjoyment of society might consist with the purest godliness. The conversation was animated, serious, and interesting. At the hour of eight o’clock the servants were called in to family prayer. The whole service was conducted with simplicity and fervency; and, upon their return home, Mr. R-could not refuse Ellen’s arguments of the propriety, excellency, and duty of the sacred practice by all who profess to be followers of Christ. Indeed, he promised to take the matter into the most serious consideration, whether it were not really his duty as the master of a house.
He now read the Scriptures with care, and appeared so willing to enter into all Ellen’s views, that her wound began to be somewhat healed for the loss of her Jane, whom she everywhere missed. The hope that possibly her husband’s change might make up for the loss of her child began now to gladden her heart, and to introduce a new feeling of animation in the prospects of life. Alas! she little thought how much her own disobedience to her Lord, in forming the connection at the first, was to overthrow every pleasing anticipation.
(Continued and to be continued)

Scripture Study: Matthew 2:1-2

We must not mistake the wise men here for the shepherds in Luke 2. Also in Luke 2 He is the babe in the manger; here He is the young child in the house. In Luke His parents return with Him to Nazareth in Galilee; here they flee into Egypt.
These wise men had seen a star in the sky. They had been led perhaps by the Jewish Scriptures to know the time the Great King was to be born, and rightly judged that it had taken place. Then they start their long journey to see Him.
As Jerusalem is the City of the Great King, they journey thither expecting every one there to know all about Him. They ask, “Where is the King of the Jews that has been born? for we have seen His star in the east, and have come to do Him homage.” Matthew 2:3. Instead of this being good news, it troubled Herod and all Jerusalem with him. He calls the chief priests and scribes together and inquires where the Christ should be born; they tell him at Bethlehem, quoting a part of Micah 5:2, leaving out the part that told of His Godhead glory “Whose goings forth have been from of old, from the days of eternity.” They tell the wise men where He was to be born, but they do not care to go and see. With all their knowledge, their hearts were far from God. How sad to see this in such a privileged people! Do we not find the same tendency in ourselves? This is lukewarmness; we need to guard against it.
Herod, the false king conspires to kill Him. This is impossible, but it shows man’s enmity to God.
These wise men start for Bethlehem, and now the star they had seen at His birth, reappears to their joy and leads them to the house where the young child and His mother were. How glad they were! They rejoiced with great joy. These were Gentiles from afar— a foreshadowing of the time when Jesus shall reign and “the Kings of Sheba and Seba shall offer gifts.” Psalms 72:10. And when they were come into the house, they saw the young child and His mother, and fell down, and worshipped Him— they did not worship His mother. Then they opened their treasures and presented unto Him gifts; gold and frankincense, and myrrh. Thus they expressed their joy, and thus too the Lord provided Joseph with necessary means, to keep them while sojourning in Egypt. God could have provided for them in other ways, but He chose this way.
God warned the wise men not to return to Herod, and warned Joseph to flee into Egypt, and this was to fulfill Scripture, that again it might be true, “Out of Egypt have I called My Son” (Hos. 11:1). Israel’s history, ruined in the first instance, begins again with the Lord in grace.
Herod’s slaughter of the infants is a foreshadowing also of the enemy’s attempt to destroy Christ and His people. (See Rev. 12.) There, too, God preserves them from the enemy’s power.
Matthew 2:19. Again an angel appears to Joseph in a dream in Egypt, commanding him to take the young child and His mother, and go into the land of Israel. Joseph does so, but goes to Galilee and to Nazareth, for he was warned of God in a dream that Herod’s son, who was then king, was just like his father. There we see Jesus as the Nazarene, the rejected One, associated with the poorest of the flock, and hiding from the false king with whom the priests and scribes were associated.
Though He was Son of God and Son of David, He was in the despised place, having come in grace among men.

Truths for Young Christians: Waiting, Part 2

Worldly Christians Cannot Wait or Watch For Christ.
To truly wait for Christ we must be unworldly. If we love this world and the things which are in it, how can we look for the One who is to take us from it forever? But if we have learned the emptiness of the world, to see through its tinsel and to discern the power of its god and prince that is behind, hurrying men on to destruction; we long to leave it, and, thus doubly welcome Christ; first for His own sake, and next, for taking us away from it all to the Father’s house.
I shall never forget some time ago when I had to fetch a dear old lady from a boarding house, where she had been badly treated, seeing her sitting in the little dark underground room in which she had been kept, in a large arm chair, with bonnet, shoes, cloak and gloves all on, earnestly waiting my arrival. Although I could not come before twelve, so anxious was she to leave the place that she had insisted on getting up between four and five o’clock that morning and being fully dressed, and she had been sitting thus in that chair for six or seven hours all ready and waiting to go. I cannot describe her look of intense joy and satisfaction when I entered the room just as she was thinking herself quite forsaken, and her delight when at last she found herself safely in the carriage. What a feeble picture this is of what our attitude ought to be, and what our joy will be at Christ’s coming!
Christ’s Coming Draws Near
Everything bespeaks its nearness. Vain though it must be for us to attempt to fix an hour, which the Lord says no one knows but the Father, still in many ways He has indicated His approaching return. First, in the remarkable spread of the gospel preaching and work among young and old throughout Christendom, thus rapidly gathering in the number of His elect. Secondly, in the fact that the midnight cry, “Behold the Bridegroom cometh, go ye out to meet Him,” that was unheard (save by a very few) eighty years ago, is now sounded everywhere; and thirdly, in that the signs of the last times, spoken of in 2 Timothy, are to be seen on every side of us. Many, however, think that unless it is quite sure that Christ will come in our time, it is no use waiting for Him, because it will be in vain. This is a great mistake. In no sense is it in vain. As regard ourselves, it exercises its purifying influence on our lives, draws us nearer to Christ, and keeps us more separate from the world. With regard to Him it is just as precious to His heart, as if He came. Unlike a king or president who can only see those who are waiting at the moment he passes, He has watched, and knows every “waiter and watcher” whose heart has communed with Him through the long dark night—not one is forgotten. And if you thus patiently wait and watch for Christ, you will be amongst those in glory whom Christ will specially come forth to serve. Oh! may He speak to each of our hearts in power, that many sleepers may henceforth become watchers to His praise and glory.
(Continued and to be continued)

Burning With Pure Oil

In a devoted Christian family, who were walking in the fear of God, the daily reading was one morning in Exodus 27. The twentieth verse was dwelt upon. The parents talked together about the oil that was used in the vessels of the tabernacle, and looked up passages in the New Testament that explained its meaning and how it could be applied. When the reading was over and the older children had left the room, the youngest child, a boy of five years, was detained, as was the usual custom, to be taught some simple verses by his mother, and to pray with her. The parents had not supposed that this child would understand what they had been talking about that morning, or that he would feel the slightest interest in a subject which they thought far beyond his age. However, when he had learned his verse, he kneeled down to pray, and in the midst of his prayer he paused, then he exclaimed earnestly, “O, my God, make me to burn this day with pure oil.”
The morning lesson had not been lost upon him. And his earnest petition to God was not lost. For, throughout the day of his life, he was a devoted Christian. Thus was the earnest cry of that little child heard and answered. God’s Word had found its entrance in his soul, and it did its blessed work.
What an incentive this case should be to young Christian parents to maintain the “family altar,” in this day of luke-warmness, when too many feel that they cannot spare the time for this. God’s rich blessing is upon it; and sorrow will be to those who neglect it.
And are there few, or many, young Christians who are seeking daily to “burn with pure oil”? Such as are, will be lights in the darkness, and will be “polished shafts” in the quiver of the Lord.

From Egypt to Shiloh: Part 3, A Picture of the Church

It would be most blessed to dwell more on this, Shiloh as a picture of what the church was as built by Christ. We shall find the future history of Shiloh, a true picture of the sad history of the church. During the days of Joshua and those that overlived Joshua, Israel served the Lord. His words are very striking: he says, “Now therefore fear the Lord, and serve Him in sincerity and in truth: and put away the gods which your fathers served, on the other side of the flood, and in Egypt; and serve ye the Lord. And if it seem evil unto you to serve the Lord, choose ye this day whom ye will serve... as for me and my house, we will serve the Lord” (Josh. 24:14, 15.) What a word to us now! Every form of idolatry in which we trusted, whether in Egypt or in the wilderness, must be put away. If we are dead with Christ and risen with Him, what need have we for all those things in which we trusted? All are now “beggarly elements.” If we now look at the book of Judges, we shall there see a most striking picture of the history of Christendom. In Judges 2, we have repeated how “The people served the Lord all the days of Joshua, and in the days of the elders that outlived Joshua.” Joshua then died, and all that generation. “And there arose another generation after them, which knew not the Lord.” Then the sad history how they did evil and served other gods. They forsook the Lord and served Baal and Ashtaroth; and for hundreds of years you do not hear a word of Shiloh, the place where the Lord had set His name at first.
Yet God did not forsake His people, but raised up judges; and though some of them were strange men, yet God did care for and deliver His people. There were Deborahs, and Gideons, and Jephthahs, and Samsons; but not one of these ever names Shiloh.
It was exactly so after the death of Paul and all that generation. There arose another generation that knew not the Lord and His ways, as at the beginning. Then did the devil teach the doctrine of development. But the Lord in His tender care, raised up individuals, and through them delivered the church from utter ruin, by the loss of all truth. But for centuries Christ is never again known and owned as the true Shiloh. Nay, a man is blasphemously put in His place as the center and head of the church. And even at the Reformation none of the reformers, so far as we have any record, ever recovered the long lost and only true position of the church as gathered to Him “whose it is.” They did escape from much of paganism, but never knew Christ as the only true center, around which the two or three should be gathered in perfect tranquility. For the most part they retained the worldly nationalism, or Babylonian principle, of confusing the church with the world. It is remarkable, that the very same idolatry has prevailed in Christendom, as in Israel during its centuries of darkness. Observance of days, turning to the east, worship of the queen of heaven, images of saints, as formerly Jupiter and Ashtaroth; monks, nuns, candles, and holy water; all these are real paganism—idolatry, so denounced in the Word of God.
But during those centuries of Israel’s history, did not the true Shiloh exist? Yes, indeed it did, and God surely remained the same. One verse proves this. “And they set them up Micah’s graven image, which he made, all the time that the house of God was in Shiloh” (Judg. 18:31). Has it not been so during the dark history of Christendom? All the time they set up their altars and images which they made; all that time, it was still true that the only true church principle was, Christ in the midst, the true Shiloh, the only One whose right it is to gather His redeemed to Himself on earth, as it shall be in heaven. And no doubt a few of the unknown hidden ones may, in unknown places, have enjoyed the peaceful tranquility of blessed presence, This is sure, the Lord remained the same, though the true place of Shiloh was as little known in Christendom, as in the type in Israel.
There is a most sad history of the one man going to the house of the Lord, as he says, “And there is no man that receiveth me to house” (Judg. 19:18). But so great had been the neglect of Shiloh during these centuries, that few knew the way; indeed, it required the most minute description how to find it! “Then they said, Behold, there is a feast of the Lord in Shiloh yearly, in a place which is on the north side of Bethel, on the east side of the highway that goeth up from Bethel to Shechen, and on the south of Lebonah” (Judg. 21:19). Would it not have been the same for centuries: if a man had inquired where was the true place, the true Shiloh, where saints were gathered to Christ, as in the Acts, could any have told him the place or the way to it? Reader, could you tell it even now?
Yet there was such a place then, and even from which a Benjamite might get a wife; and there is such a place now, where many a preacher may get a sermon, though he neither lives there nor ever gives it a good word. “In those days there was no king in Israel: every man did that which was right in his own eyes.” Last words of Judges (Judg. 21:25). Words also which describe the whole history of Christendom, perhaps the cause of every evil division is just that—the want of the true owning of the Lordship and authority of Christ, in the fear of the Lord. Where this is not, every man is sure to do that which is right in his own eyes—his own will. (Continued and to be continued)

The Coming and Reign of Our Lord Jesus Christ: The Apostasy, Part 2

THE APOSTASY.
But before I go on to speak of the antichrist, who will be one of the chief features of the apostasy, it will be well, in order to simplify these things for the help of those who are not much acquainted with this part of truth, to allude briefly to the prophecy of Daniel, usually termed “the seventy weeks,” as it is during the last of these weeks that the apostasy takes place.
In Daniel 9:24 we find, “Seventy weeks” (that is, weeks of years, 490 years) “are determined upon Thy people” (the Jews), “and upon thy holy city” (Jerusalem), and “to finish the transgression... and to bring in everlasting righteousness.”
This period in the next verses is divided into three-seven weeks, threescore and two, and one. During the seven the street and wall of Jerusalem were to be rebuilt, which took place as described in the book of Nehemiah; then at the close of the sixty-two (that is, sixty-nine altogether) we have Messiah the Prince, Christ. He came into the midst of His people. “And after (the) threescore and two weeks shall Messiah be cut off, but not for Himself, or, as the margin gives it, and more correctly “shall have nothing.” This also came to pass: instead of Messiah being received and His kingdom established, He was cut off, crucified, and had nothing down here; but God raised Him from the dead to His own right hand and commenced a fresh administration. Sixty nine weeks had run out, and one week was lacking to close the chain.
What do we get next? “The people of the prince that shall come, shall destroy the city and the sanctuary?” This is exactly what happened. The Roman armies destroyed Jerusalem and the temple. But mark carefully God says: “The people of the” (or “a,” as it should read) “prince that shall come.” The prince here mentioned is a prince who has not yet appeared. The Romans being the people, it follows the prince must be a Roman prince.
Christ being cut off after the sixty-ninth week, the chain of seventy weeks is broken, and the whole of the wonderful hour of the calling out of the church of God, the administration of the great mystery (Eph. 3:2-11, 5:32), comes in before the seventieth is completed— all the dates of Scripture having reference to the earth or the earthly people Israel, the present nearly nineteen hundred years not being reckoned in the prophecy, but let in, as it were, as a heavenly parenthesis between the dealings of God in the past and in the future with His earthly people. The church being completed, the saints will then be caught up, and the Jews, now scattered, will come into prominence again. This Roman prince, as the last verse of this ninth of Daniel (Dan 9:27) teaches us, will confirm the (“a”) covenant with (“the”) many (Jews) for one week (the missing link completing the chain of seventy). Christ will appear at the close to bring in the blessing of the kingdom and everlasting righteousness.
The object of this covenant will be twofold. The Jews at this period will have gone back in great numbers to their own land by the aid of a certain maritime power (Isa. 18), and will seek protection at the hand of the great head in the West, the Roman prince, in order to carry on their temple-worship, and also against a powerful enemy known in prophecy as the Assyrian, or king of the north (Isa. 10; Dan. 8:23-25; 11:40-45).
But in the midst of the seven years the prince (Roman) will break his covenant with them, and, aided by the antichrist, cause the sacrifice and oblation to cease, and set up an idol in the temple. This is the hour spoken of in Matthew 24:15-22 of which the Lord warned the disciples: “When ye, therefore, shall see the abomination of desolation, spoken of by Daniel the prophet, stand in the holy place, (whoso readeth, let him understand) then let them which be in Judea flee into the mountains.... for then shall be great tribulation, such as was not since the beginning of the world to this time, no, nor ever shall be. And except those days should be shortened, there should no flesh he saved; but for the elect’s sake” (the Jewish remnant) “those days shall be shortened.” The setting up of the abomination or idol in the temple is the signal for instant flight on the part of the God-fearing remnant of Judah at that day. On account of this act God will use the Assyrian as a desolator, as the rod of His wrath, to chastise the apostate Jews, until the judgment determined by Him shall have been poured out. (See also Dan. 9:27; 12:11).

Correspondence: Accomplishment of Salvation; Saints in Heaven; 1 John 1:6-7

Question 30: Did the Lord actually go down to the place of the imprisoned, antediluvian race, to preach to them (or announce) the accomplishment of salvation? J. H. J.
Answer: Read 1 Peter 3:19-22. It was the same Spirit, that quickened Christ from among the dead, that in Noah preached to the antediluvians, and it is because they did not hearken to the preaching by Noah that they are ever since in prison.
The Lord Jesus, when He died on the cross, went to His Father to whom He commended His spirit. He was absent from His body, present with His Father. The ascension of Christ is when body and spirit, united in resurrection, ascended up to the Father, a glorified Man.
Question 31: Will all the saints who die get a place with Christ in heaven?
Answer: All the saints who have died are counted in the first resurrection, and will reign with Christ.
1 Thessalonians 4:16, 17; Romans 8:17; 1 Corinthians 15:23.
Those who will be put to death in the tribulation period will also be raised. Revelation 20:4, “They lived and reigned with Christ a thousand years.” Revelation 14:13 is a promise to encourage them to be faithful unto death. This is the first resurrection, it includes all the saints who have died. “Death is swallowed up in victory” then, after that no more saints will die.
Abraham, Isaac and Jacob are included in 1 Corinthians 15:23. Matthew 8:11 is to teach that Gentiles will be included in their blessing while the Jews called the children of the kingdom will be cast out.
Revelation 7 is a picture of blessing on the earth, there is no resurrection in that chapter.
Question 32: What is walking in darkness? and what is walking in the light? (1 John 1:6, 7). M. J. B.
Answer: Walking in darkness is distance from and ignorance of God. It is the position and character of the unconverted.
Walking in the light, is our position as brought to God, who is light. We have been in His presence where our sins and our self-will have been exposed and dealt with. And since that is our position in the light, we who have been there, have like feelings, we have fellowship with one another as once guilty, but now cleansed. So it adds, “And the blood of Jesus Christ, His Son, cleanseth us from all (or every) sin.”
It is not speaking here of the time of its application to us, but of its efficacy as the only thing that can cleanse away sins, we are cleansed by it.
To walk in the light as God is in it; to have fellowship one with another; and to be cleansed from all sin by the blood; are three parts of Christian position, and not behavior.
None of us walk according to the light fully, but it is our responsibility to do so.

Inspiration of the Scriptures: Some Samples of its Alleged Inaccuracies, Part 2

SOME SAMPLES OF ITS ALLEGED INACCURACIES.
The account also recorded of the formation of Eve, gives us one of the most striking types of Christ and the church found within the whole compass of Scripture. There is also the intimation, that redemption was not brought in merely to repair what man had spoiled; but that, before sin entered, God’s eternal counsel and purpose as to Christ and the church were before His mind, and, as we are afterward told, we were chosen in Christ before the foundation of the world. The earth, however, would be the platform on which man must be tested; man innocent, and man guilty; man without law and man under law; man in Christ’s presence on earth, and now during the presence and power of the Holy Spirit come down; and man will be tested also under Christ’s reign in righteousness, and as Judge of all. And in the eternal state while the earthly people occupy the sin-cleansed earth in unchanging blessedness, the church the bride and body of Christ shall share the inheritance with the Heir of all things, and shine in heavenly glory throughout all ages (Eph. 3:21). Thus in the type Adam could say, “This is now bone of my bones, and flesh of my flesh” “one flesh,” and all the result of the man’s “deep sleep,” so that she shares with him his dominion and glory. Few chapters have been more thoroughly authenticated by our Lord and His apostles than Genesis 2. On one occasion, when an inspired writer referred to Adam and Eve, he says, “Adam was first formed, then Eve” (1 Tim. 2:13). This chapter unfolds this to us, and gives most important information as to man’s accountability, privileges, and blessings, as well as enters into detail as to what took place on the sixth day. It also gives us the divine institution of marriage, and that man and wife should now typically set forth Christ and the church (Eph. 5). Had we not all this instruction, in what darkness and uncertainly as to these things should we be; and if men were hearkening to God, and seeking to learn of Him in reading and pondering His Holy Word, instead of expressing themselves with such temerity and mistaken zeal, the alleged inaccuracies would never have been heard of.
It does not say in the first chapter (Gen. 1) that Adam and Eve were made at the same time; nor does it say in the second chapter (Gen. 2) that man was formed before the animals. Each chapter is perfect after its kind. In the first, we have the consecutive account of God having made all in six days, and then rested. In the second chapter we have the moral order of Jehovah Elohim’s relationship with man; hence the statement that, man having been formed of the dust of the earth, God breathed into him “the breath of life, and man became a living soul;” his responsibility as to the trees, his dominion, and the blessing of an help-meet, have a more fitting place here than in the first chapter.
The oft-repeated and irreverent statement, that in writing the first two chapters of Genesis, Moses copied from two different “theories of creation” which contradict each other, is a mere fable, and carries with it its own refutation. A moment’s reflection is enough to convince any fair mind, that no one could give a true account of creation unless God had revealed it. Besides, as we have seen, the second chapter, in divine perfection, follows the first in giving us further information as to Adam and Eve, which could not properly be introduced into the first, which chiefly sets forth God’s creatorial ways. We do well to remember that Moses was commanded to write, that he often gave his authority with, “Jehovah said unto Moses,” that he abode in the mount with God forty days and forty nights at a time, and is spoken of as “faithful in all His house as a servant.” Moreover we ask, Where in Scripture is there a shred of authority for such charges against the book of Genesis?
No human being, however much spiritually instructed, pretends to understand all Scripture, or to be able to clear up all the difficulties that may be presented. On the contrary, even the most gifted Apostle, when contemplating God in His dispensational actings, says, in a worshipping spirit, “O the depth of the riches both of the wisdom and knowledge of God! how unsearchable are His judgments, and His ways past finding out!” (Rom. 11:33). For any of us to suppose, that we can comprehend all the mysteries of divine truth, would only be the clearest proof of our ignorance. “If any man think that he knoweth anything, he knoweth nothing yet as he ought to know” (1 Cor. 8:2). If apostles were wont to say, “We know in part,” how small must be the measure of any of us now! It is not by argument we advance spiritually; but humble souls God will bless. We have never yet known a child of God, who has quietly waited on God for the teaching of the Spirit as to difficulties in Scripture, but could say that some things which seemed to be inexplicable, have not only been cleared up, but have brought blessing to his soul. Jeremiah felt the need of this in his day. He said, “Hear ye, and give ear; be not proud; for Jehovah hath spoken;” for it was as true then as it is now, that, “the proud He knoweth afar off.” Logic is not faith. Happy are they who bow before God, and say, “Speak Lord, for Thy servant heareth.” Happy are those who are obedient to His word.
(Continued and to be continued)

Unscriptural Marriage: Or, the History of Ellen, Part 4

Ellen observed one evening, as her husband was seeking some passages then in discussion, that he started and was absorbed in profound thought, which continued to hold him agitated and silent for nearly half an hour. At length she ventured to inquire the cause of his emotion. He laid his finger on 2 Corinthians 6:14, and following verses: “Be ye not unequally yoked together with unbelievers; for what fellowship hath righteousness with unrighteousness?”; and then asked her whether she had observed this passage before their marriage. She sighed, and confessed she had.
“Then, my beloved Ellen,” said he, “distress yourself no longer with the apprehension of any difference between your state and mine. I have not, indeed, felt as you have described, nor do I know of any change in my soul to constitute what you call being born again: but the idea is a mere delusion. I know the innocence and integrity of your heart, and am sure that if your feelings had constituted you a believer, while I, with rectitude of conduct, and full persuasion of the truth of Scripture, but destitute of such feelings, was, in the scriptural sense of the word, an unbeliever; you never could have been guilty of such a violation of God’s command. No, my Ellen, we are, and both of us have always been, truly believers; these notions of yours are the mistakes of a virtuous mind, and of a too highly heated imagination. Lay them aside, my dearest companion, and suffer not such gloomy and enthusiastical views to disturb your enjoyment of the pleasures of the present hour. I never before noticed the passage, or I am sure I should have been able to withstand and, I hope, to divert your mind from the superstitious views which have, indeed, lately had too much of your attention—.” What words can describe the emotions of the astonished wife! Reply was impossible; she sat silent, convicted, overwhelmed.
Mr. R- at length changed the subject, and tried to compose her mind. She hastened to be alone, and then confessed her transgression with sighs and tears.
A few evenings afterward she attempted to renew the subject, in hope of yet convincing her husband of his unrenewed state. But he was now dead to every impression, considering her mind as laboring under delusive views of Scripture, and as prejudiced with uncharitable notions of upright persons; nor could she obtain his attention to the subject; but was grieved beyond degree to find all his love for her urge him to divert her mind from every serious subject. The dread of their children being trained up to the same religious habits, led him to counteract every effort of hers to impress the things of God on their youthful souls. The customs and company, so far as he could affect them, became as contrary to true piety as ever, and for five years more she struggled on, sometimes momentarily seduced by the desire of answering to his unabated affection, to what she felt was a compliance inconsistent with the heart’s devotedness to God. Beset with every possible hindrance in maintaining her own steadfastness; perpetually afflicted with the vain and worldly habits of her tender offspring, imparted and cherished by her own beloved partner; sometimes her continual sorrow of heart would threaten her health; and then, oh, the thought of leaving her family to such a soul-ruining system!
But amidst all, she was enabled by divine grace to adhere to the resolution she had solemnly taken: “Let others do as they would; as for her, she would serve the Lord.” How did she mourn that she could not, like Joshua, engage for her household also; but she had read, “Whoso loveth father, or mother, or husband, or wife, more than Me, is not worthy of Me” (Luke 14:26). Especially Matthew 10:32-42, powerfully affected her mind to perseverance.
While reading Psalm 103, she felt the scourge, which by her forbidden connection with an unrenewed heart, she had prepared for herself. She loved God; but she loved one who was a stranger to Him, and He was preparing her for an unexpected trial. What was the agony of her spirit when, during a season of retirement, she was aroused with the tidings that her husband had suddenly burst a blood vessel in a fit of coughing. After the first alarm was passed, and hopes were given that he might do well, she earnestly commended his soul to God in prayer, and determined to renew her efforts to bring him to Christ. The full opportunity was presented; for a while he seemed to gain strength, and at a period when they were alone together, while expressing her delight at his advancing recovery, at the same time she plainly intimated her apprehensions concerning the safety of his soul. He heard her calmly, and then replied, with grateful and equally affectionate language, that “he did not wonder at her concern; he could enter into her feelings, and if he thought as she did, he should very probably entertain the same apprehensions for one who did not profess the change or agitation of mind, which appeared to him to be mere enthusiasm; and which indeed,” said he, with a smile, “has been the only fault I have ever had to find with my beloved Ellen.”
She answered in the words of our Lord, “Except a man be born again, he cannot enter the kingdom of heaven.” He declared that he needed no other dispositions of mind than those which he continually exercised, and that he was quite prepared for death, and had no doubts of his everlasting safety.
“Ah! my dearest husband,” said Ellen, “that self-complacency is the very thing that concerns me on your account; it is so different from the views which a David, a Paul, a Peter, and all the inspired writers took of themselves, and it is so opposed to the judgment of God’s Word concerning those whose hearts and affections are entirely engaged by the occupations and amusements of this world. ‘Love not the world,’ saith 1 John 2:3, ‘neither the things that are in the world. If any man love the world, the love of the Father is not him.’ So also Paul, 1 Timothy 5:6, ‘She that liveth in pleasure is dead while she liveth.’ And our blessed Lord has clearly shown the true condition of our state before God in declaring, ‘Where your treasure is, there will your heart be also’ (Matt. 6:21). You cannot but feel that your heart has been engaged by earth; then, what ground have you in this condition to hope for treasure in heaven?”
“Indeed, my dear Ellen,” he replied, “there have been many deficiencies in this respect; but my life has been upright, and I believe that Jesus Christ will do the rest for me. God is too merciful to condemn such an one as I have been.” “But on what declaration of Scripture do you rest this hope?” inquired she. He felt the question embarrassing, and in vain endeavored to recall some passage to warrant the vague expectation so contrary to the explicit testimonies adduced by his wife, and then cut short the subject by declaring that he should be miserable if he adopted her views, and he earnestly hoped she would never infuse them into the minds of the children. She sighed, and reluctantly acceded to his wish of changing the topic.
(Continued and to be continued)

Scripture Study: Matthew 3

Matthew 3:1-6. John the Baptist is the forerunner of the King, and comes preaching in the wilderness of Judea. He is a prophet; his dress, his food, his place outside, all tell of separation, for Israel is in a sinful condition. He came in righteousness; (Matt. 21:32), his message is a call to repentance for the kingdom of heaven is at hand. The King is coming; he tells of a change of dispensation. Israel had lost all their blessings by their disobedience; if they were to be blessed, it must be on the ground of grace, they must repent.
Matthew 3:5-6. The people flock out to him and were baptized of him in Jordan confessing their sins. Those who thus were baptized, were the excellent of the earth (Psa. 16:2, 3). Luke tells us (Luke 7:29) who accepted John’s baptism, (and Luke 7:30), the Pharisees and lawyers rejected the counsel of God, not being baptized of John.
Matthew 3:7-12. Seeing the Pharisees and Sadducees come to his baptism, he calls them the “offspring of vipers” and demands that they bring forth “fruits meet for repentance.” It is no use saying they were the children of Abraham; they must have to do with God about their sins. The judgment of God will fall on every unrepentant sinner, no matter what religion he professes. The ax is laid at the root of the tree. Man’s trial to produce fruit is nearly over; unless there is good fruit, the tree must be cut down. The King, the Judge, the Baptizer with the Holy Ghost will test every one; the good, that is, those who confess their sins and thus own their badness, will be baptized with the Holy Ghost; and the others, in their sins, will be baptized with the fire of God’s judgment, the wheat will be gathered into His garner; the chaff will be burned with unquenchable fire.
Matthew 3:13-17. Then Jesus comes from Galilee to Jordan, to be baptized of John. John knows Him and forbad Him saying, “I have need to be baptized of Thee, and comest Thou to me?” Here is the great King, Emmanuel, the Son of God, the Son of Man, the dependent man of Psalm 16. What grace! (2 Cor. 8:9).
In reply to John, He says, “Suffer it to be so now; for thus it becometh us to fulfill all righteousness.” Who could fulfill righteousness but the Lord Jesus? All the rest of us are sinners against God. Nor is this Christian baptism. Christians are not fulfilling righteousness when they are baptized, for they are baptized unto the death of Christ, thus owning they have no goodness. We do not keep ordinances to obtain righteousness, for Christ is our righteousness (1 Cor. 1:30). John’s baptism has nothing to do with Christianity; it belongs entirely to the Jewish dispensation.
The Lord Jesus did fulfill righteousness here on earth. Here He associates John as forerunner with Himself, and associates Himself with the remnant of Israel, who confessed their sins. The good trees were those who confessed they were bad, they were the true saints; it was there they must begin, casting themselves on the mercy of God, deserving only judgment.
Jesus takes His place with them. He could have no sins to confess except those of the nation. It was all grace on His part, as on the cross.
Our sins, our guilt, in love divine,
Confessed and borne by Thee,
The gall, the curse, the wrath were Thine,
To set Thy ransomed free.
How comforting to poor failing saints it is to see that when one confesses his sins, the Lord is with him in it.
When John heard His purpose, then he baptized Him, and Jesus went up straightway out of the water, And, lo, the heavens were opened unto Him.
And he saw the Spirit of God descending like a dove and lighting upon Him. And lo! a voice from heaven, saying, “This is My Beloved Son, in whom I am well pleased.” Here we get the Godhead fully revealed. God will not allow any mistake here; this is the sinless Man; the Father’s delight is in Him. “Him hath God the Father sealed” (John 6:27). In this Man the Holy Spirit could take His abode. Now that redemption is accomplished, the Holy Spirit can dwell in believers, witnessing that the work of Christ has cleansed them from all sin. On the saints before redemption, the Holy Spirit came, and John was full of the Holy Ghost from his mother’s womb, but He did not dwell, He was only a visitor. Jesus is the perfect One, the Father’s delight. In Him dwelleth all the fullness of the Godhead bodily.
Compare Mark 1:11, “Thou Art” — the Servant owned as Son. Luke 3:22, “Thou Art” — the answer to the dependent Man’s prayer; here in Matthew it is, “This is” — God thus defends Him before men from all thought of being anything less than His beloved Son, in whom He had found His delight. What a comfort to those earnest, repentant ones to find the Lord in grace among them as one of them in their true position before God: to comfort their hearts by His kindness, to sympathize with their sorrows and difficulties; pointing out the way, and leading them along as the true Shepherd of the sheep. He could not go with men in wickedness, but He could and did go with the repentant remnant.
Well the blessed Saviour knew it would lead Him on to death, the only way by which Israel or man could be saved. Such was His great love, nothing could turn Him aside from doing the will of the Father, and from making atonement for sin.
“When blood from a victim must flow,
This Shepherd by pity, was led
To stand between us and the foe,
And willingly died in our stead.”

Truths for Young Christians: Walking, Part 1

We have spoken of the Christian’s attitude in waiting for Christ’s return in the last paper, in which we also saw that two other attitudes are also closely connected with this event, namely, the Christian’s walk and work. We will now briefly consider the former of these two. In the first place, let us clearly understand that “walking” is not “working,” properly so called. The distinction, indeed, seems so plain as to be hardly necessary at all, yet there is a great deal of confusion on this very point. Many seem to think that if they are walking steadily and correctly, and are manifesting Christ more or less in their daily life, they are doing all that can be required of them; and yet it may be that with all this, beautiful as it is in its place, they may be ignoring and leaving undone a large amount of Christian work that is ready for them. We will point out one or two Scripture expressions on this subject.
Walking and Working
“To me to live is Christ” is a very comprehensive one, and includes both the walk and work, indeed, all that Christ did. Would that we knew more of its meaning.
Take, however, the exhortations to a godly walk in the Ephesians—to walk worthy of our vocation, to walk in love, and walk circumspectly, also those of Peter on the same subject, and compare them with “Always abounding in the work of the Lord” (1 Cor. 15:58). “If any man’s work abide” (1 Cor. 3). “To every man his work” (Mark 13:34), and it will be at once seen that walk is not the same as work, though in some cases the word is so used as to include it; “Walk worthily of the Lord unto all well-pleasing, bearing fruit in every good work, and growing by the true (or full) knowledge of God” (Col. 1:10. JND).
We, however, are so one-sided in our actions and views that, far from maintaining the even balance of Scripture, we either are very active in works, often seeking in that activity to cover up the want of a really godly and Christ-like walk, or else we become so occupied with the passive side of the new life as to have but little Christian activity left. Some, indeed, press work, work, work, till it would seem as if Christianity were all work. Others say only walk, walk, walk, as if the Christian had no real work to do. What Christ wants is both.
Leaving the working, however, just for the present, let us briefly consider the walking: Now our walk is characterized by two great principles, for as we “ought to walk even as Christ walked,” and as He was light and love, even as God is (1 John 1:5; 4:8-16) these two principles govern our path.
(Continued and to be continued)

I Was on the Wrong Track

The person from whose lips I heard the above words, was a worn-out old man of above seventy years.
Disease was making rapid inroads on his frame, each day found him growing weaker and weaker, so that evidently he had not long to live. He had been a quiet, inoffensive man, whose character had been remarkably good for sobriety, steadiness, and industry; but I soon discovered that however much he had been noted for these qualities in his life time, they could not satisfy his conscience, or afford him true peace when he came to die.
I found him in bed, in a sick and helpless condition, and the conversation that then took place was somewhat to the following effect: “Well, S—,” I said, “I see that you are exceedingly ill and worn out, but what about your soul? Are you certain that all is right for the future? Have you peace with God?”
He replied in two words — “I’m busy.”
“Busy,” I repeated, “at what, or about what? You are praying, I suppose, to get your sins taken away, and to have your soul saved?”
He assented.
“Ah! S—,” I said, “you are making a sad mistake—one which hundreds have fallen into. You are putting prayer in the place of Christ. You are looking to, and leaning on, your prayers, instead of the blood of the Lord Jesus. Now, this is not God’s way, and He ought to know a great deal better than you or anyone else. His plan for saving lost and ruined sinners is very different from the plan you are adopting. It is by Christ that a sinner is saved, and not by prayers or by works. The Lord Jesus Himself finished salvation on the cross. By His death and resurrection, He has accomplished all that was needed to save the very vilest of the vile. God loved sinners, but He could not endure their sins. Indeed, He so loved them, as to send His only begotten Son, who, by the shedding of His own blood, perfectly put sin away, so that now the God of all grace shows to you a full salvation for your soul in the blood of Jesus, and proclaims free mercy to you, not through your earnest prayers, but in the name of Jesus. If you prayed with the greatest earnestness for a lifetime, you could not wash away your sins, nor have settled peace. Oh, no! peace has been made by the blood, and peace, perfect peace of conscience comes to us through faith in the blood. You do not gain this by your prayers or tears, your strivings or sincere efforts. Christ has done the whole work. He did it alone, but He did it for us; and now He is exalted at God’s right hand as a Prince and a Saviour. His precious blood cleanses from all sin, and furnishes the sinner with a sure resting-place, so sure that, although the earth should crumble to pieces, and the heavens above dissolve, all is well with the soul that trusts in the blood of the Lamb. Remember, God does not say, that you shall be forgiven through your prayers, but He shows you a full and eternal forgiveness for all who believe in Jesus. Now, S-, that is God’s way. Do you see it?”
The old man raised his head slightly from his pillow and faintly exclaimed, “Oh, I see it now: I was on the wrong track! I was in error.”
“And do you,” I asked, “understand how that peace has been made by the blood, and can you get rest there?”
“Yes,” he replied again, “and I was on the wrong track.” A few days afterward, I called to see him again. His strength was almost gone, his voice greatly affected, and his outward man completely prostrated by illness; but his mind was now quite composed. His own expression was a very simple but intelligent one: “I have rest now in the blood of the Lamb.” On being asked if the thought of the sins of a lifetime did not disturb his soul, he answered, “I have no fear now, I can trust the Lord Jesus.” It was remarked that the blood was the true foundation of the believer’s peace and never could fail. He assented to this, and added, “O! I was in error; but now I have seen my error. I, many a time, thought that I had something to do, and that it was hard to be certain, but now I see that Christ has done all!”
His sole dependence and confidence were now placed in the blood of the Lamb. There was no doubt, no hesitation, no misgiving. He had got away forever from the darkness of his own reasonings into the light of divine truth, where a risen Christ is the grand object for the soul. I saw him several times afterward; but in a few days he fell asleep in Jesus, bearing unequivocal testimony to the value of the blood of Christ in cleansing him from all sin.
May many souls, through the blessing of God, be eternally profited by this interesting case of conversion.

From Egypt to Shiloh: Part 4, The Revival of Shiloh

We now come to the first three chapters of 1 Samuel.
The Revival of Shiloh
It is striking that that which introduces this is a beautiful glimpse of the bride of Christ, in the type of Ruth. This brings us to the present century of Christendom in figure. At the beginning of the previous century the Holy Ghost brought again before us the bride of the heavenly Boaz, and the glorious truth of eternal redemption, to be completed in resurrection. And now after Shiloh had been almost forgotten, all at once we have more in these three chapters as to Shiloh, than in the whole word or history of Israel before.
1 Samuel 1, 2, 3
It would be well now to read carefully these three chapters, so full of our subject, and compare them with Revelation 3:7-22. In the one case we have the closing scenes of Shiloh, in the other the closing scenes of Christendom. In both we are close on judgment. May the Holy Ghost open our eyes to see the solemn application to the very circumstances of this day.
Here then in 1 Samuel we have two families, both at Shiloh, the place where the Lord set His name at first. In one family there is nothing that God condemns; in the other, there is nothing that He approves. In the closing days of Christendom there is not one thing the Lord condemns in Philadelphia (Rev. 3:7-13). In Laodicea there is not one thing that He approves (Rev. 3:14-19).
The very names of the two families at Shiloh are most significant; and in their meaning, and all else recorded respectively of them, we learn that it is not enough to be, as is said, on true ground, that is, professedly gathered to Christ, whose right it is, the true Shiloh, the true and only place He approves; but also, what is the real state of soul of those who outwardly are so gathered.
There is then the family of Elkanah, and the family of Eli. Both are at Shiloh. Everything said in these chapters is about them at Shiloh. No one can deny or fail to admit there has been a most remarkable revival of this very truth, as to the only true place of worship and service of Christians, gathered to Christ, the true Shiloh, in these last ninety years.
As Elkanah is named first, we will first take his name. Elkanah is, “God has redeemed,” “possession of God,” “whom God created.” In this name all is of God. Redemption is an accomplished thing, God has redeemed us to Himself; we are His, and none shall pluck us out of His hand. We are His possession, we are not our own. “If any man be in Christ, he is a new creation;” and that new creation is wholly of God. This very name Elkanah indicates the blessed truths God has restored in these last days.
And the name “Hannah” is equally characteristic. “Grace and mercy.” Oh, the freeness of His favor, and the depths of His mercy!
In contrast with all this, Eli means “a foster son,” “adopted of the Lord;” and it is very remarkable that in Greek the word Diotrophes means a similar thing! All this points out officialism, in place of the enjoyed relationship of a child born of God. Is there not a difference between God adopting the flesh, and imparting the divine nature as born of God?
The names of the sons of Eli are equally characteristic of that which is not approved of God.
Hophni means “Boxer, pugilist.” In Arabic, “To fill both hands full.”
Phinehas, “Mouth of brass.” This boldness may be for good, as in the case of another Phinehas; but what one sees and deplores in some who have taken a place at Shiloh, that is, professedly gathered to Christ, is just what answers to these names. Instead of seeking to help and feed the whole church of God, wherever found, with the blessed truths of a full and eternal redemption—God’s unceasing love and care for His saints as His own possession—and that every believer is God’s new creation; instead of unfolding the riches of His grace and the depths of His mercy; instead of these things, nothing suits their nature more than to go into a village, or a town, and fight everybody and every sect like a boxing pugilist, with a mouth of brass that knows no shame. Such then are the names of the two families, both at Shiloh. And even in the family of Elkanah, Peninnah the prosperous and faithful, was not so approved as Hannah, the feeble, yet daughter of grace and mercy. What warnings and divine teaching for us.
(Continued and to be continued)

The Coming and Reign of Our Lord Jesus Christ: The Apostasy, Part 3

THE APOSTASY
We have a further allusion to this covenant in Isaiah 28:14-21: “Wherefore hear the word of the Lord, ye scornful men, that rule this people which is in Jerusalem. Because ye have said, We have made a covenant with death, and with hell are we at agreement; when the overflowing scourge shall pass through, it shall not come unto us; for we have made lies our refuge, and under falsehood have we hid ourselves.... And your covenant with death shall be disannulled, and your agreement with hell shall not stand; when the overflowing scourge shall pass through, then ye shall be trodden down by it.”
It is called a covenant with death, and an agreement with hell, because the Roman empire, with the head of which the covenant is made, is, in its last revived phase, energized by Satanic power, the dragon giving him his power, and his seat (or throne), and great authority (Rev. 13:2). But the unholy compact is broken; the temple of God at Jerusalem, rebuilt in unbelief, is defiled by idolatry; and the execution of the wrath of God comes upon the Jews to the uttermost. Jacob’s trouble is there, (the remnant shall be saved out of it, Jer. 30:7) the great tribulation. The overflowing scourge, another term for the Assyrian, passes through the land, and the Jews are trodden down by it. Jerusalem will undergo a terrible siege, and shall be taken, “the houses rifled, and the woman ravished; and half of the city shall go forth into captivity, and the residue of the people shall not be cut off from the city. Then shall the Lord go forth and fight against those nations, as when He fought in the day of battle. And His feet shall stand in that day upon the mount of Olives (Zech. 14:2).
Thus far as to the leading features connected with Daniel’s remarkable prophecy of the seventy weeks. Let us now look at a further description of judgments and events that will take place at this terrible crisis in the world’s history, as detailed in the central chapters of the book of the Revelation.
This wonderful book, the church’s prophecy, so little appreciated or understood by many of the children of God; so sadly neglected, notwithstanding the special blessings attached in Revelation 1:3 and 22:7; discloses to us the great effort of the dragon, Satan, to resist the power and authority of God and the rights of His Son in this scene.
In Revelation 4 and 5, as we have already seen, the saints are viewed in heaven in their glorified state around the throne of God, and then the Lamb breaking the seals of the book of God’s counsels and judgments.
Revelation 6 commences the details of these.
Four seals. Various providential judgments.
The first — A great conqueror, overcoming without bloodshed.
The second — Peace taken from the earth, and fearful carnage.
The third — Great scarcity; famine often follows upon warfare.
The fourth — Four sore judgments upon the fourth part of the Roman earth, similar to those of Ezekiel 14:21, which come upon Jerusalem.
The fifth — vision of souls of martyrs slain for their testimony. They cry for vengeance, which shows their Jewish character, and are told to wait for others, who would also be slain during the last half-week.
The sixth — terrible convulsion of authorities and powers, so that men prematurely think that the great day of the wrath of the Lamb is come.
Compare Matthew 24:6-10, the beginning of sorrows. The first six seals will probably be fulfilled before the last half-week.
(Continued and to be Continued)

Correspondence: Pearls before Swine; Disobedient Healed; John 10:16

Question 33: In what way can the Christian throw his pearls before swine, or give the holy things unto the dogs? J. H. J.
Answer: Matthew 7:6: By arguing and discussing the precious things of God with unconverted men, or pressing truth, precious in itself, on those who are not exercised about it. We need to bear a testimony in this world to those around, but it is to be in the spirit of meekness, and with prayer that we may minister the right word 2 Timothy 2:23-26.
Question 34: Was it disobedience on the part of the healed ones in the gospels in not heeding the Lord’s charge to say nothing about their being healed? Why is it mentioned in one gospel and not in another? J. H. J.
Answer: The Lord had always a reason for telling anyone not to speak about what had been done. Sometimes, it was because He was rejected. Other times, because as a Servant, He was hiding Himself. This is specially in Mark’s Gospel. You should specify what instances you desire light about.
Those who were healed, and told it, did disobey, but acted out of the fullness of their gratitude, and failed to enter into His purpose at the moment.
Question 35: Please explain John 10:16. H. S.
Answer: “Other sheep I have, which are not of this fold: them also I must bring, and they shall hear My voice; and there shall be one flock, and one Shepherd.”
“One flock” is the right translation in this last occurrence of the word.
A fold is an enclosure. A flock is a number of sheep; this is a contrast between Judaism and Christianity. Circumcision, laws and ordinances, kept the Jews hedged in and separate from the nations or Gentiles.
Jesus came into this fold to deliver His sheep out of it; this He did through His death and resurrection. Christ went before them, and through His death the sheep are brought out of the sheepfold (John 10:3, 4).
John 10:9 is salvation for “Any man” and liberty and pasture. This is what the believers have in Christ (Eph. 2:8,13-18). They have life more abundantly (John 10:10). There is now no fold. Christians are the flock of God (Acts 20:28; 1 Peter 5:2). They are not under law nor ordinances (Rom. 6:14; Col. 2:20).
They have a good and great Shepherd to gather round and to follow (John 10:27-30). He watches over them, He gives them eternal life, they shall never perish, none can pluck them out of His and the Father’s hand.
“Truly our fellowship is with the Father, and with His Son Jesus Christ” 1 John 1:3.
All in the fold being under the law are under its curse (Gal. 3:10).

Inspiration of the Scriptures: Some Samples of its Alleged Inaccuracies, Part 3

Some Samples of its Alleged Inaccuracies.
THE DELUGE
Nor has the inspired account of the Deluge escaped the rude hand of mistaken men. Very lately there has emanated from the press, by a professed protestant teacher, charges of “historical inaccuracies in the Bible,” and “contradictory statements,” which, he says, “cannot be true.” “As an example,” says he, “compare Genesis 6:20, with Genesis 7:3. ‘Of fowls after their kind, and of cattle after their kind.... two of every sort shall come unto thee to keep them alive.’ ‘Of fowls also of the air by sevens, the male and the female, to keep seed alive upon the face of all the earth.’” The charge of inaccuracy and contradiction is, that the direction in one chapter is that Noah is to take two of every kind, and in another chapter seven. Let us see how far it be an inaccuracy, or whether it be not a ministry of Christ, and an example of the divine perfection of the Word.
If the reader turns to a paragraph Bible, he will find these statements in two separate paragraphs, the first extending from Genesis 6:13 to the end of the chapter, and in it God is commanding, and God is obeyed by Noah. Observe, it is God here. The other expression referred to begins with Genesis 7, and ends with the fifth verse, and here it is Jehovah commanding, and it concludes with, “Thus Noah did, according to all that Jehovah commanded him.” The first allusion to “two of every sort shall come unto thee (Noah) to keep them alive,” is God’s (Elohim’s) care of His creatures to preserve every kind alive in the earth. But when we read of His taking “of every clean beast by sevens, his male and his female,” it is God as Jehovah who speaks. And why? Because He is now arranging as in relationship with man for sacrifices— types of Christ. The paragraph, therefore, begins with, “Jehovah said unto Noah, Come, thou and all thy house, into the ark; for thee have I seen righteous before Me in this generation” (Gen. 7:1). How striking this is! It is God as Jehovah owning relationship on the ground of redemption with Noah, the man of faith, as we know he was. (See Heb. 11:7). We are therefore told, in the next place, that he was to take clean beasts, by sevens, and also the fowl of the heaven by sevens, and it is then added, “to keep seed alive upon the face of all the earth,” for the judgment of the deluge was at hand. Now, the reason of this addition to the general command of two of every sort is very manifest to souls who have to do with Christ, as taught and led by His Holy Spirit. It was Jehovah’s mind that the accomplished work of Jesus, on which all our blessings are founded, should be frequently before Him in figure by the offering up of sacrifices. For this, “clean beasts,” and “fowls of the heaven” were indispensable, for surely nothing unclean could typify the Holy Saviour. Had there been only “two of every sort,” the offering of some in sacrifice would have put an end to those particular kinds of created beings. So the “sevens” left ample room both for sacrifices, and “to keep seed alive on the face of all the earth.” Thus Jehovah, who counted His people “righteous” on the principle of faith in a coming Redeemer, shadowed forth Christ in the “clean beasts,” and “fowls” for sacrifices, as Adam and Noah in their measure also were figures of Him, as “the last Adam,” in having dominion over the created things around them. We find that, no sooner did Noah emerge from the ark, and set foot on the purged earth, than he “builded an altar unto Jehovah, and took of every clean beast, and of every clean fowl, and offered burnt-offerings on the altar,” which most blessedly typified Christ’s sacrifice of Himself. “And Jehovah smelled a sweet savor; and Jehovah said in His heart, I will not again curse the ground any more for man’s sake.” Now, where is the “inaccuracy”? Is it not clear that the “sevens” were actually needed for the sacrifice of “burnt-offerings;” and, if not provided for, would have at once exterminated some, at least, of the “two of every sort”? Where is the “contradiction”? If two applied to all unclean animals, and seven to clean ones, because of the requirements of sacrifices, Where is the difficulty? The child of God, to whom the Scriptures are “profitable,” and for whose “comfort” they are written finds a real delight in the contemplation of such passages of Holy Scripture, as opened up to him by the Holy Spirit; while the philosopher, and all other of the wise and prudent of this world, see nothing to interest, and try to see much to find fault with. Now we trust it is clear to our readers, why it is said of Noah, that “Jehovah shut him in,” while in the same verse it is said, they “went in male and female of all flesh, as God commanded him” (Gen. 7:16). The Scriptures abound with such marks of their divinity. How true are our Saviour’s words to the Father, “Thou hast hid these things from the wise and prudent, and hast revealed them unto babes. Even so, Father, for so it seemed good in Thy sight” (Matt. 11:25-26).
(Continued and to be Continued)

Unscriptural Marriage: Or,the History of Ellen, Part 5

He withheld his heart from God, yet maintained that God loved him. He lived in pleasure, and although the Word of eternal truth pronounced him dead, he flattered his soul with the thought of possessing eternal life. In a few days more another blood vessel broke, and after lingering three weeks, when unable to speak, he gave signs of wanting something. After many things had been brought to him in vain, the Bible was opened; he signified that to be his object, but every precious passage seemed to make him more and more uneasy; he appeared to be desiring something to buoy his hopes which he could not find, looked inexpressible disappointment, sunk into insensibility, and so died. Who can tell the sufferings of Ellen’s mind during this awful interval? Day by day she wrestled, yea, she agonized in prayer for his soul, but the cloud grew darker and darker. She saw him approach eternity without one scriptural hope, and it seemed as if in this matter she must feel the consequences of the forbidden alliance. And oh, when all was over, and her distracted spirit inquired after his— when still the affections, which had indeed been most fondly fixed, followed their beloved object— the bitterness was worse than that of Rachel mourning for her children, who would not be comforted, because they were not! She wished to be comforted, but could not. Where could she find a ray of hope? He was gone, and she knew he was gone forever; they were parted, and she knew they were parted forever.
“Would to God I had died for thee!” was her only passionate cry, save that now and then the exclamation burst forth, “Oh, his soul, his soul!” Then, again, the consciousness of this agony of spirit being the consequence of having been unequally yoked together, and not in the Lord, extorted the piercing words from her quivering lips, “Oh that I had never loved, or never lost!” It was, indeed, an anguish of soul that no words can describe, nor any mind conceive, that has not trodden the same path of disobedience at the beginning, and of similar suffering afterward. We forbear tracing the horrors of this period—they did not soon nor easily subside; but at length her heart bowed to the will of God with more quiet endurance of its woes in a case which admitted of no consolation. The wants and cares of her family also tended to arouse, and, in some measure, to divert her from thoughts that were preying on her health, and even almost on her reason. The Lord graciously mingled spiritual comforts with these otherwise overwhelming sorrows, or she might possibly have sunk into the pitiable condition of a melancholy lunatic. Sometimes in communion with her God she felt the smiles of His countenance, and for a while forgot her misery.
The engaging conversation with her children, after some months, drew Ellen’s mind to other subjects; but even in these hours of soothing maternal employments, she was often unconsciously pierced to the heart by the questions and observations of her dear little companions. One day, while discoursing with them on the Scripture, she told her little Henry to seek of God to make him a Christian indeed, he asked her what it was to be one. While endeavoring to explain from Scripture the Christian character in its upright and beneficent conduct to its fellow-creatures, and in its holy walk with God, the young disciple assured her he would be one; but pausing, as if comparing some past recollections with what he heard, he innocently inquired, “But, mamma, was dear papa a Christian indeed?” Her heart was ready to break! She could only cover her face and weep. They gazed on her, and in the simplicity of love wept with her. There was seldom on these occasions a question asked why; for tears had been lately too frequent to be strange.
After a lapse of some months had produced an alleviation of the first burst of grief, her husband’s brother, who was left executor of his will, and appointed guardian to the children, proposed to enter on the arrangement of various matters, and asked her plan in regard to the education of the children. Ellen informed him that she intended to instruct them all herself if spared, at least till Henry’s age would require other tutors; but to her dismay he told her, in the most delicate manner, that his duty would not permit him to accede to her desires on this point; that for one twelve months he wished all the children to remain with her, as their company might soothe her affliction, but after that period, it was his imperative duty to place both Anne and Henry at schools which he named (places which Ellen knew to be fashionable seminaries, where anything but religion would be inculcated), assuring her that, although he earnestly desired that they might never imbibe her views of piety, yet he would not interfere in such a matter but for his departed brother’s express wishes, which he showed to her, written to him during the period of his first attack of illness, and that under such circumstances he thought the directions of the dead were binding on them both. Ellen did not think so. She pleaded that the directions of God superseded every other, and were binding on everybody; but she could not prevail, nor could she help herself. Mr. R-. had the power in his hands, and nothing could dissuade him from using it, as enjoined by his brother; at the same time, he politely begged that she would invariably avail herself of any assistance which it might ever be in the power of his friendship to afford her; adding his conviction, that she had fully deserved the utmost kindness of all his brother’s family; and that even with all the objection he saw to the enthusiasm of her religion, she had made one of the best wives he had ever known. With these acknowledgments he took his leave.
Ellen shuddered at the long prospect of continued evil which she saw must yet arise from her unhappy forgetfulness of God in her unscriptural marriage. To be separated from her beloved children was no light trouble to her affectionate breast; but the effect which such unchristian training must have on their souls filled her with serious alarm. Anne was now of an age wherein the character often receives impressions that last through life, and she perceived it already to need the utmost maternal pious care, to counteract a disposition naturally giddy, and much influenced by the worldliness and frivolity that had been constantly before her observation. “Alas!” said she, “And must you also, my precious children, partake the consequences of your mother’s transgression? Must it expose you also to the corruption which is in the world, of enmity to Christ? And while I would bring you in my arms to Him for His blessing, must I see you trained by His foes—His despisers—to reject and forget Him like themselves?” And now for the first time she was led to see the mercy which had sheltered her blessed little Jane in the bosom of the Saviour from the evil to come.
We forbear to trace her through complicated difficulties, which were constantly unfolding some fresh hindrance to every pious desire; What could she otherwise expect from an alliance with that world which God declares to be lying in wickedness? We close this warning to those who may be under the like temptation, with a letter written by Ellen to a young friend professing piety, who was receiving the addresses of a suitor, who, though moral, was yet destitute of the scriptural evidences of being a child of God.
“My Beloved L—, “I cannot see you advance to the brink of a whirlpool, strewed with wrecks of peace and piety, without calling to you, as you love God and your soul, to pause and escape from the ruinous snare while yet it is in your power. If you know not the Lord’s commands on the subject of the union of His people in bonds that nothing but death can dissolve, read 2 Corinthians 6:14, and then will you take the members of Christ, and make them those of a stranger to His grace? Oh, my friend, there can be no happiness in disobedience to God. Is it a light matter to prefer a heart wherein He does not dwell, to Himself? You fondly hope to be the means of converting your proposed companion for life; but is rejecting God’s Word a likely way to be thus blessed? You may say there is much that is promising in his character. Let there be all things else, if there be no evidence of divine grace in the heart, he is not ‘in the Lord,’ and therefore your Lord and Saviour forbids the union. It may now cost you much to retrace your steps, but infinitely more to continue them; and, cost what it will, God clearly requires the sacrifice, and He will support you in it. You probably know not how to break off matters; suffer me then to advise. Simply show your suitor the Word of God, and tell him you dare not disobey (1 Cor. 7:39). When you feel the pain of this step, think of me.”
“But it is not for me to complain. I am a wonder of grace. The Lord hath restored my soul, when its own folly had well-nigh ranked it among those who forget God. Oh, my friend, remember the poor sufferer who warns you! Break the snare, and endure anything but the wretchedness of an unscriptural marriage. Sometimes pray for the affectionate but broken-hearted Ellen R— .”
(Continued)

Scripture Study: Matthew 4

THE TEMPTATIONS
Matthew 4:1-11. In the Lord sealed with the Spirit, we see the One who is Jehovah’s delight, as before in His baptism, we see His delight is with the sons of men (Prov. 8:30,31). He is “the Second Man, out of heaven” (1 Cor. 15:47, New Trans). If so, He must be tested to see if He can overcome the enemy of our souls. The first man was tested in the garden of Eden, where in his circumstances, he had everything to help him. The second Man is led by the Spirit into the wilderness, where he had nothing to help Him, there to be tried on every point wherein man had failed.
When he had fasted forty days and forty nights, he was afterward an hungered; then the devil, the tempter, came to Him and said, “If Thou be the Son of God, command that these stones be made bread.” Was He not hungry? And there was nothing wrong in making bread, and He had the power to do it. But Jesus had taken a servant’s place, a dependent man’s place, and He had no order from His Father. His Father’s will must be best. He has no question about that, such as our poor sinful hearts would raise. He is perfect, no sin in Him. He could not fail, though He felt it. All depended on His victory. He won it by obedience, a man that kept His place in dependence on God.
“He answered and said It is written, Man shall not live by bread alone, but by every word that proceedeth out of the mouth of God” (Deut. 8:3). This is what Israel failed in. Israel’s history begins afresh with the Lord. He would not exercise His own will. He did the Father’s will alone. His physical wants could not give Him a motive for action. His living, as men say, was not His object in this world. What a lesson for us!
The second temptation is on the ground of privilege. Will God be as good as His word? It is this time on the pinnacle of the temple in the Holy City that the devil says to Him: “If Thou be the Son of God, cast Thyself down: for it is written, He shall give His angels charge concerning Thee: and in their hands they shall bear Thee up, lest at any time Thou dash Thy foot against a stone,” as if he said, See if God will keep His word. Jesus, ever perfect, answers “Thou shalt not tempt (or try) the Lord thy God,” It is written in Deuteronomy 6:16. (For the meaning of “tempt” see Ex. 17:7.) His confidence in His Father is perfect. He does not need to try Him. He knows and trusts God implicitly. The pattern for us all.
These two temptations were wiles of the devil. In the next, Satan the adversary is plainly seen. The devil takes Him up into an exceeding high mountain and shows Him all the kingdoms of the world and the glory of them, and saith unto Him, “All these things will I give Thee, if Thou wilt fall down and worship me.” This is plainly against God. Jesus ever true to God the Father, will accept nothing from Satan. He will wait the Father’s time and go through all the needful suffering. He at once answers. “Get thee hence, Satan: for it is written, Thou shalt worship the Lord Thy God, and Him only shalt thou serve” (Deut. 6:13). Satan is foiled and driven from Him.
Here is an important lesson for us all: Satan promises a man that he will get on in the world, and get rich faster, if he will only take His way. How many young people, both men and women fall into this trap, because they did not put God first in their hearts. “Ye cannot serve God and mammon.”
The Lord quoted the Scriptures, all from one book; the book given to Israel, with instructions to dwell in their land; one of the five books of Moses that higher critics (so-called) have set aside. The Son of God confirmed them as the word of God, out of God’s mouth, but these wolves in sheep’s clothing set aside the Lord, also to their eternal destruction.
What a lesson is here on the authority, truth, sufficiency, and suitableness of Scripture (Psa. 17:4; 2 Tim. 3:16-17).
Perfect Master, make us more like Thee.
It was here then that Satan was defeated; the second man is victorious; the strong man is bound; Jesus the stronger than he, can now spoil the strong man’s goods (Matt. 12:29).
Now the trial is over; the devil leaveth Him and angels came and ministered unto Him. A picture of the Father’s care over us also (Heb. 1:14).
Matthew 4:12-17. Having heard of John being in prison, Jesus leaves Judea and goes into Galilee. It was on this journey He passed through Samaria (John 4). The sphere of His ministry is Galilee, fulfilling Isaiah 9:1, 2. If the Romans held them in captivity, yet their King that could deliver them was there; He was the great and true light of the land.
But His people are in such a condition, blessing can only come to them on the ground of repentance. He announces this, and that the Kingdom of heaven is at hand.
Matthew 4:18-22. Simon Peter, and Andrew his brother, at His call, straightway leave their nets and follow Him to become fishers of men. James and John also leave the ship, and their father, and follow Him; companions of Jesus during His ministry, and called to share His rejection and His glory.
Matthew 4:23 is a concise account of His ministry.
Matthew 4:24-25 tell us the effect of this ministry. It was the power of God manifested in goodness on the earth, telling out the gracious and longsuffering heart of God.

Truths for Young Christians: Walking, Part 2

Walking in the Light
Taking light first, we find that all believers walk in the light (1 John 1:6-7), but often they do not walk according to it (Eph. 5:8); that is to say, being brought out of darkness into His marvelous light, we are set in a position where no darkness affords an excuse for stumbling. The twilight is passed, we stand in the full blaze of the gospel day. Hence the exhortation in Ephesians is to walk according to the sphere in which we are set. When a Christian sins, therefore, it is not in darkness, but in and against the light, so that we are without excuse. But light is not merely a question of position; from it flow several important qualities of the Christian walk. Righteousness, holiness, truth, purity, are all fruits of light (Eph. 5:9 JND), and of cardinal value in the Christian life.
Fruits of Light
Righteousness is divine light applied to the affairs of daily life, holiness is divine light applied to the life with God, truth is divine light ruling my words, purity, divine light ruling myself (1 John 3:3). We have already considered the question of a righteous walk in 1 Thessalonians 4, and that of a holy or sanctified walk in 1 Thessalonians 5; both of these, let us remember are directly connected with the Lord’s return in Revelation 22. In 1 Thessalonians 3:13, holiness is connected with the Lord’s return, but here, let us remark, it is inward “confirming of your hearts unblameable in holiness before our God.” What a thought, a heart really consecrated to God, where He is first in all things, separated to Him. What a source this is for the holy walk that follows in the next verse (1 Thess. 4:1).
Truth
“The lip of truth shall be established forever.” But God “desires truth in the inward parts,” and “walking in truth” (2 John 4) goes far beyond these words, all-important as they are. With regard to speaking the truth, one of our leading writers has expressed a very beautiful thought to the effect that we should so seek to speak that our words shall express exactly the fact, no more and no less; so that speaking like painting, shall become an art, which shall in the most appropriate word, instead of colors, lay the matter before the hearer. In the present day, especially, when exaggeration is so common, it is as singular as it is refreshing to find a Christian so weighing his words so as to be as accurate as a good picture. Is not our Lord’s reply when asked who He was in John 8, a proof how perfectly true and transparent His words had ever been? “Altogether that which I also say to you” (JND) Surely the habit of consciously being in the light of God’s presence greatly tends to this true speaking. But truth in the inward parts is what God requires, true to God, to myself and to others: to God, in all His Word requires from me to myself, in really and truly being what I am, no more, no less, putting on no false appearances, not deceiving myself; true to others, not deceiving them, avoiding all hypocrisy. This true living is of all-importance to a young believer, as many things may tend to make him unreal. If he has learned quickly much spiritual truth, and yet not been brought very really into God’s presence, he is so apt to desire to appear more than he really is, so prone to seek to be accredited for the truth he knows, rather than for the life he leads. The most dangerous position of all is when he has stepped into some right position before God without real exercise of conscience, and then supposes that the position entitles him at once to look down on others, and imagine himself far on in the school of God. Be severe with yourself, beloved reader; at all costs be truthful, underrate rather than overrate your spiritual state. This alone leads to a true and holier walk. Walking in the truth is different (2 John 1), and is accomplished by walking according to the revealed Word of God. This, it is needless to say, is of all importance. No walk, however sincere can possibly be according to God that is not according to “the truth.”
Purity is a beautiful quality in a Christian’s walk. Occupation with what is defiling can never make us pure, but occupation with Christ does. We see that in 1 John 3, where Christ is the measure of our purity (1 John 3:3), our righteousness (1 John 3:7) and our love (1 John 3:16).
(Continued and to be continued)

Is Conversion a Reality?

About forty years ago, when preaching in the open air was not as common as it is today, George Brealey, a devoted English evangelist, announced a series of open air meetings in a remote part of Devonshire. Many of those to whom he desired to present the gospel were much opposed to him, some even threatened that if he came they would burn him. He replied “They will have to get the wood ready today, as I hope to be there tomorrow.”
At the time appointed he went, and found a large number assembled, young and old. It was truly a motley group, some of them were bent double with age, some were blind, others lame, and all wore the appearance of misery and woe. But the power of the Lord was there, and many, that night, for the first time, went home to weep and cry for mercy.
Among those who were thus awakened was a man who had been greatly opposed to the meetings, and would neither come himself nor allow his wife to come. Open-air preaching was so strange and new to them that the wife told her husband she intended to go; to which he replied “If you go it will be the worse for you when I come home.” His threat however did not deter her. As he spent much time at the ale-house, she expected to be at home again before he returned.
And now behold the hand of the Lord. The meeting had just commenced with singing a hymn, when this very man, carrying a bundle of bean sticks on his shoulder, was attracted to the spot by the sound of singing. As he came in sight of the people, he placed the bundle of sticks against the hedge, and lay down behind it to be out of view. But He who had drawn him there, was about to reveal Himself to his soul, and therefore he could not be hid.
The words spoken by the preacher sounded strangely to him. The speaker told them that the great King had sent a message to every man and woman there, to invite them to spend an eternity with Him in joys that could not be told; that the message was especially sent to those in the highways and hedges.
The man came from his hiding place, and gradually drew nearer and nearer to the preacher, until he stood close to his wife. Both were so intently listening to the speaker’s words, they were not aware of each other’s presence until the meeting was over. Then they went home together to weep over their newly-discovered state as sinners. It was not long before the Lord revealed Himself to them both as the Saviour of sinners, and they were enabled to rejoice together.
And now what a difference was seen in their lives, the work was a real work, and everyone around could see a different man, and a different woman. Their one desire was to lead their children to Jesus, and to use their own words, “We are determined, through grace, to have an empty house when the Lord shall come.” After several years of living testimony to the reality of conversion, the whole family went to America, and after being a bright and shining light for Jesus in that land, he was called to the inheritance incorruptible. The following letter was received after his death from a fellow Christian:
“Beloved Brother: You will be glad to have a few particulars respecting the death of our dear brother who so sweetly fell asleep in Jesus. He was true to the Lord to the last and he fell in harness, a true devoted laborer in the gospel. He called us round his bed just before he departed to be with the Lord, and said, ‘Be whole hearted for Christ, spirit, soul and body, for when you come to the place I am now in, you will see how very little you have done for Him who has done so much for you. Therefore be whole-hearted for Him. I am about to enter into His presence who loved me and gave Himself for me. We shall soon meet yonder.’ And bidding us farewell, he called his unconverted neighbors, and filled his bedroom with them. He said, ‘I have called you to witness how a Christian can die. I have no fears, for Jesus has washed all my sins away in His precious blood; and I am going to meet Him, and I want you to meet me there also. Jesus wants you. He died to save you, and He says, Come to Me, come! come!’ He ceased to speak. He rested from his sufferings and from his labors. He had entered in to see the King in His beauty. There was great weeping by the unconverted; and from that scene, one at least received life in Christ, and we have hopes of others. While upon us it has left the feeling of Elisha, as he returned from witnessing the rapture of Elijah.”

From Egypt to Shiloh: Part 5, Hannah

Let us now look at Hannah, for the Holy Ghost brings her out the most prominent. We see her at Shiloh, provoked by her adversary, because of her barrenness. She lays all before the Lord at Shiloh. There she weeps in the bitterness of her soul. It may be, my reader, you are barren and unfruitful in the things of the Lord. Have you ever wept in bitterness over this? She wept sore; Have we? She asked at Shiloh for a man child, and she asked for this for Shiloh. Eli, the aged priest, knew nothing of all this; he saw, but did not understand; he thought she was drunken. Yes, there may be two parties at Shiloh, and they do not understand each other. Eli sits on a post, and Hannah weeps sore. But the request of the weeper is granted. She had poured out her soul at Shiloh before the Lord, and He had heard, and answered. Jesus says to His feeble, weeping Hannahs, “Verily, verily, I say unto you, Whatsoever ye shall ask the Father in My name, He will give it you” (John 16:23).
He for whom she had asked was born. “She bare a son, and called his name Samuel [asked of God], saying, Because I have asked him of the Lord.”
Men delight in what is great and showy; not so the Lord. He says, “I have set before thee an open door, and no man can shut it.” Is this because thou art become great and strong, and hast done many mighty things? No; “for thou hast a little strength, and hast kept My word, and hast not denied My name.” Is not this what the Lord approves in these very last days? Philadelphia (Rev. 3:8) is the answer.
In our lovely picture of the true Philadelphian, there are four things. Samuel is a little child. Jesus tells us there is no way of entrance but “as a little child” (Luke 18:17).
The second thing is, Samuel is weaned before he is brought to Shiloh. What sorrow in the assembly caused by persons being brought in before they are weaned from the world! There was more weaning forty years ago.
The third thing was, Samuel was dedicated through death, the death of a bullock.
And the fourth mark of this true Philadelphian, was that he was a worshipper at Shiloh; “and he worshipped the Lord there.”
No doubt two parties, the approved and the disapproved, may both be at Shiloh; that is, both take the ground of their meeting, to be gathered to the Lord. How am I to know which is right? Here are four things to guide me: little, weaned, dedicated, and a worshipper. Do these marks answer to the state of our souls, or rather, does our state answer to these marks? Are we really little in our own eyes? If not, we are not the children of Hannah, grace and mercy. Are we weaned from the world? Are we really dedicated by the death of Christ, of which the bullock was a type? Think of being crucified with Him. And lastly, Are we true worshippers in spirit and in truth? Do we delight in God, joy in God? What is the love of God to you? Is it so shed abroad in your heart by the Holy Ghost given unto you, that in return you adore Him in holy peace? Oh, that these marks did more abound wherever souls are gathered to Him, whose right alone it is, the blessed Shiloh, and in that only place of peace and tranquility of soul.
(Continued and to be Continued)

The Coming and Reign of Our Lord Jesus Christ: The Apostasy, Part 4

THE APOSTASY
Revelation 7 is a break in the dealings of God, showing His preservation of a measured number from the twelve tribes of Israel, and also of a great multitude of Gentiles through the great tribulation, for millennial blessing on the earth.
Revelation 8. Opening of the seventh seal; silence in heaven half-an-hour. Seven angels with seven trumpets. Christ is seen in angelic garb. First four trumpets sounded. Fearful judgments, falling chiefly on the prosperity, commerce, and established power in the western part of the Roman empire, which is probably meant by the third part, mentioned several times.
Revelation 9. Fifth and sixth trumpet judgments, or first and second woe.
First. Satanic delusions on the Jews, the sealed being preserved; probably connected with antichrist.
Second. Immense army of horsemen, from the direction of the Euphrates, overcoming and destroying Gentiles, judged on account of their idolatry and other sins. Probably the power of the Assyrian, the rod of God’s wrath (Isa. 10). Third part of men killed (Rev. 9:18).
Revelation 10. Jehovah, in angelic garb, intimating the near approach of the closing of the “mystery of God” (His long-suffering with men; bearing with evil in the world instead of inflicting immediate judgment, allowing the righteous to suffer meanwhile,) and the subjection of all nations to Himself.
Continued and to be continued).
“Jesus” when on earth was slain,
Now lives on high again;
He in righteousness will reign,
When He comes in glory.

Correspondence: Healing; Ps. 23:4; Birth, Conversion, Chastening; Matt. 25:32

Question 36: What are we to expect as to divine healing? M. T. C.
Answer: “As for God His way is perfect” (Psa. 18:30).
God’s power and love and wisdom are perfect. He can make no mistakes.
There is nothing too small or too great to bring to the Lord; there is nothing but what His power can accomplish for us if He sees it is for our good. This ought to work in us full submission to His will, enabling us to trust Him where we do not understand what He is doing with us.
Miracles are called “the powers of the world to come” (Heb. 6:5). When the Lord comes in glory with His saints, then He will use that power to set right His ruined Kingdom. The same power He will use to change our bodies when He comes for His saints (Phil. 3:21).
We often see the Lord’s hand working for the good of His people now, in care and protection and healing, but without any seeming miracle.
If we saw any one working miracles, (Mark 9:38-40) we can leave them to the Lord as His servants; to Him they stand or fall. The Lord can do as He pleases, it is for us to seek the path of obedience. The Scriptures you refer to do not lead us to think that every believer can work miracles; or to think that no believer should be sick. 1 Corinthians 12, refers to spiritual manifestations, but does not say that these gifts were to continue; whereas Ephesians 4:11-13 distinctly tells us what gifts will continue till the body of Christ is completed; no sign gifts are mentioned to continue.
The Corinthians had these sign gifts, but their behavior was such the Apostle had to reprove them; they were “babes” and “carnal,” no marks of spirituality about them, but the opposite. In Ephesians, Colossians, Philippians, and others, higher teaching is given, showing a better state.
2 Corinthians 12:7; tells of something the Lord sent to Paul to keep him humble. It was needed, so it was not removed, but Paul gloried in the power of Christ resting on him through it.
Philippians 2:25-30. Epaphroditus was sick nigh unto death in the Lord’s service, and was not cured miraculously, but the Lord had mercy on him with no obvious miracle.
2 Timothy 4:20. “Trophimus have I left at Miletum sick.” Here again the Apostle did not interfere with the Lord’s discipline over His servant.
1 Timothy 5:23. Timothy is told to “Drink no longer water, but use a little wine for thy stomach’s sake and for thine often illnesses” (JND). He does not direct him, to divine healing, so-called, but leaves him as he was, only giving him advice to use what might alleviate his trouble, showing us that we are free to use what we may think good and to count on the Lord blessing it (1 Tim. 4:4).
Hebrews 2:3-4. The object of the sign gifts was to confirm the Word before unbelievers, “God bearing them witness.” Mark 16:17-18 is in the same line, also Mark 16:20, “the Lord working with them, and confirming the Word with signs following.” Thus we see what was predicted, was fulfilled.
At the close of this period of grace, signs will again appear; (2 Thess, 2:7-10) powers and signs and lying wonders. We see the mystery of iniquity already working. When people turn from the Lord and His truth, they turn to Satan and to fables (John 5:43; 2 Tim. 4:3). Already we see the signs coming in, and Satan working wonders, but, alas! the truth is given up. On the other hand, in all ages, sick believers have looked to the Lord, and when He saw it fit, they have been healed.
The Epistle of James is addressed to the twelve tribes of Israel, and its subject is practical righteousness, and is therefore of general application: our special relation as children of God, members of the body of Christ, and temples of the Holy Ghost must be borne in mind. James 5:14-16 is specially connected with the Lord’s discipline, and forgiveness of sins committed is here given through confession. It would not therefore apply to every case of sickness.
1 John 5:16-17 is discerning why God’s chastening hand is on a brother, whether his sin is unto death, (the death of the body). It is not some special sin, but what a brother may have fallen into by allowing careless, or gross habits—a warning to us all to be diligent to “watch and pray” (2 Peter 1:9). It has nothing to do with such scriptures as Mark 3:29.
1 Corinthians 11:30-32 is the chastening of the Lord on some who did not discern the Lord’s body in the supper. These were believers, but were not walking right and some were weak and sickly while others died. They were “chastened of the Lord,” but not “condemned with the world.”
When the Lord raised, or healed any one, it was done at once, not gradually; except the instances of the blind man in Mark 8:22-25 and in John 9, which are pictures of the education of the soul in the things of God. Gradually the light of Christianity displaced the darkness of Judaism. It impoverished the soul for believers to get taken up with temporal blessings or mercies. We thank God for them all, but our blessings are spiritual and heavenly and eternal. “Blessed with all spiritual blessings in heavenly places in Christ” (Eph. 1:3).
(To be continued)

Inspiration of the Scriptures: Some Samples of its Alleged Inaccuracies, Part 4

SOME SAMPLES OF ITS ALLEGED INACCURACIES
JACOB’S GOING INTO EGYPT.
It has been stated, that because we read in Scripture of those who went down into Egypt being sixty-six in one place, in another seventy, and in a third seventy-five, there must necessarily be contradiction in the statements; but those who make such statements have not read the passages with sufficient care to perceive that they are three different calculations.
1. If we turn to Genesis 46, we find a complete list of those who composed the sixty-six, and accompanied Jacob into Egypt. Such accuracy is manifested in the account, that two who had died were named only to show they were not in the list. “All the souls that came with Jacob into Egypt, which came out of his loins, all the souls were threescore and six.” This is the first list.
2. The next verse tells us, “All the souls of the house of Jacob which came into Egypt were threescore and ten.” This is the second list. How is it so calculated? Because Joseph and his two sons which were born to him in Egypt, were already there. Hence these, with Jacob, as stated, made up the number of seventy souls. Now, where is the contradiction? Nay, more; is it possibly to read this chapter attentively without being struck with the care that is taken to avoid the appearance of any discrepancy? But further. If we look into the beginning of Exodus 1, we again find the list spoken of as seventy, and including not only Jacob’s eleven sons by name, who came out of his loins, and went with him into Egypt, but in strict agreement with Genesis 46:27, it is added, “for Joseph was in Egypt already.” Now, where is there any contradiction?
3. When Stephen, in his famous speech, refers to this, he says, “Then sent Joseph, and called his father Jacob to him, and all his kindred, threescore and fifteen souls” (Acts 7:14). This is the third list, and here, even to upright souls, a difficulty may present itself. Observe, however, in this calculation, that Joseph and his sons may not be included, and if so, it leaves room for nine more of the patriarch’s “kindred”; and kindred is certainly not the same thought as those who came out of his loins.” We do not offer any positive solution of the difficulty, nor is it needful to prove how exactly the list of seventy-five was made up. If we had been told that two of his sons’ wives had died, it would be made clear, but we are not told, and must be silent. It is enough to know that Stephen, in this statement, quoted from the Septuagint, the authority of which was generally allowed. The fact too, that Stephen was full of the Holy Spirit, and speaking before an assembly of masters in Israel, who were well instructed as to every detail of the history of Jacob, these and other considerations leave no opening for question as to the veracity of the martyr’s statement.
(Continued and to be Continued).

To Christians Contemplating Marriage

“Be ye not unequally yoked together with unbelievers” (2 Cor. 6:14-18).
This is instruction for the guidance and encouragement of believers, and is applicable to all their voluntary associations. It should lead a Christian to ask the question, Does God ask me to choose my company? He does, and in doing so teaches us to take into account our relationship with Him. If a Christian thinks of marriage, this scripture instructs him to choose one who is the Lord’s, so that they can have fellowship in His things, and be of one mind to serve Him.
If he is a person seeking to walk with the Lord, his conversation will turn to spiritual things, and he will soon discover whether this is agreeable to the one he is keeping company with. He ought to seek by prayer, discernment from the Lord, so that he might make no mistake. It is easy to let our natural affection or liking for a person blind us in such a matter.
A Christian knowingly marrying an unbeliever, cannot count on the Lord’s blessing on their union. Often Christians disobey the Lord in the fond hope that the person they allow their affections to run out to, will be converted; but it often turns out a lifelong hindrance to their true happiness, and deep sorrow and chastening follow such a step. How often we have heard the sad confession, “I have myself to blame.”
“Can two walk together except they be agreed?” (Amos 3:3), is another important question, and suggests that it is not every Christian that is suitable for a life companion. 1 Corinthians 7:39, says of a widow, but it applies to all Christians who have thoughts of marriage, “She is at liberty to be married to whom she will; only in the Lord.”
“In the Lord” means with the Lord’s approval.
“In Christ” would indicate our standing before God as Christians.
“In the Lord” supposes two that have the Lord’s interests before them, are owning His authority over them, and seeking to have the stamp of His approval on their ways. They own that the Lord has the first claim on each of “His own”; they “are bought with a price,” and desire to glorify God in their bodies (1 Cor. 6:19-20).
It is the Lord who has instituted all our relationships, and told us how to walk in them (Eph. 5:22-6:9; Col. 3:18-4:1; 1 Peter 3:1-9). A child is to be subject to his parents, but the Christian child must own the Lord’s claims to be higher than his parent’s; so with the husband and wife, the Lord’s claims come first. A husband who demands that his wife obey him, when it is against her conscience before God, is assuming an authority over her that God has not given him.
How important it is therefore for Christians to make sure of agreement in divine things before entering into a life-engagement.
Obedience to God and love to Him should control our natural affections.
If the Lord has the chief place in our hearts, it will be so (Luke 14:26-27,33). If we do not give Him the chief place, we will find to our sorrow, sooner or later, what a loss has been ours.
In days like the present, of trial and division in the church of God, if a sister is of a gentle, leaning disposition, and knows the truth of being gathered to the name of the Lord, while her husband does not, and is not willing or able to learn it, she is made unhappy by going into meetings that are not on scriptural ground, in her desire to please or even win her husband; or if she holds to what she knows is the truth, then there is a divided house, and both suffer through not considering this question before they were engaged. If there are children, how are they to be taught and guided aright, when the parents are divided and going different ways?
The Lord would have us consider this beforehand. Let those thinking of marriage seriously wait upon and ask the Lord what He would have them do, and they will put the Lord’s honor first, and seek to obey His word. They will seek His interests first, through grace.
If two fail to agree before marriage as to God’s path for His people, can they walk together, agreed to glorify the Lord as heirs together of the grace of life? (1 Peter 3:7) Can they give up the truth and not suffer spiritual loss? If they put natural affection first, is it not plain that it is not the Lord’s glory, but rather their own likings that they consider?
It is quite true that sometimes those who started right at the beginning, have gone wrong. This may have been through not considering together everything that comes to them, and their hearts have consequently been alienated from each other. It is needful to begin right with each other and the Lord, and then through grace to go on together with Him, “as heirs together of the grace of life” (1 Peter 3:7). Natural affection is not enough to carry us through. We need to have the Lord ever before us.

Scripture Study: Matthew 5:1-16

The ministry and miracles of the Lord had attracted the multitude. He took an elevated place and sat down and taught His disciples in the hearing of the multitude. As the prophet, He tells them the character that suited His kingdom. It is His own character that comes out in it all.
Immanuel (Isa. 7:14; 9:6) was here among men. He knew what was before Him; He was rejected and despised of men because of the lowly place He took in grace. His people share with Him in His rejection.
The believing remnant of the Jews, the disciples in that day, were brought into the church at Pentecost. The believing remnant of the tribulation period will share the kingdom in the millennial age. Believers now are a heavenly people. We are children of the Father; members of the body of Christ; temples of the Holy Spirit. The principles contained in the sermon on the mount apply to us also, but we need to notice the above distinction from those who had an earthly calling.
Matthew 5:3-12 is the character and portion of those who are of the kingdom.
Matthew 5:13-16 is their position in the world.
Matthew 5:17-48 is the connection between the principles of the kingdom and the law.
Let us look at them a little in detail.
Matthew 5:3, “Blessed are the poor in spirit: for theirs is the kingdom of heaven.” This is accepting a despised place; it is not seeking great things for self in a scene that is contrary to God—these possess the kingdom.
Christ’s character is seen in all these as the example for us. What He taught is what He is (John 8:25; see new translation.)
Matthew 5:4. “Blessed are they that mourn: for they shall be comforted.” This is mourning before God and in sympathy with Him over the sinful condition of men. God comforts them with promise of blessing in which they shall share. (See Mark 7:34; 8:12; John 11:33, 35).
Matthew 5:5. “Blessed are the meek: for they shall inherit the earth.” Meekness does not maintain its own rights; it does not retaliate, it rather suffers wrong; but God defends the meek. It is one of the fruits of the Spirit (Matt. 11:29; Psa. 22:26; 25:9; 37:11). “They shall inherit the earth” applies specially to the earthly calling; but we who are heavenly should receive all things from our God and Father in the same way. We should not be conformed to the world’s ways, and should be done with selfishness. He will “supply all your need” (Phil. 4:19).
Matthew 5:6. “Blessed are they which do hunger and thirst after righteousness: for they shall be filled.” Here is earnest desire for good: the inward condition and activity of the mind hungering and thirsting after the things of God. The answer is Psalm 107:9; Ephesians 3:20.
Matthew 5:7. “Blessed are the merciful: for they shall obtain mercy.” This is consideration for others. It is the deep fountain of God’s nature in the believer, for “God is love,” and He is “rich in mercy” (1 John 4:8; Eph. 2:4). In us it is, “bowels of mercies” (Col. 3:12).
Matthew 5:8. “Blessed are the pure in heart: for they shall see God.” We get here the pure motive that keeps God in view. The pure in heart see God’s ways in everything.
Matthew 5:9. “Blessed are the peacemakers: for they shall be called the children of God.” This is indeed the activity of the divine nature; God’s character among men, carrying peace with them, their feet shod with the preparation of the gospel of peace (Eph. 6:15). Thus they would win the title, “sons of God.”
Matthew 5:10. “Blessed are they which are persecuted for righteousness’ sake: for theirs is the kingdom of heaven.” Here we have the treatment from others of those who do the will of God, and walk in practical righteousness before Him. The soul that walks with God cannot do what it knows is wrong. It thus brings persecution upon itself from those affected by its integrity (Psa. 25:19-21). Theirs is the kingdom of heaven.
Matthew 5:11-12. “Blessed are ye, when men shall revile you, and persecute you, and shall say all manner of evil against you falsely, for My sake. Rejoice, and be exceeding glad: for great is your reward in heaven: for so persecuted they the prophets which were before you.” Notice the change to “ye.” It is suffering for Christ; it is precious to Him. He sees it all, and encourages the sufferer with His promise, “great is your reward in heaven.” To do well and suffer for it, is indeed acceptable with God, and brings the soul its sure reward (1 Peter 3:14-17), but to suffer for Christ’s sake is still more precious (1 Peter 4:12-16); “the Spirit of glory and the Spirit of God resteth upon you”; “on your part He is glorified.” Thus we are encouraged to bear testimony and to walk in the ways of the Lord.
Matthew 5:13. “Ye are the salt of the earth.” As the salt, the believer strengthens and upholds all that God has instituted; it is preserving what is good. If the believer goes with the evil, his influence is lost, and his own communion is marred. It is not a question of losing eternal life — that could not be. It is our influence, as walking in the fear of God.
Matthew 5:14-16. “Ye are the light of the world.” “The world lieth in the wicked one” (1 John 5:19), and is in darkness, while the believer is brought out of it (Eph. 5:8). They are to bear testimony, like a candle in a candlestick gives light to all in the room, and like a city set on a hill that cannot be hid. Then the warning: if it is put under a measure, the benefit is lost, the things of the world have covered up the testimony. May we ponder verse 16, so that our words and works may glorify the Father which is in heaven (2 Thess. 2:16-17).
This is the first mention of the Father’s name, “which is in heaven,” as suited to an earthly people. In John 20, when the Lord rose from the dead, He could say, “My God and your God,” “My Father and your Father.”
(To be Continued).

Truths for Young Christians: Walking, Part 3

WALKING
The Threefold Sphere of Love
Love is the second great characteristic of God, and therefore of the Christian’s walk. We are exhorted to walk in love. Love to God, to our fellow-believers, to our fellowmen. Love to God shows itself in obedience. Obedience, to be worth anything, is the offspring of love. Thus alone Christ obeyed, and to His obedience are we set apart (1 Peter 1:2-4). Turning instinctively to God for direction in every event of life, waiting till we get it, and then following it. Such is the path of Psalm 32, and that of the obedient child; a path of security, of happiness, of freedom from care or anxiety though not from watchfulness. “Obedience,” too, “is better than sacrifice,” and it springs from “hearkening, which is better than the fat of rams” (1 Sam. 15:22). It may not bring us much praise or credit, but it always pleases God, and even when we are “slow of understanding,” if the desire is to obey, the Lord will guide. This, then, is the proof of love to God, and a special blessing is reserved for those who thus walk (John 14:23). Love to our brethren is mostly shown in washing one another’s feet, this is the most delicate proof of real love that can be given, and the rarest (John 13). Love can be shown in the cup of cold water, in the offering of a sweet-smelling savor (Phil. 4:18), in caring for bodily or spiritual needs. The heart that is at leisure from itself, to soothe and sympathize, will readily discover the appropriate way of showing love. Love to the world at large is most shown in pointing them to Christ. Caring for the suffering and the poor is an essentially Christian duty, but care for the soul comes first, though it may not always be made the most prominent.
Such, then, is a brief and imperfect sketch of the Christian walk, all perfectly summed up in the three words, “as He walked.” This is the best direction of all “as He walked” in righteousness (Isa. 53:11), goodness (Matt. 19:16), truth (John 7:18), guiltlessness (Matt. 27:4), lowliness (Matt. 11:29), patience (Matt. 27:14), self-denial (Matt. 8:20), humility (Luke 22:27), obedience (John 4:34), compassion (Luke 19:41) benevolence (Matt. 4:23,24), and love (John 13:1).
May the Lord exercise our hearts to a more godly, truthful, and lowly walk in view of the nearness of His return.

Fragment: Evil and Good

To return evil for good is Satanic. Psalm 109:5.
To return evil for evil, and good for good, is human. Matthew 5:46.
To return good for evil is divine. Matthew 5:44-45. Romans 12:20-21.

The Power and Value of the Name of the Lord Jesus

It is truly edifying to trace through the New Testament the varied virtues of the name of the Lord Jesus. We shall refer to just a few passages.
1. There is Salvation in the Name of the Lord Jesus. “Neither is there salvation in any other; for there is none other name under heaven, given among men, whereby we must be saved” (Acts 4:12). The soul that trusts in the name of the Lord Jesus, gets all the saving virtue which belongs to that name.
2. There is Eternal Life in the Name of the Lord Jesus. “These things are written, that ye might believe that Jesus is the Christ, the Son of God; and that believing ye might have life through His name” (John 20:31). The soul that simply trusts in the name of the Lord Jesus, becomes a partaker of His life, and that life can never be forfeited, because it is eternal.
3. There is Remission of Sins through the Name of the Lord Jesus. “To Him give all the prophets witness, that through His Name whosoever believeth in Him shall receive Remission of Sins” (Acts 10:43). The soul that simply trusts in the name of the Lord Jesus, is forgiven, according to the value of that name, in God’s judgment. It matters not what or who he may be that comes to God in the name of the Lord Jesus; he gets all the credit, all the value, all the virtue of that name, and could no more be rejected than the One in whose name he comes. If I go into a bank with a wealthy and trustworthy man’s name on a check, I enter in all the confidence which his wealth and credit can give. It matters not who or what I am; I come in his name. Thus it is with a sinner who comes to God in the name of the Lord Jesus.
4. The Name of the Lord Jesus is the Power of Prayer. “And whatsoever ye shall ask in My name, that will I do, that the Father may be glorified in the Son. If ye shall ask anything in My name, I will do it” (John 14:13-14). The believer, coming to God in the name of the Lord Jesus, could no more be refused than Jesus Himself.
5. The Name of the Lord Jesus gives Power Over Satan and all manner of evil. “And these signs shall follow them that believe: In My Name shall they cast out devils: they shall speak with new tongues; they shall take up serpents; and if they drink any deadly thing it shall not hurt them; they shall lay hands on the sick and they shall recover” (Mark 16:17-18. See also Acts 3:6; 16:18; James 5:14). It may be said that this power is no longer available. I reply, we are merely tracing through the New Testament, the power and value of the name of the Lord Jesus. That name has power in heaven, power on earth, power in hell, power over angels, power over men, power over devils. Let faith use that precious, matchless, powerful, all-prevailing name.
6. God’s Assembly, wherever it is, is gathered in the Name of the Lord Jesus. “For where two or three are gathered in My Name, there am I in the midst of them” (Matt. 18:20). Observe, it does not say, “where two or three are met. Men may meet upon any ground, or for any object they please, but the Holy Ghost can only gather in the name of the Lord Jesus. The church is not a meeting convened by the will of man, but an assembly gathered by the Holy Ghost.
7. The Name of the Lord Jesus is the Church’s Power in Discipline and Excommunication. “In the Name of our Lord Jesus Christ, when ye are gathered together, and my spirit, with the power of our Lord Jesus Christ, to deliver such an one unto Satan for the destruction of the flesh, that the spirit may be saved in the day of the Lord Jesus” (1 Cor. 5:4-5). The full credit of the name of the Lord Jesus is attached to the act of the assembly, when divinely gathered. Heaven gives the sanction of His name to the act.
8. The Name of the Lord Jesus will be the Object of Universal and Everlasting Homage. “Wherefore God hath also highly exalted Him, and given Him a name which is above every name; that at the name of Jesus every knee should bow, of (those) in heaven, and (those) on earth, and (those) under the earth” (Phil. 2:9-10).
May God the Holy Ghost unfold to our souls more of the power and value of the name of the Lord Jesus, so that we may more fully know what we have in Him, and be enabled to use His name in more holy confidence, at all times, under all circumstances, and for all purposes.

The Father's House

High, in the Father’s house above,
My mansion is prepared;
There is the home, the rest I love,
And there my bright reward.
With Him I love, in spotless white,
In glory I shall shine;
His blissful presence my delight,
His love and glory mine.
All taint of sin shall be removed,
All evil done away;
And I shall dwell with God’s Beloved,
Through God’s eternal day.

From Egypt to Shiloh: Part 6, The Song of Shiloh

Before we go on to the further characteristics of the two families at Shiloh, showing so distinctly what is pleasing to God at this very time, and what is not, let us ask ourselves, Are we real worshippers? Can we sing the song of Shiloh? Yes!
The Song of Shiloh
Sing, you say, how can we sing, and the church in such a state, and judgment close at hand? It was exactly the same when Hannah sang that song of Shiloh; she was like a lark. You might say to the lark, How can you sing? Do you not see what a fog there is on the ground? This way I sing, the lark would say, higher and higher, far above the fog, in the heavenly blue above. Thus sang Hannah, higher, and higher still. Is there a higher swell of joy in the whole Scriptures? As the sun fills the sky of the lark, so the Lord Himself was before her soul.
She did not rejoice in Shiloh, as a place, but in the Lord who is the Shiloh. It is the person who makes the place. “My heart rejoiceth in the Lord... because I rejoice in Thy salvation.” In the spirit of Revelation 3, as a Philadelphian, she says, “There is none holy as the Lord: for there is none beside Thee.” How soon we may slip away from this; how soon get occupied with men! Such was the Person of the Holy One to her, that He exceeded all others. What an exclusion, and what a song!
Is Jesus the Shiloh thus before our souls? If so, how could we leave Him? “Lord, to whom shall we go?” Do our actions show that He is enough, and we cannot allow any beside Him? “And by Him actions are weighed.” The mighty men, and the weak, are all made known in His presence. His wonderful ways are known at Shiloh. Read every sentence. How needed to the children of God now at Shiloh! “He bringeth low and lifteth up.” Could anything prove more distinctly the inspiration of the Holy Ghost than this song at Shiloh? Hannah’s faith and song rises to a theme utterly unknown at that time. The church, of which this is a picture, full of typical instruction, was as yet hidden (Eph. 3:9).
Mark the order of 1 Samuel 2:8-10, in Hannah’s song. She sees humanity a dunghill! Out of that dunghill, God lifteth up or raiseth up the beggar, “To set them among princes, and to make them inherit the throne of glory.” Is not that exactly what God is doing now? Oh, have you been taken as a lost beggar out of the dunghill, as Paul? And is God determined that you shall be set amongst princes, and that you shall, as part of the body of Christ, inherit the throne of glory? Yes; He has no lower thought or purpose for us. But does it in some way depend on my holding fast? May I not turn aside, though a true saint, and after all be lost?
Nothing of the kind, says Hannah: “He will keep the feet of his saints.” True, she says, “And the wicked shall be silent in darkness; for by strength shall no man prevail.” Cheer up, my soul! these are weighty words—they are the words of this day of grace, during which God is gathering the church for the throne of glory in the heavens.
Then in 1 Samuel 2:10. The next thing is the time of tribulation: “The adversaries of the Lord shall be broken to pieces, out of heaven shall He thunder upon them.” Then the judgment of the quick: “The Lord shall judge the ends of the earth.” After this, the reign of Christ on earth: “And he shall give strength unto His king, and exalt the horn of His anointed.”
It was simply impossible for any one to have invented this very order, then utterly unknown to man; but now become the well-known order, and purpose of God. First, the taking out the church for the throne of glory, kept for that purpose by the power of God. Secondly, the day of the Lord, ending with the judgment of the living nations. And, thirdly, the setting up Christ as the true Shiloh, Messiah, King on earth. Let the skeptic tell me how it came to pass, that Hannah sang of all this, in type, more than a thousand years before it was revealed? This amazing song of Shiloh is an unanswerable proof, then, of divine inspiration. It is the very theme at this moment of the songs of those gathered to Him, the only Shiloh. Oh that we were more like Hannah, instead of being crushed with the state of things in Christendom, and even at the very place or position that answers now to Shiloh, as a place, the place in His presence, of peaceful tranquility. Yes, instead of looking at the state of things until our hearts sink within us, may we, like Hannah, and like the lark, thus rise above them, and rejoice only in the Lord.
We have noticed that there were two families at the only true place — Shiloh. One is approved, the other disapproved. What was the difference? This may help us to discern the Lord’s mind now, especially where there may be two companies, both declaring they are gathered to the Lord. The words are very plain. “The child did minister unto the Lord, before Eli the priest.” This is the first test. Are we ministering unto the Lord, or to self and self-importance? Which is it? Do not evade this question.
“Now the sons of Eli were sons of Belial; they knew not the Lord.” Yet they were the officials of Shiloh! Is it possible to be so now? We do not ask, Is it possible for a true Christian to fail, to fall? Alas! every true Christian knows and owns it is. But is it possible for evil to be practiced by those who are in the true place, on true ground, as they say, and that by those who know not the Lord? Yes, and for the very same motive as that of the sons of Eli. Ah! it is what they can get. It is “the pan, or kettle, or cauldron, or pot; all that the flesh-hook brought up the priest took for himself. So they did in Shiloh,” (1 Sam.2:14). Wickedness was practiced at the very doors of the tabernacle at Shiloh.
“Wherefore the sin of the young men was very great before the Lord; for men abhorred the offering of the Lord.” This is the root, the practice of evil by those who know not the Lord. Mark again the contrast: “But Samuel ministered before the Lord, being a child, girded with a linen ephod.” Ah, my brethren, this is the remedy for all the evil at Shiloh, to really minister before the Lord, girded with practical righteousness. Do weigh this.
The Lord now sends a messenger, a man of God, to Shiloh (1 Sam. 2:27). God makes known the coming judgments on the house of Eli. He now makes known to us the coming judgment on Christendom, on Laodicea. But what was the marked failure of Eli? It was the allowance of evil. He seems to have been an amiable old man himself; but whilst himself condemning the evil, he was loose in allowing and going on with it. And what he allows, he is reckoned as a partaker of. And did this bring down the judgment of God on his house, and on Shiloh, where He placed His holy name at first? It certainly did, according to the word of the man of God.
And has there been no man of God in these days who faithfully warned the house of the amiable Eli, who allowed and went on with what he condemned? Is it not astonishing what light these three chapters throw on our very path in these days? Surely we can thus discern what God approves, and what He condemns. No doubt all the world may condemn the exclusion of evil, and all who bring and practice it. Nothing has been so hated and misjudged in these days, as faithful exclusion of known evil, especially evil doctrine against the Person of Christ. But does God misjudge, like man, or disapprove? Read the message of the man of God to Eli. May we all read it in the fear of the Lord. He says, “But now the Lord saith, Be it far from Me! for them that honor Me I will honor, and they that despise Me shall be lightly esteemed” (1 Sam.2:30). This is a most important word to us. We may be too much occupied with the authority of the assembly; with questions of united judgment, or a majority; although necessary to give due consideration to them, but do we know the real presence of the Lord? Do we really know Him as we should if we saw Him? This is the point. Do we really own Him present, and seek His mind? Who would rail and question the decision of a few thus gathered in His presence? Is He not really present to faith? He says it; and it will be found wherever He is truly owned.
These He will honor and preserve. It has been so in every case. But we must hasten on to
1 Samuel 3
“And the child Samuel ministered to the Lord before Eli. And the word of the Lord was precious in those days: there was no open vision.” It is so now. It is only as we are as little children, we can really serve the Lord. And though there is no open vision, no further development, yet cannot we say that the word of God is precious in these last, closing days of Christendom, as in those closing days of Shiloh? The eyes of Eli began to wax dim that he could not see. It is so wherever known evil is allowed or palliated. Dimness of perception of divine truth is sure to be the result. “And ere the lamp of God went out.” Is it not a solemn thought that the bright testimony of the Holy Ghost will soon cease to shine in this poor world, ere God shall give the rejecters up to dark and strong delusion? The night grows dark, already pagan ritualism covers the land with many a rite of Baal. Is this a time for indifference? Are the Elis and Samuels to lie down to dream, being neither cold nor hot? No, the voice of the Lord is heard, but not by Eli: “He that hath an ear to hear, let him hear what the Spirit saith unto the churches.” But he who allows the evil he condemns, has no ear to hear. Eli heard not that voice, though it had to say to him.
(Continued and to be Continued).

The Coming and Reign of Our Lord Jesus Christ: The Apostasy, Part 5

THE APOSTACY
Revelation 11. Preservation of Jewish worshippers. Two prophets, or witnesses, testifying to God’s right to the earth, during the last half-week, and slain at the close by the beast.
Seventh trumpet, or third woe. Announcement of the establishment of Christ’s kingdom introduced by His personal judgment of the quick, and closing with the judgment of the dead. The day of the Lord (Rev. 11:15-18). Revelation 11:19 commences fresh details.
Revelation 12. Vision of the birth of a man child, Christ, who is caught up to God. War in heaven. Satan, seen as the dragon, cast out from his present sphere into the earth, having great wrath, and makes war with the faithful remnant of Jews during the last half-week.
Revelation 13. Rise of the two beasts.
The first, the Roman empire, revived, and receiving power from the dragon for the last half-week.
The second, the antichristian beast, or the antichrist. Direct mimicry of Christ. Exercises all the power of the first one, working deceptive miracles.
Revelation 14. Seven visions.
First. The Jewish remnant associated with the Lamb, first-fruits for millennial blessing. Distinct from 144,000 of Revelation 7. There it is out of all the twelve tribes; here only the two (Revelation 14:1-5).
Second. Announcement of the everlasting gospel; that is the message that had been declared from the first. Call to men to fear God and look for the establishment of Christ’s kingdom, and to cease from idolatry and antichrist (Rev. 14:6-7).
Third. Announcement of the fall of Babylon, that is, the spiritual Babylon, religious corruption (Rev. 14:8).
Fourth. Threat of eternal judgment and torment on any who should worship the beast and his image, or receive his mark openly or secretly (Rev. 14:9-12).
Fifth. Blessing on those who die in the Lord from henceforth (Rev. 14:13).
Sixth. The Son of Man harvesting the earth, dividing the nations (Rev. 14:14-16).
Seventh. Gathering of the vine of the earth, and treading of the winepress of the wrath of God. Judgment on Israel in open apostasy under antichrist (Rev. 14:17-20).
Revelation 15. Vision of seven angels with the seven last plagues, in which is filled up the wrath of God (Rev. 15:1).
Vision of martyred remnant, who have been in great tribulation, victorious and triumphant (Rev. 15:2-4).
Preparation of the seven angels to execute the judgment.
Revelation 16. Pouring out of the seven vials or bowls full of the wrath of God. Compare the plagues in Egypt (Ex. 7-12).
The first four fall upon the same objects as the first four trumpets, but not limited, like then, to the third part. Notice the repeated mention of “MEN.”
The fifth. Terrible judgment on the Roman empire.
The sixth. Drying up of the river Euphrates, and gathering of the kings of the whole habitable world by Satanic agency to Armageddon, preparatory to the battle of the great day of God Almighty.
The seventh. On the air. Terrible convulsions of European civilization. Babylon brought to remembrance. Great hailstorm; fearful downpour of God’s wrath.
These awful judgments lead to blasphemy against God on the part of men (Rev. 16:9, 11, 21).
Revelation 17. Vision of a woman, gorgeously arrayed, sitting upon a scarlet-colored beast, and upon her forehead a name written, “Mystery, Babylon the great, the mother of harlots and abominations of the earth.” Figure of ecclesiastical corruption. Rome the center, supported by the civil power, the beast (or head of the revived Roman empire) and ten horns (ten kings) until the hour of her judgment, and then destroyed by them. It is the false union of church and State.
Revelation 18. Destruction of Babylon, here viewed under the figure of a mighty city, enriched by illicit commerce with the kings and merchants of the earth, receiving judgment from God.
Revelation 19. Five visions.
First. Much people in heaven praising God for His judgment on the false and wicked religious system of the world.
Second. The marriage of the Lamb. The Bride, the Lamb’s wife, the church of God. The friends called to the marriage supper, probably the Old Testament saints.
Third. Heaven opened, and the vision of Christ prepared to execute warrior-judgment upon the beast, and kings of the earth arrayed at the head of the military power of western Europe against Him. The armies following in His train, the heavenly saints (Rev. 17:14).
Fourth. The fowls called to the great supper of God.
Fifth. Summary judgment upon the beast and the false prophet (or antichrist). The armies slain.
(Continued and to be Continued).

Correspondence: Meaning of "Baptized for the Dead"

Question 37: What is the meaning of the words, “baptized for the dead” (1 Cor. 15:29)?
Answer: In 1 Corinthians 15, Paul is proving that the saints rise from the dead, some were denying it. From verses 20 to 28 is a parenthesis. 1 Corinthians 15:19 says, “If in this life only we have hope in Christ, we are of all men most miserable.” As much as to say, Christians are put to death; what is the good of turning to be a Christian, if it is only in this life. Baptized for the dead, is taking the place of those already put to death as martyrs for Jesus.
They would have no object if the dead do not rise.
Question 38: “Thy rod and Thy staff they comfort me.” Does the rod mean chastening? A. M. C.
Answer: Psalm 23, is so ordered by the Spirit as to apply to Christ here on earth, or to saints now, or in the future. The rod would signify Jehovah’s power wielded for our good, the staff our stay and support. Chastening could not apply to the Lord, but is included in what we need.
“He restoreth my soul,” is the thought of refreshment, when faint and weary, or cast down and troubled.
Question 39: What is the difference between New Birth, Conversion and Salvation?
Is Salvation instantaneous, both as to a fact before God and as to the enjoyment of it by the one that is saved?
Can one be saved and not know or enjoy it? J. H. J.
Answer: New birth is the communication of a new life in a soul. Without it one cannot understand the things of God (John 3:3). This life is wrought in the soul by the work of the Holy Spirit, using the word of God. (See John 3:5; James 1:18 Peter 1:23).
Conversion is the soul turning to God (Matt. 18:3).
Salvation is deliverance. As this word is used in various connections, we need to consider the context each time it is used. Eternal Salvation is given to the soul the moment it receives the gospel (Acts 16:31; Rom. 10:9). This is a fact before God and also for the one who believes.
Often, through careless walking, or looking into oneself, the enjoyment of this truth is marred, and doubts arise.
Saved ones are to work out their own salvation with fear and trembling, lest they should hinder the work of God in their souls. He is to do the willing and the doing for us (Phil. 2:12, 13). This is day by day.
Then we are to wait for the coming of the Lord, and when He comes He will save us out of this world (Rom. 13:11; Phil. 3:20-21).
Question 40: Who are the sheep and goats in Matthew 25:32? N. C.
Answer: These are the nations or Gentiles, the living people who help or persecute the Jews, the King’s brethren, during the tribulation period.
Those who believe the gospel of the kingdom, that is that the King (the Son of Man), is coming to set up His kingdom, and help His brethren, the Jews. These are the sheep, and they enjoy the kingdom— everlasting life on earth, while the goats, those who persecute the King’s brethren, go away into the lake of fire, prepared for the devil and his angels.

Inspiration of the Scriptures: Some Samples of its Alleged Inaccuracies, Part 5

SOME SAMPLES OF ITS ALLEGED INACCURACIES
THE QUAILS
In Numbers 11, the account of the quails has certainly puzzled many; but like some other apparently insurmountable difficulties, they vanished before those who cast themselves on God, and wait on Him to instruct them. As it stands, infidels have made a great deal of it, from the statement in our version that the quails were “two cubits high,” or, about three feet high, “upon the face of the earth,” whereas it should be above the face of the earth; that is, they would be made to fly about three feet high, so that a man would be able to take as many as he chose. We have looked into the best translations of the Hebrew that we know, and also in the Revised Version, and in all it is rendered, “above the face of the earth;” and not as infidels have said, packed from the ground for three feet high, over a distance of forty miles across.
River, Not Flood
There is a somewhat similar error of translation in Joshua 24, when Abraham is said to have been taken from the other side of the flood, as if this account made him to be living in Noah’s day, instead of long after, as Genesis 11 tells us. But the mistake is obvious to any fair mind, for flood is a word that is translated in many other scriptures, river, and means that Abraham was taken by Jehovah from the other side of the river Euphrates. This, too, is corrected in the Revised Version, and in three other of the best translations.
Sun, Stand Thou Still!
As to the sun standing still (Josh. 10), a lady said to us lately, “Of course, that cannot be true, because it is entirely contrary to the laws of nature.” To which we replied, “Let us read it: ‘Then spake Joshua to Jehovah, in the day when Jehovah delivered up the Amorites before the children of Israel, and he said, in the sight of Israel, Sun, stand thou still upon Gibeon; and thou, Moon, in the valley of Ajalon. And the sun stood still, and the moon stayed, until the people had avenged themselves upon their enemies.... and there was no day like that before it, or after it, that Jehovah hearkened unto the voice of a man; for Jehovah fought for Israel’” (Josh. 10:12-14). “That would be an impossibility,” repeated the lady, “because it would be entirely opposed to the laws of nature.” How sad to find people speaking of God as if He must be subject to any of these laws, for “with Him,” we are told, “nothing shall be impossible.” Again, we asked, “Was not the ascension of our Lord Jesus Christ entirely contrary to that law of nature termed attraction of gravitation? And will not the resurrection and rapture of the saints, when the Lord comes, be also entirely contrary to all the laws of natural philosophy?” The fact is, if in our calculations, we leave out God who is omnipotent as well as omniscient and omnipresent, there is no knowing to what length of skepticism and infidelity we may go. Well did our Lord say to some, “Ye do err, not knowing the Scriptures, nor the power of God.”
When we remember that, for many hundreds of years after the days of our Lord, the sacred Scriptures were only made known by written copies, and how difficult it is to copy anything with perfect accuracy, and then, as before noticed, add to this the possibility of errors in translation, and also the mistaken zeal of the most upright adding or erasing what they could not but think desirable from their meager or incorrect view of a passage, it is marvelous that we have the Bible so kept and preserved from the tampering of infidelity as it has been. Difficulties we all experience as to portions of the Scriptures here and there, but to upright souls God still fulfills His own word, “I will instruct thee and teach thee in the way which thou shalt go: I will guide thee with Mine eye” (Psa. 32:8).
(Continued and to be Continued).

Our Man's Rejected

Our Man’s rejected; don’t you know?
It happened many years ago.
Yea, centuries have passed away
Since it was Great Election Day
In Salem’s city—e’en the same
Where God the Lord had set His name.
From every corner of the land,
O’er mountain slope and desert sand,
With eager step they gather in,
Till, ‘mid the clamor and the din
Of busy streets, a voice is heard:
“Come now, your votes you must record.
Christ, or Barabbas; whom say ye?
Which of the twain shall I set free?”
And then, as with one mighty voice,
The multitude proclaim their choice:
“Our man’s Barabbas; set him free,
Although a murderer is he.”
“What shall I do with Jesus, then?
Am I to crucify your King?”
“Away with him!” they loudly cry;
“We own Him not; let Jesus die.”
And yet He was the Elect One,
Chosen of heaven—th’ Eternal Son.
Thus Jew and Gentile did unite
To quench the “Everlasting Light.”
Such is the World the same today
As when they cried, “Away, away
With Jesus! for no king we own
Save him who sits on Caesar’s throne.”
Then how, belov’d of God, shall we,
By that same Jesus’ blood made free,
Join in the world’s unhallowed strife,
And vote, forsooth, as if that life
Pour’d out on Calvary’s cruel Tree,
Meant nothing now to you and me?
It cannot be. Are we at one
With those who still reject God’s Son?
Are Light and Darkness so agreed
That mingled is the heaven-born seed?
Or, is this world changed so
That it is fond of Jesus now?
Ah not for if you court its smile,
As swiftly speeds this “little while,”
‘Mong worldlings one thing is clear-
“You must not speak of Jesus here;
Aware with Him—now we’ll agree”—
No room for Jesus yet, you see.
As Pilate was made friends again
With Herod, when our Lord was slain,
So is it still; and men, whose souls
Are wide asunder as the poles,
Can join their hands and loudly say:
“Our man is in—we’ve gained the day!”
And who may “our man” be, whose name
Is borne upon the wings of fame?
If you with worldlings combine,
Then he’s the world’s man and thine.
O child of heaven, can it be so?
May we have grace to answer, “NO.”
Our Man’s rejected; and, until
He comes to reign in Zion’s Hill,
Like Mephibosheth we shall mourn,
And wait our absent Lord’s return.
Why then, impatient, seek to reign
While He’s rejected? Where’s the gain?
Yea, rather, reckon up the loss,
If we thus fail to bear the cross.
We’ll hear the shout of the Harvest Home;
And then our time to reign shall come.
Election days can move us not;
For that great day, by man forgot,
Is still remembered by our God
And all redeemed by Jesus’ blood.
It seems as yesterday that He
Was “voted” to the shameful Tree.
Yet from the portals of the tomb
A mighty Conqueror He did come:
Head of the New Creation He;
Now Sons of God, in Him, are we;
To tell abroad His matchless worth
While we may sojourn here on earth,
And show, throughout our little time,
We’re children of a heavenly clime;
Content to wait th’ appointed hour
When, ‘mid the glory of His power,
The dawn shall break, the shadows flee,
And Christ shall reign from sea to sea.

Wholeheartedness for Christ

Perhaps one of the most affecting indications of Judson’s entire consecration to Christ, as his one object and theme, was afforded when he revisited Boston in broken health, after an absence of thirty years.
He was announced to address an assembly of many hundreds who had gathered from great distances to hear him. He arose at the close of the usual service, and as all eyes were fixed and every ear intent, he spoke for about fifteen minutes, with great pathos, of the “precious Saviour”; of His great love and what He had done for us, and of what we owed to Him; then he sat down again, visibly affected.
The people listened with great attention, but when the meeting was over, a friend of Mr. Judson said to him, “I am afraid the people were a little disappointed today.”
“Were they?” replied Mr. Judson; “I am sorry; but as I know my voice is still weak, perhaps they could not all hear what I said.”
“No, I do not think there was any difficulty in hearing you,” was the reply; “what I mean is— knowing, as they do, that you have lived so long among the heathens, and traveled so many thousands of miles, I think they rather expected you would tell them some interesting story.”
“Well, did I not tell them the most interesting of all stories?” he asked. “I told them to the best of my ability of One who loved them even unto death; of One who came all the way from heaven to earth, to seek and to save that which was lost.”
“Yes, I know that,” said his friend; “but still I think they expected something rather different.”
“Did they? Well, I am glad that they have it to say that a man who had traveled thirty thousand miles and who had just come from the heathens, had nothing better to tell them than the wondrous story of the dying love of the Lord Jesus. My business is to preach the gospel of Christ; and when I can speak at all, I dare not trifle with my commission. When I looked upon these people today, and, remembering where I should next meet them, how could I stand up and furnish food for vain curiosity— tickle their fancy with amusing stories, however decently strung together on a thread of religion? That is not what Christ meant when He said, ‘Go ye into all the world, and preach the gospel to every creature.’ When I meet my Master, I should not like Him to say to me, ‘I gave you one opportunity of telling those people about Me, and you let it slip that you might talk about yourself and your doings.’”

Scripture Study: Matthew 5:17-48

Matthew 5:17-20. “Think not that I am come to destroy the law, or the prophets: I am not come to destroy, but to fulfill.” This does not mean His personal obedience to the law—in that also He was ever the Perfect One. He came to make good all that was written in the law and the prophets. He came to complete all God’s mind written in them.
Matthew 5:18 declares that all must be fulfilled (Luke 24:44). His life on earth was the keeping of it without a flaw; His death on Calvary, bearing its curse for transgressors, maintained its authority. Thus He established it, and in grace bore its curse to set the believer free.
He will not weaken its authority (Matt. 5:19), but it must be the law of God and not the traditions of men, and it must be obedience from the heart and not the outward lip worship and righteousness of the Pharisees and scribes (Mark 7:6-7; Psa. 51:6). The Lord insists on godliness in His people, and works in them desires to please Him. They must judge sin in their ways if they are to walk with God.
Matthew 5:21-24 apply the law to the heart. Violence and anger that end in murder, spring from the heart. Our Lord treats all feelings of contempt or hatred as coming under the same list as murder, in the sight of God. How careful therefore we should be not to allow evil thoughts or words against any one.
Matthew 5:23. “Therefore, if thou bring thy gift to the altar, and there rememberest that thy brother hath ought against thee, leave there thy gift before the altar, and go thy way; first be reconciled to thy brother, and then come and offer thy gifts.” This is in Jewish form, but its meaning is plain, that we are at once to do all we can to be reconciled to the brother offended. We are not supposed to have anything against our brother. Notice that a brother may have something in his mind against us that we cannot remove, but we are to do our utmost to remove it, then we are reconciled to him.
Matthew 5:25 and 26. Israel should have agreed with Jehovah quickly, but did not; consequently they are now as a nation shut out from the promises and are in prison till Isaiah 40:1-2, is fulfilled. They are guilty of Christ’s death, and, like the manslayer in the city of refuge, they cannot get their inheritance till the priesthood is changed (Num. 35:25-28).
Matthew 5:27-30. Here again the evil in the heart is exposed and judged as if the sin were committed; so in verses 29 and 30, self-judgment is put before the soul as of more importance than a right eye or a right foot; no matter how valuable the thing is, it must be judged, if it leads the soul away from God. Hell here is Gehenna, and answers to the lake of fire as a figure.
Matthew 5:31, 32. The Lord declares the solemnity and lastingness of the marriage tie, going back to its original intention, so that no one can, in the fear of God, break the tie. The only reason adduced as valid to put away a wife, is, where the bond has already been broken by unfaithfulness; and to marry one put away, is to commit adultery.
Matthew 5:33-37. The disciples are warned here to let their language be simple, “yea” and “nay,” and this to them should be just as binding as an oath.
This does not hinder the Christian taking the oath in a court of justice. There he is to own the authority of God in the magistrate to govern the world, but in ordinary life anything more comes of the evil one.
Matthew 5:38-42. How different is the path of the disciples of Jesus here than Moses’ disciples. It is not retaliation or demanding justice, but “I say unto you, that ye resist not evil,” and show grace instead (Phil. 4:5). We can suffer, and commit our souls to Him in well doing as unto a faithful Creator (1 Peter 4:19) It is acceptable with God if we suffer and take it patiently; but we may show the oppressor his evil ways in what he is doing.
We could not walk in such a path with a fallen nature. Only as sustained by grace divine can we follow our Lord here.
Matthew 5:42. Notice, it does not say, “Give to him that asketh of thee, what he asketh of thee,” for then we would be often in fellowship with, and helping on, sin; but we are to give, that is, to meet the need, if we can, before the Lord, but with wisdom from Him. I might only give my advice or sympathy, yet it is meeting the need; that is before us (1 John 3:17).
Matthew 5:43-48. We can think of the Lord Jesus carrying out all this in His own life so blessedly. He loved His neighbor more than Himself. He gave His life for us. Let us ponder this portion. He shows us the Father’s goodness to all men, as a pattern for His followers. How plain are the words “Love your enemies, bless them that curse you, do good to them that hate you, and pray for them that despitefully use you and persecute you; that ye may be the children of your Father which is in heaven”—that is, in character. This is the perfection that is spoken of here.
(Continued)

Truths for Young Christians: Working for Christ, Part 1

WORKING FOR CHRIST
In our last chapter we briefly considered the Christian’s walk, his life, himself in short, as practically shown in this world. We now turn to consider his work, a subject as distinct, as we have seen, as a man’s life and habits are from his daily business, although he may carry the one into the other, Perhaps it can hardly be said which is the more important, when both are supremely so: still this, at least, is clear, that the walk must come first, and that work only is right which is accompanied by and flows from a godly walk.
Mary and Martha
There can be no doubt as to the importance of this question, which some would exalt at the expense of the inner life, others vice versa. The latter often think they are taking “Mary’s part,” and that workers are only “Marthas” after all, forgetful that the most blessed work ever done on earth was done by Mary (John 12), who lavished her money and care on the Lord’s feet, at which also she laid her glory (1 Cor. 11:15). Who, then, are at Christ’s feet now? The answer is not hard to give: not the poor, merely as such (John 12:8), but the poor of His flock on every side of us, needing our love and care.
What Scripture Says
Let us just glance at what Scripture has to say on the subject: We are created in Christ unto good works (Eph. 2:10), we are also exhorted to be careful to maintain good works (Titus 3:8-14), to be fruitful in them (Col. 1:10), to be perfect in them (Heb. 3:21), to be prepared or ready to every good work (2 Tim. 2:21, Titus 3:1), to be rich in them (1 Tim. 6:18), to be established in them (2 Thess. 2:17), to be zealous in them (Titus 2:14), to abound in them (2 Cor. 9:8), and to provoke one another to them (Heb. 10:24). Beloved, What do we really know of all this?
One thing we must be very clear about; and that is, because we have received salvation without working, we are not to lead idle lives ever afterward, put to shame by earnest, though mistaken, souls, who are, alas! thinking to win heaven by their good deeds. On the contrary, every Christian has his work to do in this world for Christ.
What Is My Work?
Dear reader, do you know yours? Or is it possible that having been a Christian one year, two years, ten years, twenty years, as you read this, you find it impossible to answer the question clearly and decidedly, “What is my work in this world for Christ?” The night is far spent, but it is not yet gone; let us, then, “who are not of the night, but of the day,” wake up and cease to slumber in our privileges and begin our long-neglected work at once.
“But what is my work?” you say.
Ah! that is a sad question for us to have to ask if we have been Christians any time at all; but if sincerely asked of God, even though so late, it will surely be answered. It is surely a most important question, for we are all members of Christ’s body, and the hand cannot do the seeing, nor the eye the walking, nor the feet the talking, nor the tongue the working. The head alone can rightly set each part of the body its appointed work.
Are you, then, doing nothing for Christ?
Well, I try to live like a Christian.
If you are occupied with the Lord, you will live like a Christian; but you have to work for the Lord too. What work do you do for Him?
I am afraid I don’t do any!
So it is, then, true that if you died this moment no soul on earth beyond the circle of nature would miss you? Alas! I have heard those who have been Christians for years confess such was the case, so useless did they feel in this world of woe and need. I am sure that many of us are quite unaware of the selfish, and idle lives we often lead. We have got so accustomed to think that if we avoid gross sins, if we are pretty regular in our reading and prayer and in our attendance at meetings and services, that we have done all that can be required of us, that we are positively surprised to hear that we are not quite so satisfactory in the Lord’s eyes as in our own, and that for years we have been neglecting, utterly neglecting the Lord’s work, our work, and it may be adding to our own sin, by hindering, finding fault with, or looking down upon, those who are more diligent than ourselves. Let no readers of these pages rest satisfied until they both know their work and are doing it.
God Will Guide the Willing
But again the question is asked, “How am I to know what my work is?”
God’s principle is, “If any man will do His will” (John 7:17); there must be first a willing mind (2 Cor. 8:12). If the Lord sees you humbly taking up what is nearest your hand, in dependence upon Him, He will show you if He would have you continue in it, or if not, He will certainly lead you into what He has ready for you. Study the parables of the talents and the pounds, and see what bearing they have on this subject; also Revelation 22:12.
Christ Must Be the Object
We must be clear in our work that Christ is the object; not that we are not to delight in it, and be zealous and active in it, but even in our hearts, the motive, the mainspring, that produces the zeal and activity, must be Christ. Otherwise the work may be useful and may be highly praised of men, but we shall get no reward, and our work loses its character of a sacrifice of a sweet smelling savor to the Lord. It is the fragrance of Christ’s name that gives the value to all we do in God’s sight. Busy bodies are no more use to God than lazy bodies, and are often more hurtful to others.
(Continued and to be Continued).

From Egypt to Shiloh: Part 7, The Spirit of Samuel

Samuel, the little, weaned, dedicated worshipper, heard. His ear was open, but at first he did not understand. Do we hear the Spirit say, Give out such a hymn; read such a portion of the Word; or lead the assembly in prayer or worship? Well does the writer remember the first time the Spirit said to him, Read the first chapter of 2 Corinthians, and the thoughts that were then impressed on his heart, though much over forty years ago. Like Samuel, he did not then know the Lord after this manner. Yes; if really waiting before the Lord, it is our privilege to be unmistakably guided by the Spirit, ever present with the saints on earth. But if we allow evil, this cannot be; and the official priesthood never thus hear the voice. Nay, in poor, fallen Christendom, the real guidance of the Holy Ghost, as to what shall be done when gathered together, is never thought of. Oh, to be a little child, and with Samuel say, “Speak, for thy servant heareth.”
Now is it not most remarkable that the doom and judgment on the house of Eli is communicated to the child Samuel? And what is the sin that brings down this terrible judgment? Is it not repeated again, as we have seen, this one thing the allowance of sin which he condemned? “And therefore I have sworn unto the house of Eli, that the iniquity of Eli’s house shall not be purged with sacrifice nor offering forever.” Very affecting are the words of the aged Eli: “What is the thing that the Lord hath said unto thee? I pray thee hide it not from me.” “And Samuel told him every whit, and hid nothing from him. And he said, It is the Lord: let Him do what seemeth Him good.” Yes; in many respects, this seems to have been an amiable, aged priest. Was it not even human kindness, or parental kindness? He might call it love, as many have done in this day. They have called it love to allow and pander to the false doctrine and evil they condemn. Have they not even slandered those who have sought to exclude the evil and give it no shelter? Oh let us all take this solemn lesson of Eli’s house to heart! Remember, brethren in Christ, judgment will begin at the house of God, as it swept away the house of Eli, at the close of the history of Shiloh.
We thus learn it is not enough to be at Shiloh. We must have the spirit of Samuel the little. Mark these results: “Samuel grew” (1 Sam. 3:19). Where there is the suited condition of soul, suited to Shiloh, there will be real growth. “And the Lord was with him.” Are you quite sure the Lord is with you? It is no Shiloh if He is not, for He is the true Shiloh. “And did let none of His words fall to the ground.” It is so now, and will be to the end. (See Rev. 3:9). Yes, all shall know this. “And the Lord appeared again in Shiloh: for the Lord revealed Himself to Samuel in Shiloh, by the word of the Lord” (1 Sam. 3:21) Nothing could be more cheering to the true Samuels in this day, though just at the end. Yes, up to the end, as at the beginning, the Lord will reveal Himself in Shiloh, in the place that He hath chosen. Wherever two or three are gathered to His name, there He will be, there He is. It is not where there is a splendid cathedral, or a splendid organ, or a gorgeous ritual, or priestly robes of cost! Not where riches and fashion are displayed. No, all this is Laodicean, and where that is, He reveals not Himself, but stands outside and knocks (Rev. 3).
Hold fast, then, this blessed fact; that to the very end, as the Lord appeared to Samuel in Shiloh, as He revealed Himself to Samuel in Shiloh by the word of the Lord, so He will now to the end. Let the house of Eli rail and misrepresent you, the Lord’s dear presence is enough the mind and heart to fill.
The house of Eli may often say, “Ebenezer,” hitherto the Lord hath helped us. There is much of such boasting in Christendom. It is the spirit of Laodicea. Do not forget that the Philistines are not far off the same place. The Philistines, those who are in the land, but not of it, are gathering and preparing their forces. Shiloh was destroyed; Shiloh as a place came to an end. Samuel went to Ramah, his home— Ramah, “the high places.” Christendom will indeed be destroyed; but the church of God —Samuel, so to speak —will be caught up to the high places, and be seated around the throne of God in glory. From Ramah Samuel judged Israel (1 Sam. 7:17) Paul says, “Do ye not know that the saints shall judge the world?” (1 Cor. 6:2).
We will close these brief remarks with a short review.
1. A redeemed people, not only sheltered by the blood, but brought out of Egypt.
2. They must also be brought in, through the Jordan, into the land.
3. Then, when they had rest and possession, the Lord placed His name in Shiloh.
4. For centuries Shiloh was almost forgotten.
5. The great revival of Shiloh in 1 Samuel 1-3. To pursue the type, we have Ruth, the bride before the reign. Then Saul, head and Shoulders above the rest. Then follows the reign of David.
Thus also the church is redeemed by the blood of the Lamb. Brought out of darkness and slavery, and blest in Christ in the heavenlies. All this must be known ere church position can be understood. The church, or even two or three, are now gathered to the risen Christ, the true Shiloh, He “whose it is.”
Blessed place of peaceful tranquility. This was practically lost and unknown for centuries. Then, as Shiloh was so remarkably revived in 1 Samuel 1-3, so in these last ninety years, the true principle of being gathered together has been restored. That is, to Himself, the only One whose right it is. As there were two parties, or families, then, so again now: One who has allowed the evil they condemned; the other desiring to exclude all evil in separation to Christ, the only Shiloh.
May the Lord apply His truth to us all. May Hannah’s song be ours, however reviled as exclusive. May we learn in this lesson what is pleasing to the Lord! May we know the Shiloh, Emmanuel, “He whose it is.” May we honor Him, cleave to Him, glorify Him, for He alone is worthy. “Worthy, O Lamb of God, art Thou.”
In the midst of the redeemed in glory, Thou shalt be the Shiloh. When Thou shalt come to this poor, sad earth, Thou, whose it is, shall have the glory. All nations shall worship Thee. Thine be the glory forever and ever. Amen.
(Continued)

The Coming and Reign of Our Lord Jesus Christ: The Apostasy, Part 6

THE APOSTASY
We may note the contents of the remaining chapters further on.
Let us now turn to the Second Epistle to the Thessalonians, where we have many details of the circumstances connected with the apostasy — a passage to which we have already referred in showing the removal of the heavenly saints.
Now we beseech you, brethren, by the coming of our Lord Jesus Christ, and by our gathering together unto Him, that ye be not soon shaken in mind, or be troubled, neither by spirit, nor by word, nor by letter as from us, as that the day of the Lord (New Trans.) is at hand. Let no man deceive you by any means: for that day shall not come, except there come a falling away (or an apostasy) first, and that man of sin be revealed, the son of perdition (the antichrist); who opposeth and exalteth himself above all that is called God, or that is worshipped; so that he, as God, sitteth in the temple of God, showing himself that he is God” (2 Thess. 2:1-4).
The elements which go to form the apostasy are already being manifested, but they will not ripen to a head until after the rapture. We must not look at that event as a date.
The Apostle beseeches the Thessalonian saints, by the coming of our Lord Jesus Christ, and by the translation to meet Him in the air, that they should in no way be disturbed or deceived by any man, as that the day of Christ (or “the Lord”), that is, the day of judgment, had come. (It is important to distinguish between the terms “the day of Christ,” and “the day of the Lord.” The former begins when Christ comes for the church, and continues until He comes out of heaven; the latter begins when He comes out of heaven and continues until the new heavens and the new earth are established.) It appears that even an anonymous letter had been sent them to this end, purporting to come from the Apostle and his fellow-servants (2 Thess. 2:2). Before that, they would all be gathered together unto Him, caught up to meet Him in the air; and also the apostasy would come, and that man of sin, the son of perdition, would be revealed. All those events must transpire first. The Jews must go back to their own land, rebuild the temple of God, and receive the wicked one, the false messiah, who should come in his own name (John 5:43), and whose coming is after the working of Satan, with all power, and signs, and lying wonders, and with all deceivableness of unrighteousness in them that perish, because they received not the love of the truth, that they might be saved (2 Thess. 2:9-10).
Satan is blinding and deluding the minds and hearts of men now, lest the light of the gospel of the glory of Christ should shine into their hearts and they should be saved. In this coming hour of trial, God will be dealing judicially, and shall send “strong delusion, that they should believe a lie: that they all might be damned who believed not the truth, but had pleasure in unrighteousness” (2 Thess. 2:11-12). If men will not have Christ, they must have antichrist; if they will not bow to God’s truth, delusion (or a working of error), will be sent, and they will bow to the devil’s “lie.”
This man of sin, having deceitfully exalted himself amongst God’s earthly people, opposeth and exalteth himself above all that is called God, or that is worshipped, and seats himself in the temple of God at Jerusalem, showing himself that he is God. This will be one of the culminating acts of wickedness perpetrated by this arch-leader of devilry on the earth; the heading-up of the awful mystery of iniquity, that will draw down the judgment of God. “Remember ye not,” continues the Apostle, “that, when I was yet with you, I told you these things? And now ye know what withholdeth that he might be revealed in his time. For the mystery of iniquity doth already work: only He who now letteth (or hindereth) will let, until He be taken out of the way. And then shall that Wicked (one) be revealed, whom the Lord shall consume with the Spirit of His mouth, and shall destroy with the brightness of His coming” (2 Thess. 2:5-8).
Here we have clearly revealed the fact that there is a power which restrains the increasing flood of evil, but that at a certain moment this power will cease to withhold. The mystery of iniquity had already commenced to work in the days of the Apostle, and has continued in varied phases ever since; things now get worse and worse; but there is One who restrains the bursting forth of this terrible evil. I doubt not that the One spoken of is the Spirit of God, who restrains evil both through the powers that be, which, though ordained of God (Rom. 13:1-2), will eventually fall under the power of Satan, and also as a divine Person dwelling in the church of God. This latter will cease at the rapture of the saints; the former, I judge, will probably continue a short period, possibly till the casting out of Satan, the great dragon, to the earth, consequent upon his defeat by Michael, the archangel, in the war in heaven (Rev. 12:1-10).
Losing his place on high as accuser of the brethren, opposing the intercession of Christ on their behalf, the devil will then come down, having great wrath, exercise his power on earth, energizing both the Roman empire, headed by the beast; and also the antichrist or false prophet. The restraining power being taken out of the way, and the evil no longer being held in check, Satan will bring matters to a crisis. The wicked one will be revealed, coming out in his true colors, the masterpiece of Satan, with such awful deceit, that if it were possible, the very elect would be deceived (Matt. 24:24). They continue for a while, and then receive summary judgment at the Lord’s hand (Rev. 19:20).
In the dragon, the beast and the false prophet we have a kind of Satanic anti-trinity.
The dragon gives his power, his seat (or throne), and great authority to the beast; also a mouth speaking great things, and blasphemies, and power to continue (make war, marg.) forty-two months; that is, Daniel’s last half-week (Rev. 13:1-10). The term “beast” (or wild beast) is applied both to the Roman empire, and also to the head of it, the one who wields its power. This individual corresponds with the little horn of Daniel 7:8,11,20,25, where we again read of a mouth speaking great words against the Most High. Under this terrible being there will be ten kings, at the head of ten kingdoms, within the territory of the empire, and who will receive power as kings one hour, or at the same time with the beast (Rev. 17:12). These have one mind, and shall give their power and strength to the beast. He will be the head of the Gentile apostasy in the West, a false king of kings and lord of lords.

Correspondence: Rom. 13:14; Acts 2:16-18; 1 Tim. 5:8

Question 41: What does Romans 13:14 mean? A. G. P.
Answer: Read Romans 13:11-14. We are waiting for the Lord’s coming, and our behavior is to correspond with our character as children of light. We are to walk honestly, as in the day. We are not to give way to the lusts of the flesh, but make the Lord Jesus our pattern, and thus put on in our practical life, the Lord Jesus Christ, making no provision to indulge in worldly desires and pleasures.
It is quite right to make provision for our households and bodily needs. The contrast in verse 13 will help to make plain what the Apostle meant.
Question 42:What is the meaning of Acts 2:16,17,18: “But this is that which was spoken by the prophet Joel,” in reference to “your sons and your DAUGHTERS shall prophesy?” W. H. E.
Answer: It was so marvelous to hear men speak “the wonderful works of God” in tongues they had not learned, that in amazement they say, “What meaneth this?” Others said, “These men are all full of new wine.” Peter answers, “These men are not drunken as ye suppose.” “This is that which was spoken by the prophet Joel.” Joel prophesied of the pouring out of the Spirit on all flesh when the millennial reign of Christ begins, in the time of Israel’s restoration; and extraordinary signs as in Acts 2:17-18, will accompany it. But, notice, Acts 2:19-20, show signs of a different kind, in heaven and earth, in sun and moon. These will precede the blessing, as said, “before the great and notable day of the Lord come.”
What we see in Acts 2, is the same Spirit that now indwells believers only, and baptizes them into one body (1 Cor. 12:12-13), and forms the limited aspect of the house of God (1 Tim. 3:15; Eph. 2:22); but this is not yet the fulfillment of Joel’s prophecy; it is a different people who now receive the Spirit.
In Acts 21:9, Philip’s four daughters prophesy. (See also 1 Cor. 11:4-5.) We do not get women in Scripture praying in public or preaching to saints or sinners. So we need to think of them exercising this gift where and when it would be according to Scripture.
Question 43: Does 1 Timothy 5:8 include the thought of one laying by in store against death: as by life insurance and the purchase of a house for the benefit of those dependent upon him? If not, what scripture would show the Lord’s mind in regard to those matters? F. F. S.
Answer: 1 Timothy 5:8 speaks of a believer’s duty to provide for the present need of those dependent upon him, and this extends sometimes to caring for a mother or sister (1 Tim. 5:16), that they might not be a charge to the assembly. Unbelievers, with only natural affection to guide them, will care for their households. If a believer did not, it would be a reproach to the truth.
Matthew 6:19-34 tells us not to be anxious about the future. We can only provide for them according as God has prospered us; the rest we are to cast upon Him (Phil. 4:6).
Man is a steward of what is committed to him. The believer with a heavenly calling is also a steward of all the Lord gives him. If he allows his heart to be taken up with earthly things, his spiritual enjoyment is hindered; he does not enjoy “his own” portion, the “true riches” (Luke 16:11-12). He is responsible, and therefore should seek guidance from the Lord (Prow, 3:6), when to spend, when to keep, when to give, and how to trade with it (Luke 19:13).
It is against natural affection, and it would certainly be wrong, not to do our best to provide for our children (2 Cor. 12:14): at the same time we should seek to teach them dependence on God, and exercise our own hearts to walk in it, avoiding a covetous spirit that desires what we have not, lest to get it, we leave the path of dependence upon God, for some invention of man’s, that appeals to our natural selfishness, by offering us something large for very little, and, what is worse— for it destroys dependence on God— promises immunity from need, thus depriving the soul of the exercise needed to walk with God.
The path of faith is to trade with what God has put into our hands; to go beyond this is getting into debt, to trade with other people’s capital. We should avoid debt, and seek to be content with what God has given us (Matt. 6:24; 1 Tim. 6:9-11).
There is nothing to hinder us buying a house, or whatever we may need, if we pay for it honestly, thus doing all for the glory of God (Col. 3:17).

Inspiration of the Scriptures: Some Samples of its Alleged Inaccuracies, Part 6

SOME SAMPLES OF ITS ALLEGED INACCURACIES.
THE THRESHING FLOOR
Take another case. Rationalists tell us there is a serious contradiction between the account given in 2 Samuel 24:24 and 1 Chronicles 21:25, as to David’s purchase of the threshing floor from Araunah. This allegation is now again being widely circulated. It says in Samuel that David bought it for fifty shekels of silver, and in 1 Chronicles the price is six hundred shekels of gold by weight; so our opponents tell us both cannot be true. The fact is, however, that both are perfectly correct. In 2 Samuel we learn that he bought the threshing floor and the oxen for fifty shekels of silver, and in 1 Chronicles 21:22,25 we are told that he bought “the place” for six hundred shekels of gold by weight. It is obvious that “the place” might have extended over a large area beyond “the threshing floor.”
The Census of Israel and of Judah
In the same chapters, another question has been raised; because in 2 Samuel 24:9, when the census was taken, we are told there were in Israel eight hundred thousand valiant men that drew the sword; and the men of Judah were five hundred thousand men; whereas, in 1 Chronicles 21:5. “All they of Israel were a thousand thousand and an hundred thousand men that drew sword, and Judah was four hundred, three score and ten thousand men that drew sword.” It is alleged that there is a contradiction as to the numbers given; but let us consider, that in Samuel it is the valiant men that are enumerated, and in 1 Chronicles all that drew sword, making it possible there were then among them those who, though they drew sword, would not be ranked among David’s valiant men; and of Judah we find in Samuel there were five hundred thousand men, and in 1 Chronicles only four hundred and seventy thousand of them drew sword, whether from old age or any other reason we are not told.
Now all thought of inaccuracy as to these sacred Scriptures vanishes, and we find, that the more we prayerfully ponder the Word, in humble dependence on the Holy Spirit, the more divinely perfect the written Word appears; so perfect that we are struck sometimes with the importance of the addition or omission of a single letter. For instance, when reference is made to Christ as the seed of Abraham, the Holy Spirit by Paul says, “He saith not, And to seeds, as of many; but as of one, And to thy seed, which is Christ” (Gal. 3:16,) Again, we read of Christ being “the Lamb of God, which taketh away the sin [not sins but sin] of the world” It is obvious that if Christ had taken away the sins of the world, the world would have been saved; but the new heaven and the new earth will then show that He has taken “sin” completely out of the world, and that there righteousness dwells. Again, we find the written Word saying sometimes, “that the Scripture might be fulfilled,” but when our Lord in Gethsemane spoke of His competency to have from His Father, if desired, twelve legions of angels, He added, “But how then shall the Scriptures be fulfilled?” and again, “that the Scriptures of the prophets might be fulfilled.” And why Scriptures and not Scripture? Because all the prophetic writings as to His humiliation, rejection, sufferings, pain, forsaking, must all have their accomplishment in His death as a sacrifice and offering to God on the cross.
Before leaving our consideration of the Old Testament, it is interesting to notice that, early in the book, we find man’s utter ruin and total unfitness for God’s holy presence; then we have largely and repeatedly set forth in types that it is only by the shedding of blood there can be remission of sins or approach to God; and, toward the end it is plainly written that the “just shall live by faith.” Thus, sinner as man is, in virtue of the blood of Jesus, he is cleansed and justified before God on the principle of faith. Peter put it simply when he said, “To Him give all the prophets witness that, through His name, whosoever believeth in Him shall receive remission of sins” (Acts 10:43). Such is God’s unutterable goodness and grace, long ago declared by the Holy Spirit through His ancient prophets; and such grace abounds still to every one who takes his true place before God as utterly unclean, lost, and unmendably bad, and who has to do with the precious blood of Christ, as his only ground of peace and title to glory. How truly the believer can say of the Holy Scriptures:
“Here the Redeemer’s welcome voice
Spreads heavenly peace around;
And life and everlasting joys
Attend the blissful sound.
O may these heavenly pages be
My ever dear delight;
And still new beauties may I see,
And still increasing light.”

The Lost Crown

“So run that ye may obtain.... Now they do it to obtain a corruptible crown, but we an incorruptible” (1 Cor. 9:13).
Among my early acquaintances in my Christian course was a young man, who was very zealous for the honor of God upon earth, and also for the salvation of souls. But after a time, the interest of a young family, and his success in the world as a man of business, gradually drew his attention from eternal things to the cares of this life: and often, when those with whom he usually met in Christian fellowship were assembled together for worship, and speaking to each other of the things of God, he was to be found at home with his ledger, attending to his accounts, and arranging what must be done on the morrow.
This went on for several years, not, however without many warnings from the Lord, and admonitions from his brethren, till he was nearly forty years of age, when consumption set in, and gradually he was reduced to almost a skeleton. Still his business absorbed his attention as long as he was able to be about. But the time came when he was laid aside entirely, and then he saw his sin and folly; and more than once when I have been with him in his last hours he has spoken to me in language as follows:”O, how unwise and wicked I have been to do as I have done, to neglect the things of God for the cares of this life, and to prefer what I thought was my duty to my family, to the society and communion of His people. Although my conduct before men has been blameless, yet the last twelve years of my life are lost, quite lost. It would have been better for me had they never been given me, because I have used them for myself, and not for the Lord. I have been careful about my own things, and not the things which are Jesus Christ’s. I shall blush when I see His face (1 John 2:28). I shall hold down my head as I go into His presence, for I have deprived Him of the joy He would have had in saying to me, ‘Well done, good and faithful servant;’ For how can He do so, when I have been so unfaithful?”
“I know He has forgiven me, but O, what a loser I have been in my own soul! and I shall also be a greater loser, in that day of glory when the Lord Jesus will give to every one according to his works (Rev. 22:12). I shall see others come forward in that happy day and take their crowns of reward, and their positions in the glory, for faithful service done to Him while they were upon earth; but I shall have none to receive, because I have allowed this present world to rob me of them. I shall rejoice to see others receive that which I have forever lost through my unfaithfulness” (Rev. 3:11; 1 Cor. 3:15; 2 John 8).
“I am thankful that I know I am saved through His death. The blood of Jesus Christ, God’s Son, cleanseth from all sin, and I know I shall form a part of that favored company which will surround His throne in the glory, and join in singing that happy song, saying, `Thou art worthy to take the book, and to open the seals thereof, for Thou wast slain. and hast redeemed us to God by Thy blood, out of every kindred, and tongue, and people, and nation, and hast made us unto our God kings and priests, and we shall reign on the earth.’ But where will be my honors of reward for faithful service done to the Lord in this world?” (John 12:26; 1 Thess. 2:19). I shall have none; I have lost my crown.
“O, how foolish I have been to make such an exchange, to neglect that which is eternal for the things of this world, which are perishing with the using, and which I must now leave forever.”
“How I have dishonored Him who bought me with His own blood; and made me His forever. How unlike in my walk have I been to the Apostle Paul, who, when speaking of the things which were gain to him, those he counted loss for Christ, and said, ‘Yea, doubtless, and I count all things but loss, for the excellency of the knowledge of Christ Jesus my Lord, for whom I have suffered the loss of all things, and do count them but dung, that I may win Christ.’” (Phil. 3:7-8).
“But how very different to this have I been, and because I have been so unlike him in my walk through life, I am unlike him now in my last moments: When he was about to leave this world, he could say, ‘I have fought the good fight, I have finished my course, I have kept the faith; henceforth there is laid up for me a crown of righteousness, which the Lord, the righteous Judge, shall give me in that day” (2 Tim. 4:6-8).
“But with me it has been just the opposite to this: for instead of my having fought the good fight, I have been gathering of this world’s goods, and seeking to enrich my family, and make myself comfortable and at home in this scene, which has crucified my Lord and Master. And therefore instead of finishing my course with joy, I am cut off in the midst of my days, and am full of sorrow and regret. Neither have I kept the faith, for I have been very unfaithful, and have not obeyed the voice of the Lord Jesus, when He said, ‘If any man serve Me, let him follow Me.’ This I have not done; I have not followed Christ through this world, therefore I know there is no crown of righteousness laid up for me, as there was for Paul. I have not loved His appearing; and the time for service which was given me is past, it is gone forever. The Lord in His great mercy is now taking me away from the earth, because He knows the longer I am left here, the more I shall be taken up with the things of this world, I own His goodness and love, and bow with thankfulness because it is His own hand which is doing it.”
“O, dear Mr.- let my loss be your gain, and let me entreat you to use faithfully every moment of time which is given you in this word and everything you possess, for Christ, and I shall have the delight of seeing you in that day, have an abundant entrance into the everlasting kingdom of our Lord and Saviour Jesus Christ. It will please me very much to see you receive from His hands your crown and place of honor in the glory, while I shall take a much lower position, because I have lived to myself and not to Him who died for me and rose again.”
To this I could make no reply, because I knew he had lived to himself; he had attended the meetings of God’s people on the Lord’s day, but in a cold, formal way, with but little enjoyment in his own soul, or help to others; and I was glad to hear him make such a full and beautiful confession before he left the world; and it has not been without its good effect upon my own soul; for when I have felt disposed to absent myself from the meetings of those who love and follow Christ, or to turn aside from the path of separation from the world, I have remembered the lost crown, and it has helped me to break the snare.
But the night in which he died was still more solemn. There were two or three Christians in the room with him, when he asked them why they had put out the light, they told him they had not done so, the candle was burning as usual. He then knew his sight was gone, and therefore his end was very near. Then after lying quiet for a short time, he all of a sudden raised himself up in the bed with great amazement and delight, and lifting up both his hands, said, “O! O! there is something going to take place. The Lord Jesus is just about to leave His Father’s throne, and come in the clouds to call His saints up to meet Him in the air. DON’T SLUMBER, NOR SLEEP.” He then laid himself back on the pillow, and breathed his last.
This is the dying testimony of one who belonged to Christ and it is very solemn, not only as it regards the near approach of the Lord Jesus to gather up His saints to meet Him in the air; but also as to our faithfulness to Him during His absence. It also shows the great desire this dying brother had, that we should not do as he had done, but that we should be watchful and sober; and though I know that this is not the time for revelations, but believing the written Word of God, yet many times since, when I have felt my eyes begin to grow drowsy and myself tired of watching, I have remembered his dying words, and they have helped to quicken my step onwards, to run with patience the race set before me, looking unto Jesus; and surely whatever will help to do this in such days as these is very valuable, when everything around us is so calculated to make us grow weary and faint in our minds, and hide that day of glory from our view; and when so many whom we believe to have eternal life through believing in the Lord Jesus Christ, yet like this young man, are allowing themselves to be deceived by present things, thinking they can make the best of both worlds, as if the Lord Jesus had not been down here, rejected and crucified.
Not that our salvation depends on our doings, but on what the Son of God has done for us, and those who are saved must hate iniquity; it cannot be otherwise. But Scripture tells very plainly that believers are saved by grace, through faith, and that not of themselves, it is the gift of God, “not of works, lest any man should boast, for we are His workmanship, created in Christ Jesus unto good works, which God hath before ordained that we should walk in them” (Eph. 2:9-10). And we are also told that God “hath saved us, and called us with an holy calling, not according to our works, but according to His own purpose and grace, which was given us in Christ Jesus before the world began. But it is now made manifest by the appearing of our Saviour Jesus Christ, who hath abolished death, and hath brought life and immortality to light through the gospel” (2 Tim. 1:9-10). But it is also very clearly revealed in Scripture that those who are thus saved may, through carelessness of walk, become blind and not see afar off, and forget that they were “purged from their old sins” (2 Peter 1:9). They are not able to look back to the cross, or on to the glory, with clearness and certainty, therefore they are not living in the enjoyment of that which is really true of them, and are great losers, both here and in that coming day (1 Cor. 3:13-15; Phil. 4:4).
We are told over and over again in God’s Word, that the Lord Jesus will give to those that are saved, different positions in the glory, according to their works (Rev. 2:23; 21:12).
They who have sown sparingly shall reap also sparingly, and they who have sown bountifully shall reap also bountifully (2 Cor. 9:6). We read of some having that which they have been building, burnt up, but they themselves saved, yet so as by fire (2 Cor. 3:15). Others again will have an abundant entrance into the everlasting kingdom of our Lord and Saviour Jesus Christ (2 Peter 1:11). The white stone, the new name, the hidden manna, are all marks of special favor for those who have been faithful to the Lord Jesus upon earth during His rejection. And O, how solemn the thought, that we who are linked up there with Him at God’s right hand, should seek to please ourselves down here in this world where He is still despised and rejected; knowing too that He is coming soon, coming in a moment, in the twinkling of an eve, and before another twinkle we may see His face, and hear His voice, and be with Him forever. How very important therefore, it is, that God’s people should ever remember that beautiful exhortation, “Therefore, my beloved brethren, be ye steadfast, unmovable, always abounding in the work of the Lord; forasmuch as ye know that your labor is not in vain in the Lord” (1 Cor. 15:58).

Scripture Study: Matthew 6

Matthew 6:1-18 is the spirit in which the disciples were to perform good works.
Matthew 6:19-34 is separation from the spirit of the world and from its anxieties.
Matthew 6:1-4. The disciples are to have the sense of being under the Father’s eye. Their inner man is with the Father which is in heaven, and this molds their conduct. If they did their righteousnesses to be seen of men, that is all the reward they would get. If done in secret before the Father, He would openly reward them.
“Let not thy left hand know what thy right hand doeth.”
Matthew 6:5-15. This secrecy is applied also to prayer: “Enter into thy closet” “Shut thy door.” It is to tell out their hearts to the Father, and “He will reward thee openly.” It is not saying prayers, nor “much speaking’’, nor “vain repetition”; the Lord Jesus and the Apostle Paul each prayed three times for the desires of their heart (Matt. 26:44; 2 Cor. 12:8) Supplication is prayer intensified, and with what assurance they could ask, when they knew that “the Father knoweth what things ye have need of before ye ask Him.”
We get no forms of prayer in Scripture. The prayer the Lord taught His disciples was not a form to repeat, but a model to teach them to pray. It was suited to the time before the Lord died, and before the Holy Spirit was given.
We need again to notice the difference of their position from ours. They had an earthly calling, so were taught to pray, “Our Father which art in heaven.” “Heavenly Father,” or “Father which art in heaven,” is not used after redemption is accomplished. We, Christians, have a heavenly calling, are “blessed with all spiritual blessings, in heavenly places, in Christ” (Eph. 1:3); we are also sealed with the Holy Spirit, and say “Father,” in nearness and relationship (Rom. 8:15, 26; Gal. 4:6). Also we pray now in Christ’s name (John 16:23-24). Thus they and we are taught to think whom we are addressing.
“Hallowed be Thy name,” expresses the holy reverence they should have in the Father’s presence.
“Thy kingdom come” is the Father’s kingdom; all that “die in the Lord,” and all the living that are changed and caught up, will have their place in the heavenly part of it (Matt. 13:43). Israel and the Gentiles who are saved through the tribulation period, will have a place in the kingdom of the Son of Man on earth, when the Lord comes to reign. This is what they prayed for. We are waiting for the coming of the Lord to gather His church home, and now pray that the gospel may reach many souls till His body, the church, is completed.
“Thy will be done in earth, as it is done in heaven”; this is looking for the fulfillment of the prophecies, and will be fulfilled in the new heavens and the new earth; but now it applies itself to our lives here. Jesus, the Faithful witness, did the Father’s will here on earth. He leads us to walk as He walked.
Up to this point they have asked nothing for themselves; teaching us that in prayer, God’s glory and the advancement of His interests should have the first place. Next comes what concerns ourselves: “Give us this day our daily (or needed) bread.” This is expressing our dependence on God. Thanksgiving would surely accompany this.
“And forgive us our debts, as we forgive our debtors.” Those disciples did not yet know eternal forgiveness through the work of Christ; it was governmental or daily forgiveness. We know and have eternal redemption (Heb. 10:14; 1 John 2:12), but we also need daily forgiveness in connection with communion, if we would as children walk with the Father and the Son. We are told to confess our sins (1 John 1:9). We cannot ask forgiveness, for, “He is faithful and just to forgive us our sins, and to cleanse us from all unrighteousness.”
We forgive others because we are already forgiven (Eph. 4:32; Col. 3:13); but if we do not forgive others, What then? Then our Father will need to deal with His naughty children (Heb. 12:5-11). We will lose the sense of His grace in our souls till we do forgive (Mark 11:25-26). This is God’s government over His family.
Israel as a nation are now cast off because of their ways (Matt. 5:25-26; 18:23-35; 1 Thess. 2:14-16); but they will be restored as a nation (Isa. 40:1-2).
The gospel takes in individuals of every nation in that word, “preach the gospel to every creature” (Mark 16:15).
“And lead us not into temptation, but deliver us from evil” expresses their felt weakness and tendency to evil, as if one said, “Father, I have no strength, do not try me, or I will fall; I need Thee to keep me all the way.” There in reality the prayer ends, then the Lord emphasizes the seriousness of allowing an unforgiving spirit (Matt. 6:14-15). May we all take heed to it.
Matthew 6:16-18. Fasting expresses humbling and denying of self, and where it is real before God, it is to be out of sight of men, “and thy Father, which seeth in secret, shall reward thee openly.” Fasting gives deeper reality to the exercises of the heart before God.
Matthew 6:19-21 tell us where their heart’s object was to be. If the Lord is their treasure, there will their heart be also. They are enjoined not to allow their affections to go after things of earth, or disappointment will result, for loss and decay are felt in temporal things. Nothing fades and no robber can steal the heavenly treasures that are in Christ.
We now are Christ’s treasure, and His heart is ever with us. Bless His name!
Matthew 6:22-23 lead the disciples to examine whether the light they are walking by is of man or from God. The Pharisees thought they had light, but it was men’s traditions: it was real darkness (Compare Matt. 15:14, with John 8:12).
Matthew 6:24. It is impossible to serve two masters. If self is allowed, covetousness (which is idolatry, Col. 3:5), goes after mammon; if our object is Christ, God’s claims over us are owned in the soul. Paul said (Phil. 1:27), “To me to live is Christ.”
Matthew 6:25-34. The Lord would remove from His disciples all anxious care. The fowls of the air neither sow nor reap, nor lay up in store; “your heavenly Father feedeth them,” ‘‘Are ye not much better than they?”
And why take thought about raiment; the lilies neither toil nor spin, but surpass Solomon in all his glory. If God clothes the grass of the field, which is but for a day, shall He not much more clothe you, O ye of little faith? “Therefore take no (anxious) thought, saying, what shall we eat? or what shall we drink? or wherewithal shall we be clothed? (for after all these things do the Gentiles seek:) for your heavenly Father knoweth that ye have need of all these things. But seek ye first the kingdom of God, and His righteousness; and all these things shall be added unto you,” We are not told to seek these things second: Our Father, who knows our need, will give them—“shall be added unto you.”
“We have nothing to do with tomorrow,
Our Father will make that His care;
Its grace or its strength we can’t borrow,
Then why should we borrow its care.”
“Sufficient unto the day is the evil thereof.”
Note. This does not interfere with our lawful duties, such as parents providing for their families’ need. It is to check anxious care, teaching us to trust the Father, who loves us, to supply what we need and have no means of getting.

Truths for Young Christians: Working for Christ, Part 2

WORKING FOR CHRIST.
THERE is WORK READY FOR YOU.
The variety of work is endless, and may range from pastoral care over hundreds of God’s people, to giving a cup of cold water in Christ’s name. There is work suited to each, and there is work suited to you. Take the most difficult possible position for active service—that of a young girl brought up in the seclusion of the family circle, which she has not yet left; it may be with no opportunities of visiting the poor and afflicted (though this is very rare), What can she do? What can she not do? If she has a heart, is rather the question. Has she unconverted relatives and friends, any for whose souls she particularly cares? Can she not do a real work for Christ by sending them regularly—it may be unknown to them—gospel books and papers, accompanying each with earnest prayers? And when that relative or friend is saved, none may know save the Master and the workman, to whose instrumentality it is due. Prayer, definitely continued for others, is a very real work for the Lord. But all work involves some amount of self-denial, and above all, steady perseverance. How many lives of service have been given up, through want of this one necessary quality!
Idleness Injures Everybody
The Lord’s work must be done, and if we do not do it, He often has to set others to do our work; but, of course, if the hand is paralyzed, and the foot has to act in its stead, it cannot do the work as well; especially as it has its own besides. Idleness, therefore, is a great evil, causing not only some to suffer from neglect, but others who are willing, to be overworked, and, after all, the work is not so well done. Consider, then, if ever you are tempted to criticize the work of another, whether that servant may not be doing double duty for some lazy Christian who will do nothing, and it may be that “thou art the man.
Let us, then, encourage one another in the work of the Lord, and see that none of us are mere lookers-on, for a looker-on is generally a fault finder. Let us remember, too, that our labor is not in vain in the Lord, but that our loving Master is only too glad to give each one His full reward of praise for every bit of work done in His name, and that will therefore stand the fire.
The time is short, and much has been wasted by all of us; before the Lord’s return, then, let each of us be found steadily at our posts, working for Christ.
“With the first faint blush of morning
Hasting from thy still retreat,
Labor on until the evening,
Heedless of the noontide heat.
Labor till the far horizon
Paleth with the setting sun:
Then the Master’s voice shall greet thee.
With the welcome words, ‘Well done!’”

The Breaking of Bread: Acts 20:7

“This do ye, as oft as ye drink it, in remembrance of Me. For as often as ye eat this bread, and drink this cup, ye do show the Lord’s death till He come” (See Luke 22:19; 1 Cor. 11:24-26)
Our first thought should be, We have come together to meet the Lord; He is in the midst (Matt. 18:20). We are in His presence.
“Gathered to My name” is the condition attached to the promise of His presence.
We can claim the promise, if we are gathered in holiness and truth, for He is “holy” and “true” (Rev. 3:7). To Him alone the Holy Spirit, who is also called the Spirit of truth, gathers us in separation from evil and in the unity of the Spirit.
Have we His presence in all our meetings? Yes, the promise applies to them all: breaking of bread, prayer, open meetings for ministry, study of the Word, or discipline, whenever we come together as an assembly.
Gospel meetings or those held on the responsibility of a servant of the Lord, are not assembly meetings, but may have the fellowship of those so gathered.
We should therefore keep before us the character of the meeting so that our hymns, scriptures read, worship or prayers, may be appropriate for the occasion.
When we come together to remember the Lord, and thus show His death till He come, which leads our hearts to worship Him, would not the Holy Spirit lead our thoughts towards the sufferings of the Lord, the lamb of God without blemish and without spot, and to the love that led Him down here to die for us? We remember Him in death.
The finished work and consequent glory of the Lord connects itself with this, as what has given us a standing before God. Thanksgiving fills our hearts for these blessings, but the occasion is higher than our blessings. We think of Himself and of His sufferings, and His perfection in them, and how He glorified God about sin.
We worship Him. “This do in remembrance of Me” expresses His desire that our hearts should follow Him in His path of suffering that led Him in love into death for us.
In keeping with this, we can see that if hymns are sung or scriptures are read before the breaking of bread, it should be such as would lead our thoughts into fellowship with the Saviour in His sufferings and death, of which the bread and the cup are symbols.
Ministry or prayer should be left till the Lord has had His portion in the adoration and praises from the hearts of His redeemed people.
We do not want nor need rules, for we have Himself in the midst. We have liberty, not for the flesh, but of the Spirit. We are guests at His table; deep reverence therefore becomes us in His presence.
Avoid a criticizing spirit, for that destroys worship in the soul, robs it of its happiness, and the Saviour is robbed of His praise from it. We must not be occupied with failure in each other. The Lord bears the iniquity of our holy things (Ex. 28:38). If anything comes in that tends to mar, let the heart cleave still closer to Christ, the eye more firmly fixed on Him, seeing no man but “Jesus only,” that praise to Him in our hearts may not be hindered.
It is a time for worship, where we present and He receives our “spiritual sacrifices, acceptable to God by Jesus Christ” (1 Peter 2:5).
If the Lord leads us to some ministry after the remembrance of Him, it is good; but do not let us cut short His praises, if there is liberty in worship to go on. It is not out of order before separating for a brother to commend us and all the church of God to His care, remembering tried ones and absent ones, and giving the assembly as such an interest in praying for the gospel towards the unsaved.
The prayer meeting, which is next in importance, has a different object. We come to tell the Lord our desires for His glory and His people’s good, and for the spread of the gospel. Someone has said, “The prayer meeting is the pulse of the assembly and shows its condition.” Those who neglect it, neglect a great privilege of fellowship with the Lord. He never neglects it.
In this and in other assembly meetings the subjects are more general, but we can count on the Lord’s presence and the guidance and ministry of the Holy Spirit.

The Coming and Reign of Our Lord Jesus Christ: The Apostasy, Part 7

THE APOSTASY
In league with this first beast will be a second, viewed in Revelation 13, as having two horns like a lamb, but speaking as a dragon. This is antichrist, the false messiah, a mimicry of the true Lamb of God, combining, in his own person, power in the land of Israel (being accepted as their prince by the mass of Jews), the head of Jewish apostasy (Rev. 13:11-18), and also exercising all the power of the first beast before him. Arch-leader too of the apostasy of Christendom, he denies the Father and the Son (1 John 2:22). Men in that day will be so deluded that they will worship the dragon, who gives the beast his power; also the beast, saying, “Who is like unto the beast?”; and also the antichrist, who sits in the temple of God, and shows that he is God (2 Thess. 2:4).
In addition to this, the wicked one leads men to make an image to the beast; and he has “power to give life (or breath) unto the image of the beast, that the image of the beast should both speak, and cause that as many as would not worship the image of the beast should be killed. And he causeth all, both small and great, rich and poor, free and bond, to receive a mark in their right hand, or in their foreheads; and that no man might buy or sell, save he that had the mark, or the name of the beast, or the number of his name” (Rev. 13:14-17). Some will be so bold-faced in their wickedness that they receive the mark in their foreheads, willingly and openly owning the beast; others who receive it in their right hand, which would be more secret, can show on whose side they are if necessary. Those who refuse the mark and will not bow down to and worship the image of the beast are prohibited from buying or selling, and must in consequence pass through terrible sufferings, many being martyred.
There will be a terrible hour of temptation, not only for Judah, but for the whole world (Rev. 3:10), to try them that dwell on the earth. These latter are a special class, mentioned many times in the Revelation, termed variously in our translation “inhabiters of,” or “dwellers on the earth,” a class whose hearts are fully set on the earth and earthly things, to the exclusion of the claims of God.
Towards the closing hour of this awful scene that we have been tracing, the evil system that is viewed in the seventeenth and eighteenth chapters of Revelation as a woman, and also as a city, whose name is “Mystery, Babylon the great, the mother of harlots, and abominations of the earth,” receives the cup of judgment from God. This represents ecclesiastical corruption— Rome, the mother and center, enriched by unholy traffic and intimacy with the worldly powers. The beast and ten horns or kings, that is, the civil power, after supporting her for a time (for she is seen riding upon the beast), at last hate her, and are the instruments in God’s hand of her judgment (Rev. 17:3,16-18). Kings, merchants, shipmasters, traders, sailors, bewail her fall (Rev. 18).
In comparing Revelation 17:1 with Revelation 21:9, we find the prophet in the former passage invited to come and see her judgment, and he is carried in the spirit into the wilderness; in the latter passage to come and see the bride, the Lamb’s wife, and he is carried in the spirit to a great and high mountain. There is shown to him the Holy Jerusalem descending out of heaven from God, having the glory of God.
(Continued and to be Continued).

Correspondence: Explanation of Ex. 33:11, 20; Acts 1:11

Question 44: Please explain Exodus 33:11: “And the Lord spake unto Moses face to face, as a man speaketh unto his friend”; also Exodus 33:20: “And He said, thou canst not see My face: for there shall no man see Me, and live.” A. G. C.
Answer: Exodus 33:11, tells us the intimate way the Lord chose to speak to Moses. (See also Num. 12:8; Deut. 34:10). To him alone did He speak face to face, mouth to mouth, and His similitude did he behold; that is, part of Jehovah’s character was revealed to him; but 2 Chronicles 6:1 tells us that He dwells in thick darkness.
Moses had been on the mount with Jehovah, and when he came down with the tables of stone in his hands, the people were worshiping a golden calf. Moses had pled for them and Jehovah repented (Ex. 32:11-14), and through all this Moses learned what suited Jehovah. He saw the calf and the dancing, and cast the tables down and broke them, and this saved Israel from destruction. Moses further takes the sword and calls on all who were on Jehovah’s side to take vengeance on the idolatry, and then he goes up to offer himself in atonement, but this act waited for a greater than Moses. Jesus, the Son of God, could alone accomplish this. Jehovah hears Moses’ pleadings, and spares the people, but smote some of them. Jehovah says, I will send an angel. I will not go up with you, for this people are stiff-necked. Moses answers, Jehovah must go with him, because they are stiff-necked. Jehovah called them, “thy people.” Moses says, “Consider that this nation is Thy people.” What intimacy is seen in all this, yet when Moses says, “Show me Thy glory,” He shows him but a part, putting him in the cleft of the rock, and covering him with His hand, so that in safety he could behold part of Jehovah’s character, for He said, “Thou canst not see My face; for no man can see My face and live.”
Exodus 34:5-7, is the declaration: “God, merciful and gracious, slow to anger, and abundant in loving kindness and truth; keeping mercy for thousands, forgiving iniquity, transgression and sin, and that will by no means clear the guilty.”
God could not reveal Himself fully, till the Lord Jesus came, and atonement was wrought. “How am I straitened till it be accomplished.” Luke 11:50, tells of love that was pent up till God’s righteousness was declared (Rom. 3:25-26). Now God is revealed. “The only begotten Son who is in the bosom of the Father, He hath declared Him” (John 1:18). “God is light” and “God is love” (1 John 1:5; 4:8). And we are children of light, and our home is in His presence. No longer do we need to put our shoes from off our feet, but clothed and fitted for His presence we joy in God as His dear children.
The absolute deity of God, all that He is, no creature can see or know, to whom be honor and power eternal. Amen. The finite cannot comprehend the Infinite. We know God in those blessed ways revealed to us as Father, Son and Holy Ghost. Praise His most blessed name!
Question 45: What does “ALL ISRAEL shall be saved” mean? (Rom. 11:26). N. C.
Answer: It means that Israel as a nation will be saved. It does not mean that every Israelite will be saved. All of them, as well as all Gentiles, who have died without Christ, are lost for eternity, for they have died in their sins. Israel is to be gathered into Palestine again, and all the ungodly ones will be sifted out from them and the rest will have the national place and be twelve tribes again (Ezek. 38; Rev. 7; Heb. 8:8).
Question 46: Does Acts 1:11 refer to Christ’s coming for us or to His appearing to Israel? H. G.
Answer: The disciples here are still the remnant of Israel, looking for the setting up of the kingdom.
When the Lord comes for us, who are His heavenly people, we will hear His shout, and will be caught up in a moment to meet Him in the air (1 Thess. 4:16, 17).
When He comes to Israel as King, we will come with Him, and every eye shall see Him. He shall so come in like manner as they saw Him go; this is His appearing.
But the precious truth expressed here is sweet to each saved one—that it is “this same Jesus,” the Lord “Himself,” whom we have known as our Saviour, will come for us. He will not send an angel for us.
Question 47: Is Melchizedek the Lord?
Answer: No, I do not at all think he was the Lord. He was for a purpose made like unto the Son of God; and that he might be like Him, no record of his birth or death is given, and it is to this Hebrews 7:3 refers. We know nothing of his pedigree, though Hebrews 7:6 lets us know he had one; at least, so I have taken it.
In the passage in Genesis he is brought before us as “King of Salem,” and “Priest of the most high God,” but of his beginning and end, of his father and mother, we know nothing, though in Genesis we naturally look for this. This is in order, as just said that he might be the type of the Lord. I have never thought he was actually without father and mother, and beginning of days nor end of life. If he was the Lord, then it would be His typifying Himself in a future day.

Inspiration of the Scriptures: The Silent Interval of Four Hundred Years

THE SILENT INTERVAL OF FOUR HUNDRED YEARS.
The silence of the prophetic testimony between the close of the Old Testament and the commencement of the New, which continued over four hundred years, is very significant. Before that, all through the course of God’s ancient people, notwithstanding their many sins and departures from Jehovah, sacred history faithfully gave its inspired record of them, until Malachi closed it with the saddest denunciations of their ways. And what makes this long and silent gap so remarkable, is, that the final testimony of the prophet links itself with the beginning of Luke’s Gospel in foretelling the coming of John, the forerunner of our Lord. He said, “Behold, I will send My messenger, and he shall prepare the way before Me” (Mal. 3:1; Luke 1:13-17; Mark 1:2-3). Our Lord also associated the prophets with the Baptist— “The law and the prophets were until John.” These links certainly unite the Old and New Testaments in a very remarkable way. May we learn the lessons they are intended to teach us!
Very solemn indeed is Jehovah’s silence for four centuries in the history of His favored people; especially when we consider the abounding evil which they so long pursued, as recorded by their inspired writers, notwithstanding the goodness and patience God had exercised toward them. Their sad state weighed heavily on the spirit of the prophet Malachi. He began his message to them by saying, “The burden of the word of Jehovah to Israel by Malachi,” and ended it by alluding to “the great and dreadful day of Jehovah.” True, he added, that Elijah would be sent before that, to “turn the heart of the fathers to the children, and the heart of the children to their fathers, lest I come and smite the earth with a curse” (Mal. 1:1; 4:5-6). In this solemn way the Old Testament writings were closed; and we have no divinely given annals of the Hebrew people during the following four hundred years. May we ponder its grave significance! Was it because of their many sins that divine communications and miraculous intervention ceased? Was it on this account that prophetic inspiration was discontinued, and they were left to their own devices? Of one thing we may be well assured, that, under the circumstances, inspiration was suspended in God’s wisdom and faithfulness.
On looking into Luke’s Gospel, which is so blessedly linked, as we have seen, with the last of Israel’s prophets, we find toward the close of this gap, some turning to Jehovah and His Word. Zacharias and Elizabeth were spoken of as “both righteous before God, walking in all the commandments and ordinances of the Lord blameless” (Luke 1:6). Others, too, were, through grace, acted on by the Holy Spirit, and therefore turned to the Word of Jehovah; and were so bowed by it, that they “looked for redemption in Jerusalem,” and served Him day and night with fasting and prayers.
In the Jewish people, and Zacharias, who was a true descendant of Aaron, and whose lot was to burn incense when he went into the temple of the Lord, we find a further recognition of Jehovah in turning to Him according to His Word. We are told, that “the whole multitude of the people were praying without at the time of incense” (Luke 1:9-10). And there appeared unto Zacharias an angel of the Lord with cheering and encouraging words as to John, as afterward he came to Mary concerning the Messiah. These facts are very significant, and accordingly the forerunner was, ere long, to be born into the world— the prophet of the Highest, and to be soon followed by the birth of Messiah, the Son of the Highest. Jesus was the true Shepherd of the sheep, and to Him the porter opened; for holy men and women, taught by the Holy Spirit, and taken up with Jehovah and His interests, were, by their rejoicing and cordial welcome, like the porter opening the door. If angels heralded His coming into the world with, “Glory to God in the highest, and on earth peace, good will toward men,” Simeon could take the babe into his arms, and with a grateful and worshiping heart say, “Lord, now lettest Thou Thy servant depart in peace, according to Thy word, for mine eyes have seen Thy salvation.” Anna also found her way into the temple at that moment, and “gave thanks likewise unto the Lord, and spake of Him to all them that looked for redemption in Jerusalem” (Luke 2:14,29-30,38)
Divine communications which had been suspended for such a lengthened period as four hundred years, could now be resumed according to the wisdom, and power, and goodness of God; but, as we afterward find in the New Testament Scriptures, not according to the kind of ministry of olden prophets; though by the same Spirit, but according to the grace, and gifts, and qualifications they received of the Lord, who fitted and furnished them for the service to which they were called as His friends and servants.
(Continued)

The Mighty Hand of God

Did you ever ponder that expression, “The mighty hand of God,” and did you ever take time to consider its meaning?
Let us look at, two or three scriptures which tell us something of what that mighty hand has done. In Psalm 102:25, we find these words, “Of old Thou hast laid the foundation of the earth; and the heavens are the work of Thy hands.” What a wonderful work! how truly the work of a mighty hand. Again in Psalm 19:1, “The heavens declare the glory of God and the firmament showeth His handy work.” And once more, “The heaven is My throne, and the earth is My footstool.... All those things hath Mine hand made,.... saith the Lord” (Isa. 66:1-2). Surely when we look around us at all the wonders of the heavens, the sun, the moon, the innumerable stars — each one a world in itself — and then at all the wonders of the earth, in its animal, in its vegetable, and in its mineral kingdoms, we are ready to exclaim, “The hand must indeed be mighty which has made all this.”
But has that mighty hand only created all these marvelous things and then left them? Ah no! He supports and cares for the works of His hands, as we read in Psalm 145:16, “Thou openest Thine hand, and satisfiest the desire of every living thing.” Not one creature, however small or despised, is forgotten by Him, And will He forget the youngest and the weakest of His children?
How much more that hand has done for us, than for any other creature; turn once more to the Scripture, and read in Zechariah 13:6, “And we shall say unto Him, what are these wounds in Thine hands?” Can you not answer this question and say, “He was wounded for my transgression”? How it surpasses all our thoughts, that mighty One, the Creator of heaven and earth, humbling Himself, and becoming obedient unto death, allowing one of the creatures His hand had made, to wound Him, to nail Him to a cross, and all that He might bear the punishment of their sin, yea of our sin.
And in resurrection, we still see Him the same loving, tender, Saviour; we hear Him saying to Thomas, “Reach hither thy finger and behold My hands” (John 20:21) and again, those hands take bread and fish, and give them to the weary disciples, after their night of fruitless labor on the sea; and the last sight they had of Him, was with hands uplifted to bless them, as “He was parted from them, and was carried up into heaven” (Luke 24:51). And is that mighty hand still in exercise for us? Yes, it is holding us day and night; and we have our Lord’s own word that, “Neither shall any man pluck them out of My hand” (John 10:28). And who is it that He is holding so safely and so surely in His mighty hand? His sheep; those who have heard His voice and have come to Him for salvation and protection.
Before closing this little paper, shall we go back to the verse we began with? We shall find it in 1 Peter 5:6, “Humble yourselves, therefore, under the mighty hand of God.” Does it not humble us to think of His might, and does it not humble us to think of His love? But what is the next verse? “Casting all your care upon Him, for He careth for you.” Did you ever care for a little helpless child, or even a little bird, or a little puppy or a kitten? Then you have some faint idea of what caring for another means. It does not mean loving from a long distance; no, it means daily, hourly, almost constant acts of thoughtful love.
“Do you not often forget about your baby, when you are attending her?” a friend once asked, and the young mother replied in surprise, “Why no, she is always in my mind.” And if that is the way we care for what we love, is not our Lord’s care for His loved ones far better? Can we not safely trust Him with everything that troubles or distresses us, knowing that the hand that cares for us is “mighty.”

Scripture Study: Matthew 7

Matthew 7:1-5. The disciples were to beware of allowing in themselves a fault finding spirit. To imagine evil or to judge motives is forbidden. If any one does, he gives others occasion to blame him. He sees a mote in his brother’s eye, but does not see the beam in his own eye. Likely, if the beam was out of his own eye, he would see that the mote was out of his brother’s eye. Perhaps he imagined it.
Love hopes all things and believes all things. Hypocrisy is spoken of here in its full character, but we are all in danger from it.
But Christians are to judge evil, in order to have true fellowship. 1 Corinthians 5:12-13, instructs us, as an assembly, to refuse evil when it would come in, and to put away wickedness if it has come in. To exercise scriptural discipline, needs godly care, and long-suffering grace, and dependence on the Lord to act for Him. To allow unrighteousness, fundamentally wrong doctrine, or schism to go on unreduced, hinders fellowship in the truth.
2 Timothy 2:19; 2 Thessalonians 3:6,14, teach us to judge evil individually.
Matthew 7:6. It is right to preach the gospel to every one, in love, meekness and faithfulness, but arguments and loud discussions are worse than fruitless, and to discuss the precious treasures of the riches of Christ with dogs that have no conscience, but bite and devour; or with swine— creatures of no discernment, they cannot chew the cud. If you do, you waste your time, grieve your heart, and the precious truth is degraded. The gospel alone suits the unconverted. There “is a time to keep silence, and a time to speak.”
Matthew 7:7-12. This is to teach confidence in the goodness of God, the Father; giving perseverance in desire, in endeavor and importunity, and this reality obtains the blessing. The Father’s goodness is to be counted on as a parent for his child’s good. Verse 12, puts them in the giver’s place, and in treating others as we would be treated, the Word is fulfilled in us (Rom. 8:4; James 2:8).
Matthew 7:13-14. It needs energy to enter the kingdom—"strive.” It is easy to enter the wide gate and to walk the broad road, but alas! it leads to destruction. The pleasures of sin are only temporal; the pleasures of Christ are for evermore—it is worthwhile overcoming difficulties to possess the treasure. What inward satisfaction is formed in the “strait gate,” and the “narrow way.”
Matthew 7:15-20 puts them on their guard against false teachers, for their teaching produces bad fruit; and what they teach, leads away from Christ, rather than to Him, and produces coldness, indifference, and other evils (Acts 20:28-30; Rom. 16:17-18; 1 John 4:1). We are not called upon in this portion to judge if a man is saved or not. That remains with the Lord who knows them that are His (2 Tim. 2:19).
Matthew 7:21-23 is profession without possession of eternal life. Those who are real, seek to do the Father’s will, for there may be close imitation and display of power, signs, miracles, yet be workers of iniquity. Judas Iscariot, who had a devil (John 6:70), was sent with the rest to work miracles.
Matthew 7:24-27. Practical obedience is true wisdom. The wise man builds on the Word of God; the foolish heareth but doeth not; then dreadful ruin comes upon them, every one will be tested. We need to dig deep to get down to the rock (Luke 6:48).
Matthew 7:28-29. The Lord’s teaching was with authority; it was the Word of God. No wonder the multitudes were astonished. The scribes gave their opinions.

Love Your Enemies

Everybody in the store could see what a marvelous change had come upon young B. He was evidently a different man. Formerly, he was first in evil ways and rough speech; now, all was changed. Not only he no longer joined in the lightness of his companions, but he would reprove them, at the same time confessing with simplicity, that, by grace, he had been saved through believing in Jesus, and urged upon them the necessity of being born again. From that time, all his fellow-workmen turned against him, and with one mind, tried by all possible means to provoke him to anger, and to prove that he had not changed.
One evening, when he was about to go to a prayer meeting, his hat was not in its accustomed place. It had been hidden. Without saying a word, he went quietly to the meeting bare headed.
Another time, it was his coat that was missing. But this did not stop him, and as he had no time to go home for another, he went in his shirt sleeves.
But something worse was yet to come. On a hot summer day, one of the apprentices placed a piece of tar in B’s hat. Without noticing it, the latter who was late, put on his hat hurriedly and ran home for dinner. The heat had melted the tar, and when B. took off his hat, he found that it had adhered strongly to his forehead. The effort he made to pull it away, tore off the skin leaving a very painful wound.
B.’s father, at the sight, became very angry and threatened to complain. “O, father!” answered the young man, “You have been the Lord’s so many years. I am surprised to hear you speak so!”
His mother wept to see her son so exposed to the malice of his companions, but B. answered her by the words of the Apostle, “Christ, when He was reviled, reviled not again, when He suffered, He threatened not” (1 Peter 2:23).
On his return to the store, his companions surrounded him. “What has happened to you? Who has done that?”
“God knows all about it,” he answered; “and I know what I shall do to the culprit!” A short while after, the guilty apprentice came to him, and said, “What will you do to the one who has played you this trick? O, B! I am sorry, but please do not tell the master, he would dismiss me. You know, I am an orphan, what should I do?”
After a moment’s silence B. answered, “Now, my boy, I am going to tell you what I will do.”
“What?” anxiously inquired the boy.
“I will forgive you.”
That is the way this young Christian sought to walk in the foot steps of his Lord, remembering that after having been reviled, mocked, scourged, and finally crucified, He said: “Father, forgive them, for they know not what they do” (Luke 23:34). He knew that his divine Master had taught His own how to act toward those who ill treat them, “Love your enemies, bless those that curse you, do good to them that hate you, and pray for them which despitefully use you and persecute you” (Matt. 5:44).
But how can we become conformed to Jesus? By looking to Him, by having our thoughts and our hearts occupied with Him. We see this in Stephen, the first martyr. While ill treated, and stoned, he saw Jesus at the right hand of God, and in the spirit of his Lord, he could pray for his murderers, “Lord, lay not this sin to their charge.”
Dear young Christians, think upon Him, be occupied with Him, and the Holy Spirit will enable you, in all your ways to walk, even as He walked” (1 John 2:6).

Truths for Young Christians: Selfishness

SELFISHNESS
In our previous papers we have considered various simple doctrinal subjects of great interest and value to the young believer, connected with his standing before God, his place in this world, his future hopes, in short, “his portion, his path, and his prospect.” In the present series we propose, with God’s help, to take up some of the special dangers or besetting sins to which young Christians (and old ones as well) are liable, and respecting which it may be helpful to see what Scripture has to say. “To be forewarned is to be forearmed,” and these papers are written in the earnest hope and prayer that they may be practically used in pointing out and guarding some against those sins and failings which so often ruin a walk otherwise consistent, and bring reproach upon the name of Christ. It is by our actions in small matters that the world judges us; not by the amount of our knowledge of Scriptural principles, but by our application of them in daily life.
Selfishness Is Anti-Christian
Let us, then, now briefly consider this emphatically anti-Christian sin of selfishness. We call it anti-Christian, because it is expressly recorded of Christ our Lord, that He “pleased not Himself” (Rom. 15:3). This strikes at once at the root of the matter, for when we read 1 John 2:6, that we ought to walk as Christ, and remember these are the words of God, and then turn to the Scripture just quoted, we must at once see that all selfishness is truly anti-Christian. If, however, example is not enough, we have precept as well. “Let no man seek his own, but every man another’s wealth (or good)” (1 Cor. 10:24). “Look not every man on his own things, but, every man also on the things of others” (Phil. 2:4). Most touching of all, perhaps, to the heart that has tasted the love of Christ; to whom He is precious is 2 Corinthians 5:14-15, “For the love of Christ constraineth us... He died for all, that they which live should no longer live unto themselves, but unto Him who for their sakes, died and rose again” (R. V.).
A Sign of the Last Times
Selfishness is shown in many and various ways: As one of the signs of the last days, it is said, “Men shall be lovers of their own selves,” or in other words, “selfish” (2 Tim. 3:2). This is the root from which every variety springs. The selfish man seeks his own things, not the things which are others, still less those that are Jesus Christ’s (Phil. 2:21), as the Apostle so touchingly writes to the Philippians, complaining that this sin was a great and crying evil in his day.
It is found everywhere, even amongst believers, although it is a vice so repulsive in its nature, that the man of the world outvies the Christian in despising it when shown in certain grosser ways; the latter only, however, can know what it is to be truly unselfish in spirit in all things. How ashamed we feel when we consider how often our best actions are blighted by the foul spot of selfishness.
Self the Object
“Pleasing ourselves,” directly condemned in Romans 15:1, is a common form of “seeking our own.” It is seen in great and little things; in our choice of work for the Lord, in our choice of residence, of companions, of dress, of occupation, and in many petty ways in which we daily indulge, instead of denying ourselves. O, how ashamed we feel when we just sit awhile and think of our dreadful “self-pleasing” in little things; always looking out for “number one.” So contrary are we in spirit to our beloved Lord.
Seeking Our Own
Another phase of “seeking our own” (Phil. 2:21), is in eagerly pursuing some worldly advantage, being unscrupulous in money-making, or keen in money saving. All this becomes much worse, terribly worse, if in any way hypocrisy comes in to aid our selfishness. Is it not fearful to think how the name of Christ is despised by men of the world through those who should be “His epistle,” who thus seek their own? For the worldly man well knows that Christians should be unselfish, though all the time they may be selfish enough themselves. Paul was not like this; “Not seeking my own profit” (1 Cor. 10:33). This line of conduct is powerfully described in Isaiah 56:11, “They all look to their own way, every one for his gain.” Surely it is a sign of the last times when one professing Christian is heard urging another to “raise himself in the world by pushing others down.” It may be there are not many bold enough to give such fearfully unchristian advice, But are there not hundreds who in the main practically follow it? Another form is seeking precedence of others. Such selfishness was displayed in Matthew 20:20, and gently rebuked by Christ. It is often seen, alas! in spiritual as well as worldly matters, and many have been the bitter parties or factions that have been developed from this form of selfishness. Let us judge ourselves as to this, seeking neither the chief seats in synagogues, nor the greetings in the markets.
Not Caring for Others
Neglecting the poor (1 John 3:17) is a flagrant form of selfishness strongly condemned by the Word. Often it is unintentional, and arises simply from a habit of considering ourselves instead of others. In some cases selfishness may give, to get rid of annoyance, but it can never give with true sympathy. That rare and tender plant of Christian growth cannot grow in the same atmosphere as “self.” “Finally, be all of one mind, sympathizing, full of brotherly love, tenderhearted, humble minded” (1 Peter 3:8, JND). In many cases we do not mean to be selfish, but by being careless in following Christ and having naturally ourselves instead of Him as our object, this vice shows itself in little ways in almost all we do. We trust that to many of our readers a word will be enough to point out this un-Christlike action, which perhaps unknown to themselves has been undermining their Christian life and taking away from the power of their words to others. If we look at one of the characteristics of “love” in 1 Corinthians 13:5, “Seeketh not her own,” and then turn and quietly look at our own lives by the side of it, the light of the Word like a sunbeam in a dusty room throws out into strong relief all the “little foxes” of small petty selfish deeds that have so spoiled “the tender grapes” of our spiritual life.
The Remedy
What, then, is the remedy for selfishness? One might answer, “To think of others,” as in the parable of the certain Samaritan. This is a good and Christian habit—to find a neighbor in every one whom I can serve and to love him as myself. It is most important to acquire a habit of thinking of the comfort, convenience and wishes of others on all occasions, and seeking to please my neighbor for his good unto edification at all times, but there is a more excellent way yet, and that is for Christ to become the center of my thoughts instead of myself, so that all my actions naturally have reference to Him. In this way, I not only become truly unselfish, but I become (not only negatively but) positively like Christ.
Dear fellow believer, this is the sort of Christianity which is understood amongst men, and brings true glory to God. When a man gives up voluntarily the best place, to which he has an undoubted right, when he foregoes his own advantage, and to his own loss goes out of his way to show kindness to others, when he becomes poor, and not merely gives of his abundance, for the sake of Christ’s people whose needs he provides for, and when he not only spends, but is spent for others, then indeed does he become an epistle of Christ, known and read of all men. None can pass a man unobserved in whom the brand of selfishness has been obliterated by the fresh brand of Christ (Gal. 6:17).
O, may His love constrain us thus to live to His glory!

Christ the Center: Or, Why Christians Should Meet in His Name Alone, Part 1

OR WHY CHRISTIANS SHOULD MEET IN HIS NAME ALONE.
The question presents itself to the Christian who seeks to walk with the Lord, “Where amidst all the Babel of profession do the Scriptures place me as a Christian? How and with whom should I meet to worship my blessed Saviour?” It is as an attempt to answer these all-important questions, that the following considerations are affectionately presented to all the beloved children of God. That Christ is the only divine center of worship for the Christian—that His is alone the name in which Christians should be gathered— is the teaching of the Holy Spirit in the Scriptures.
First. The worthiness of Christ! It is God who “hath highly exalted Him, and given Him a name which is above every name: that at the name of Jesus every knee should bow... and that every tongue should confess that Jesus Christ is Lord, to the glory of God the Father” (Phil. 2:9-11). Thus hath our blessed God and Father delighted to honor Him, who “is the head of the body, the church: who is the beginning, the firstborn from the dead; that in all things He might have the pre-eminence” (Col. 1:18). In this name, so precious to every believer, did all Christians meet in the days of the apostles; and when the veil of the future was drawn aside, what did John the servant of Jesus Christ behold? When he saw Jesus Christ he says, “His countenance was as the sun shineth in his strength. And when I saw Him, I fell at His feet as dead. And He laid His right hand upon me, saying unto me, Fear not; I am the first and the last” (Rev. 1:16-17).
“A door was opened in heaven.” What a sight! The vision of the future glory of the Lamb in the midst of the millions and millions of the redeemed! A Lamb as it had been slain.
“And they sung a new song.” What will it be to be there; to hear that swell of joy unspeakable — to join that song? Not one redeemed to God by His blood will refuse to sing, “Thou art worthy.” Angelic hosts cry with a loud voice, “Worthy is the Lamb that was slain to receive power, and riches... and honor, and glory, and blessing:” yea, all redeemed creation shall be heard saying, “Blessing, and honor, and glory, and power, unto Him that sitteth upon the throne, and unto the Lamb forever and ever” (Rev. 5:6-14).
Thus shall our adorable Lord be adored, and owned in heaven and throughout all creation. This is God’s estimate of the risen Christ, who once died for our sins the Just for the unjust, to bring us to God. And thus shall God’s will be done in heaven. Should an anxious, troubled soul read these lines, mark well that this is the redemption-glory of Christ. And who were those worshiping millions, redeemed by His blood? Dying thieves, Mary Magdalenes, sinners of the city. And is Jesus worthy of bringing such to glory? Yes, the most holy, holy, holy God says He is worthy! and all creation shout Amen. O, do you, my reader, now give God credit? Such is the worthiness of this risen Jesus, that God says, “Be it known unto you... that through this Man is preached unto you the forgiveness of sins; and by Him all that believe are justified from all things, from which ye could not be justified by the law of Moses” (Acts 13:38-39). Thus salvation is wholly through Christ. Blessed are they who can say, “We have redemption through His blood, the forgiveness of sins.”
I do not presume to be able to set forth, by pen or tongue, the glorious pre-eminence of Christ. I point to the Scriptures that so clearly declare the worthiness of Christ. But many who read this paper will say, “What true Christian doubts for a moment the worthiness of Christ, or the greatness of His exalted name?” True, true, there is a chord in every Christian’s heart that responds to the name of Jesus. But the question is How much, or how great, is that worthiness? There may be one thousand Christians in a town, or ten thousand in a city. I mean such as really have redemption through the blood of Christ, whose sins are forgiven. Now if Jesus be worthy of the united praise and worship of all creation, if all the millions of the redeemed in heaven shall gather around His adorable Person, then is He not worthy of the united worship of one thousand in a town, and ten thousand in a city on earth? Surely in heaven every name and sect must fall. And why not on earth?
It is a great mistake to suppose, then, that Christians should separate from every name and sect because they think themselves better than the dear children of God in those sects; far be the thought; no! it is because Jesus is worthy—yes, worthy the sacrifice of at once giving up every name and sect, and of gathering to His blessed name and Person alone. Yes my fellow believer, He is worthy that you, whoever you are, and to whatever sect you belong, He is worthy that you should own no other name but His. What must angels think, knowing and delighting as they do in the exalted name of Jesus; when they see our ways on earth? The divisions on earth must present a dark contrast to the unity of heaven. In many places all God’s redeemed people may be seen bearing various names; and not even two, or three, meet in the whole town in the name of Jesus alone. And yet, most assuredly, Jesus is worthy that every believer in the place should meet only in His name.
Now, if God’s will is so plainly done in heaven by all gathering to the Person of the Lamb, how can I pray, “Thy will be done in earth, as it is in heaven,” unless I am prepared to give up every name and sect on earth as it is done in heaven? Would it not be more consistent to say — I have been in such a sect, and all my friends are there; excuse me therefore from doing Thy will on earth, as I shall do it, and as it is done, in heaven? Is it narrowness to do the will of God on earth as it is done in heaven? Is it too much to own the Lordship of Christ, to the glory of God the Father, and to own no other but Christ? God sets the highest value on the name of Jesus. Man says it is no matter what name you bear.
(To be Continued).

The Coming and Reign of Our Lord Jesus Christ: The Apostasy, Part 8

So too, dear reader, if you would escape from the terrible vortex of religious corruption that surrounds us at the present day, and which will be part of Babylon in its last phase, you must be in spirit in the wilderness. In separation from it, in the presence of Christ, there alone can you see it in its true colors. And if you would enter into God’s thoughts about the church, and discern its distinctive heavenly character, you must in spirit enter where Christ is, and accept by faith the high and exalted position which is the Christian’s true portion.
The evil religious power being crushed, the beast and the kings of the earth and their armies are gathered together against the Lord, and He judges them also; the beast, and the false prophet with him, being cast into the lake of fire (Rev. 19:11-21). Details of this we will reserve till we dwell on His coming in power and great glory to judge and reign.
The Assyrian who attacks Jerusalem from without, and takes it (Zech. 14:2), overruns with his armies the land of Israel, as well as many other countries; and after further warlike exploits, himself finally falls as he is about to attack the city a second time; but we shall see more of this also further on (Isa. 10:24-34; 29:1-8).
Two-thirds of the Jews are cut off in this great tribulation; but one-third, preserved by God, are brought through the fire (Zech. 13:8-9). Some are martyred for their testimony, and for refusing the mark of the beast, and have part in the first resurrection; one-third bear testimony, and are brought through by the power of God into millennial blessing; the remainder, the apostate, idolatrous Jews, who receive the mark, and think they are safe, are cut off in their wickedness, either by the awful invasion of the Assyrian, who is the rod of God’s anger, sent by Him against the hypocritical nation (Isa. 10:5-6), or by other summary judgments of God.
This fearful hour was referred to by the Lord when He spoke of the unclean spirit who saith, “I will return unto my house whence I came out. And when he cometh, he findeth it swept and garnished. Then goeth he, and taketh to him seven other spirits more wicked than himself; and they enter in, and dwell there: and the last state of that man is worse than the first” (Luke 11:24-26). Terrible picture of Israel’s condition under antichrist.
But judgment will not be limited to the land, but go out far and wide in the earth. The God-fearing Jews who bear witness for Jesus, publishing the glad tidings of the kingdom among the nations, will be special objects of persecution wherever they are found. “Take heed,” said the Lord, “to yourselves: for they shall deliver you up to counsels; and in the synagogues ye shall be beaten: and ye shall be brought before rulers and kings for My sake, for a testimony against them” (Mark 13:9). The early Christians received similar treatment; but the fulfillment of this passage is yet future. So terrible will be the hatred, even in their own families, that “brother shall betray the brother to death, and the father the son; and children shall rise up against their parents, and shall cause them to be put to death. And ye shall be hated of all men for My name’s sake: but he that shall endure unto the end, the same shall be saved” (Mark 13:12-13).
Thus will be fulfilled in the most special and literal manner the words of our Lord, “A man’s foes shall be they of his own household.” Men generally will show their persistent hatred of the name of Christ by persecuting those who publish the glad tidings of His coming kingdom and glory. But he that endures to the end, that is, to the end of the hour of tribulation, shall be saved— saved out of it for blessing in the millennial earth (Jer. 30:7). “In those days shall be affliction, such as was not from the beginning of the creation which God created unto this time, neither shall be” (Mark 13:19). “And except that the Lord had shortened those days, no flesh should be saved: but for the elect’s sake, whom He hath chosen, He hath shortened the days” (Mark 13:20). “This generation” (or race), said the Lord to His Jewish disciples, “shall not pass, till all these things be fulfilled. Heaven and earth shall pass away, but My words shall not pass away” (Matt. 24:34-35).
How blessed for the Christian to know that he is delivered from this terrible hour! that as the Lord delivered Lot before judging Sodom, and translated Enoch before drowning the world with a flood, so will He bring His own loved ones of this present interval of grace to the glory above, before He judges the habitable earth for its wickedness, sweeping it with the besom of destruction.
How false the security of the poor world, deluding itself, and deluded by Satan, that things are getting better! Many cry, “Peace, peace, and there is no peace.” How vain to think that God’s grace to a lost world will lead to its conversion! Nay; but the Word of the Lord is yet to be fulfilled: “When Thy judgments are in the earth, the inhabitants of the world will learn righteousness” (Isa. 26:9). But O, how blessed to know that when the consumption decreed shall have had its course, it shall overflow with righteousness (Isa. 10:22), and the despised and rejected Nazarene, Jesus, the Son of God, will have His rights and title in the earth!
Could we stay where death is hovering?
Could we rest on such a shore?
No; the awful truth discovering,
We could linger there no more;
We forsake it,
Leaving all we loved before.

Correspondence: Burdens in Gal. 6:2, 5; Luke 11:24-26; Matt. 16:17

Question 48: What is the difference of the burdens in Galatians 6:2,5? A. M. C.
Answer: In the 2nd verse it is helping others to bear their trials. This is the law of Christ, this is what He did when here on earth. We can do it by prayer for them and sympathy with them, and relieve them when we are able. Verse 5 means that each of us is responsible before God for our own work. The false teachers thought of themselves instead of serving Christ. (See Gal. 6:3-4). This will come out at the judgment seat of Christ.
Question 49: Please explain Luke 11:24-26? J. E. K.
Answer: In Matthew 12:43-45, We have the same truth applied to the Jews as a nation, “Even so shall it be also unto this wicked generation.”
In Luke it is applied generally. The Jews, since Ezra’s time, had been reformed, so that in the days of our Lord on earth, they, as a house, were empty, swept and garnished. So a man may be reformed, give up much outward evil, and may be proud of the change. The Jews did not receive Christ, and remained short of having Him as the One to fill them. So a man must he born again, must receive Christ as his Saviour, and have the Holy Spirit dwelling in him, or his reformation will be of no avail. In the time of tribulation, the Jews will go back to idolatry, and their last state will be worse than their first. So a man that is only reformed, is taken possession of by Satan, as his house. “I will return unto my house, whence I came out,” and his last state is worse than his first. (Compare 2 Peter 2:20-22.) Man must have an object, and if he has not Christ, he will have self in one form or another.
A profession without reality avails nothing. We are to hear the Word of God and to keep it (Luke 11:28). Obedience is one of the characteristics of eternal life.
Question 50: Explain Matthew 16:17, especially “The gates of hell shall not prevail against it.” H. E. B.
Answer: Peter’s confession: “Thou art the Christ, the Son of the Living God,” is the occasion for the Lord to reveal what He was going to do. Verse 17 tells us, this revelation of His person to Peter did not come from flesh and blood, but from His Father which is in heaven. “And I also say unto thee, that thou art Peter (a stone), and upon this rock (Christ, the Son of the living God), I will build My church (assembly), and the gates of hell (hades), (meaning the unseen world), shall not prevail against it” (Matt. 16:18).
This is the first time in Scripture we find the church mentioned. Peter was a stone to be built into it (1 Peter 2:5). Christ is the rock on which it is built. He is also the Builder. I will build, shows it was yet future.
At Pentecost, when the Holy Spirit came down to dwell on earth, this church, or assembly was formed, “a spiritual house,” of which every believer is a living stone. Hades is the unseen world or state of the dead. Satan, through sin, has the power of death; but against this church, Satan has no power; the gates of hades, the power of the enemy, shall not prevail against it; for it is built on Christ, the Son of the living God. He is the living One and has the keys of death and of hades (Rev. 1). His is a life that cannot be overcome or destroyed, and what is founded upon this rock of the unchangeable power of life in the Son of the living God shall not be overthrown by the kingdom of death. Satan can go no further than God allows him to go. How blessed and secure are all that belong to Him in all ages! (Num. 23:23; Isa. 54:17).

Inspiration of the Scriptures: The New Testament, Part 1

THE NEW TESTAMENT.
In approaching our consideration of the New Testament, it is well to premise that there are two common errors in the present day, and found almost everywhere in Christendom:
1. That the use of the Bible is only to teach persons the way of salvation.
2. That the book of Revelation is too difficult for any one to understand.
As to the first point, it is plainly said that the Scriptures are not only able to make “wise unto salvation through faith which, is in Christ Jesus,” but that every Scripture is God-breathed, or “given by inspiration of God, and is profitable for doctrine, for reproof, for correction, for instruction in righteousness; that the man of God may be perfect, thoroughly furnished unto all good works” (2 Tim. 3:15-17). Thus we see that the Scriptures are the complete and all-sufficient guide of man after he has been born of God and saved from coming wrath. There is another point of all importance here. When the inspired Apostle says that “evil men and seducers shall wax worse and worse, deceiving and being deceived,” he turns to Scripture as the only resource in an evil day; and would have Timothy know of whom he had learned the things he had been assured of. If many were asked in the present day from whom they had learned this doctrine and that, they would find it difficult to reply; some would say, “Our church teaches it.” Now, true Spirit-taught and Spirit-led souls would say they learned it from the Scriptures, and therefore they can assert its divine authority. This is the exercise of faith, and nothing short of it can be pleasing to God. And as to the expression, “Our church teaches,” it is not only unauthorized by the Scriptures, but exactly opposite to Scripture, for there we learn that, instead of the church teaching, the church is taught, and built up by various gifts from an ascended Saviour (Eph. 4:11-12). If any take refuge in another snare, that such and such doctrines must be true because the clergy have accepted them, we do well to remember that “the faith was once delivered to the saints,” and is therefore the common possession of all true believers. The Scriptures, not clever men, or preachers, or traditions, are our resource in these evil days; but we are enjoined to have that abiding in us that which we have heard from the beginning; that is, the beginning of Christianity (1 John 2:24).
It has been truly said, that the church has no power to give authority to the written Word, because it is the Word of God; but, on the contrary, the Word speaks to us of the authority of the Lord in the church, for He is Lord of all. The Scriptures call for submission, because they are God’s Word; by their own moral evidence and intrinsic authority, they commend themselves to the conscience. Confidence in them, as the Word of God is to us of infinite value.
As to the book of Revelation being too difficult to be understood, it is only another instance of man’s perversion of what is of God; for “Revelation” means revealing, or making known, which is surely something exactly opposite to difficult and inexplicable. Those who approach that book in unfeigned dependence on God’s teaching by His Spirit, not only will certainly have the blessing promised in the third verse, but will have an intelligence as to things around, and their hearts drawn into the path of devotedness in a way that they could not otherwise have known.
With regard to “every scripture being God-breathed,” if they were merely the expression of the judgment of even good men, we should then have only human instead of divine authority, and have no basis for faith, no authority of God on which to rest. Those, therefore, who deny inspiration are always restless, and have nothing but uncertainty as to the eternal future.
We shall be told by some that the human element is easily perceived in the sacred writings; to which we reply, of this there is no doubt. No one can be familiar with the writings of Paul, John, Peter, or James, without being struck with the style with which each sets forth his particular line of truth. The same thing is seen in the Old Testament. How different was the manner in which Moses, Isaiah, David, Jeremiah and others, communicated the instruction for which the Spirit of God employed them. No doubt God not only selected His workmen, but each at the very time, and in the state and circumstances He was pleased to appoint, as best suited to carry out His mind and will. He called into the service of inspiration a king or a fisherman, a man of wealth or of poverty, a learned man brought up at Gamaliel’s feet, or an unlearned, a mighty man or a feeble woman, just as it pleased Him. He used their tongues, tears, affections, memories, or pens. He instructed them by direct intercourse with Himself, by visions, dreams, what others had written, or by the Spirit’s teaching and revelation. He used them in a palace, or a dungeon; in a shipwreck, or before magistrates; in poverty, or in abundance; in distress, or in joy, or other circumstances; as well as concerning what they saw, and heard, and felt. He who had used all kinds of instruments in the history of His people to accomplish His purposes, could use any means He was pleased to select in giving us His own revelation of His will. All are His servants.
The various writers were not only enlightened, but they were inspired. In giving us their writings, they acted not according to their own will, but in so doing, carried out God’s will; so that what they communicated is over and over again called “the Word of God.” For example, in the Old Testament we read, “The Spirit of God came upon Azariah,” and, “The Word of God came unto Nathan, saying,” etc., and in the New Testament we read of one who had “abundance of revelations,” and of his communicating to believers what he had received, as “the Word of the Lord.”
No doubt, God could do without men in communicating His mind if He saw fit. An unseen hand has written it on the plaster of the wall. He also opened the mouth of an ass to speak with man’s voice. He can use any instrument He pleases. He has also put words by His Spirit into the mouth of a wicked Balaam, and allowed the foul betrayer to work miracles as the other apostles. But He is usually pleased, in His loving kindness and tender mercy, to take up such in His service as walk in His fear. He has also caused the words and ways of Satan and wicked men to be recorded, but the writer was inspired to write so much or so little of them as suited the will of God, in exposing their wickedness, and in ministering for our warning and blessing. Faith rejoices in the perfect love and almighty power of our Saviour God.
(To be Continued).

The Fruit of Faithfulness

An only daughter of wealthy and worldly parents, when once absent from home, was brought to hear of and to accept Christ as her Saviour. With joy she returned to tell her parents; but, in their displeasure, they alternately reproached and pleaded with her to give up her new found riches; finally, her father, threatening her with loss of home and inheritance, asked her decision. With tearful eyes and aching heart she listened; and then, drawing up to her piano, with beautiful voice she sang out her reply in the following words:
I’ll live for Him who died for me,
How happy then my life shall be!
I’ll live for Him who died for me,
My Saviour, and my God!
My life, my love, I give to Thee,
Thou Lamb of God, who died for me;
O may I ever faithful be,
My Saviour, and my God!
Her testimony was afterward the means of bringing both her parents to Christ.

Scripture Study: Matthew 8

In the miracles of the Lord Jesus, we have a testimony that God, full of goodness which can meet man’s need, has visited the world. The power of God is manifested in goodness and grace. Where the need is felt, there the blessing is realized.
Matthew 8:1-4. The first one brought before us is a leper, a Jew who knows that the Lord has the power, but does not know His willingness to cleanse him. He put forth His hand and touched him, saying, “I will; be thou clean.” And lo, the leprosy fled. Any other man would be defiled, but Jehovah-Jesus cleansed him by His touch. None but He could say, “I will”; His word declares it done. And the man is sent to Israel’s priest (see Lev. 14), to offer an offering seldom used; for none but God could heal leprosy, and this to the priest bore witness that Emmanuel was come.
Matthew 8:5-13. Here we have the Gentile centurion’s servant healed. It is God’s purpose to bless Gentiles also. Luke tells us the centurion sent the elders of the Jews to Jesus. This was to show the Gentile’s place as getting blessing through them. The centurion’s faith was beyond Israel’s; for owning his utter unworthiness, he owns the Lord as omnipotent, with all principalities and powers at His command. His faith gets its full answer.
Matthew 8:14-15. Peter’s wife’s mother is raised out of a fever by the touch of His hand, and she arose and served Him. What a rest to her spirit to serve Him.
Matthew 8:16-17. In the evening many were delivered from the power of demons, and healed of their sicknesses by His word, and Isaiah 53:4, is fulfilled: “Himself took our infirmities, and bore our sicknesses.” God was there delivering from the effects of sin and Satan’s power; as another has said, “Jesus put Himself in heart under the weight of all the sorrows that oppressed Israel, in order to relieve and heal them.”
Matthew 8:18-22. He does not seek popularity, but to do the Father’s will, so He gave commandment to depart to the other side. A scribe proposes to follow Him whithersoever He goeth. Then the truth comes out that He is a homeless stranger, with nowhere to lay His head, and those who follow Him, must be prepared for this. Another of His disciples said, “Suffer me first to go and bury my father,” but the Lord’s claims are above everything else. If we follow the Lord, we will lose our character as well as our comforts, but we are gainers in the end (Matt. 19:29).
Matthew 8:23-27. Then His disciples follow Him, but it leads into a storm where He does not seem to regard their danger. He is testing their faith. How doubting we are. They could not perish with Him in the ship. Can we? The Lord of glory cannot fail; there are no accidents with Him. It was a deep lesson of their unbelief, and His faithfulness, Who can raise a storm and calm the winds and waves? What manner of Man is this, that even the winds and the sea obey Him?
Matthew 8:28-9:1. Here we have the power of Satan holding its victims, and how this poor world would rather be permitted to go on in its own way, living in sin, under the power of Satan and without God. (The gospel turns the world upside down. Acts 17:6). Jesus delivers this remnant who own Him as the Son of God, and the demons enter into the swine; a picture of apostate Israel, they rush on to their destruction. They that saw it, who kept the swine, fled and told it in the city, and the whole city came out, not to welcome Him, but besought Him to depart out of their coasts. And entering into a ship, He passed over and came to His own city.
To this day the world does not want the One they put out.

Truths for Young Christians: Pride

PRIDE
The sin of selfishness, of which we spoke last, may be specially characterized as the sin most unlike Christ, but the sin of pride is directly of the Devil; if the one is anti-Christian, the other is Satanic. Such, indeed, is the calm language of the Scripture. In 1 Timothy 3:6 we read that being lifted up with pride was the cause “of the condemnation of the devil,” and in Ezekiel 28, we read the detailed account of how the heart of one who was once “full of wisdom and perfect in beauty,” was “lifted up because of his beauty,” and his “reason was corrupted by reason of his brightness,” and who therefore fell.
Pride Springs From the Heart
Pride is in every human heart, it runs in man’s blood, all are afflicted with this disease, though by too many, alas, it is regarded rather as an ornament than a blemish. The Word of God says simply of “a high look and a proud heart” so much thought of in the world, that they are sin (Prov. 21:4). They are hateful to God (Prov. 6:16-17; 16:5), and to Christ, typified by wisdom (Prov. 8:13).
The root of all pride is in the heart, “out of the heart proceed pride, foolishness” (Mark 7:22). How can a young believer get rid of a proud heart? There is, indeed, but one way, that is by sitting at the feet of Him who is meek and lowly in heart until we are ashamed any longer to cherish a quality so unlike Christ, so like Satan.
Spiritual Pride
Let us consider one or two varieties of pride spoken of in the Word. We find the type of one variety “spiritual or religious pride” in the Pharisees of old, who were not ashamed to come before God with words like these, “God, I thank Thee, I am not as other men are.” Surely no vestige of such an expression finds a place in the prayers of our readers.
We must remember that pride is one of the characteristics of the last days (2 Tim. 3:2), and therefore we have need to be greatly on our watch against it. Spiritual pride is perhaps the worst variety, because it is not ashamed to show itself in connection with Christ’s name, a terrible thing we think that such profess to be followers of the meek and lowly Jesus. Let this sin at least then be kept far from us and let none who read these lines sin so fearfully against God, as to use His truth to help them to commit the very sin of the Devil —spiritual pride. When we really get into His presence this can never be the case, “Then went King David in, and sat before the Lord, and he said, ‘Who am I, O Lord God? and what is my house that Thou halt brought me hitherto?’” (2 Sam. 7:18). But when we are out of God’s presence then boasting begins (2 Cor. 12:7).
Pride of Position
Another sort of pride arises from riches and position. We may see an instance of this in Hezekiah (2 Kings 20:13), in Nebuchadnezzar (Dan. 4:30), in Belshazzar (Dan. 5:22), in Herod (Acts 12:21), and in many others. The question is, Is it seen in us? Do we in any of our acts betray this mean, this debasing, this un-Christlike spirit to any who are poorer and humbler than ourselves? Surely not; for if spiritual pride is terrible, this is contemptible, and clearly shows that we have never really understood the place where God’s sovereign grace has set us. It is alluded to in James 3.
The Remedy
But it does not need riches to produce pride; this fatal seed is seen, alas, everywhere, and often those who are poorest are most proud, and this is especially the case amongst the Lord’s people. Many having become Christians and mixing freely on equal terms as Christians with those they never could have met on any other, instead of increasing in humility, have lost what little they possessed, and developed a proud heart.
When we talk of having very sensitive feelings, and being hurt by remarks of others, it is often only pride, and shows how miserably we are taken up with ourselves. Another variety of pride is shown in outward adornment, dressing after the fashion of the world, and in a manner unsuited to Christian position. Another variety is being puffed up by gifts God may have bestowed upon me.
But I am sure that we have spoken enough of the evil; for the remedy let us look for a moment at the Lord Jesus Christ.
We find in the first place that He Himself expressly declares “that He is meek and lowly in heart” (Matt. 11:29). We find Him showing this in various ways; by taking our nature, (Phil. 2:7; Heb. 2:16) in His choice of station in life (John 9:29). How many of us who profess to show His spirit, if left to ourselves to choose our place in this world, would have made such a selection? We are called to be conformed to the image of our Lord. Which among us is so? We may well ask this question, when we see Christians trying to be more than their fathers were, and pushing their children still higher than themselves. We strictly obey the first half of James 1:9-10, But how many rich rejoice when they are made low?
Christ or Self
There is a line visible from heaven whether we on earth can distinguish it or not. On one side of it are those who, be what they may, would still be something more, or seem to be something they are not; who cannot enjoy what they have, because they desire more, and cannot be gratified because they are not satisfied. There are those who are ashamed of the position their Master chose, and who are proud of one He refused to occupy. Christ and those that bear His image are not on this side of the line. It is not that we are called to change our station, but we are called to change our mind. But we must pass on.
Not only humbling Himself to the position of a carpenter, Christ became the servant of all, (Matt. 20:28; Luke 22:27,) even washing His disciples’ feet (John 13:5). On account of all this He was despised, (Mark 6:3; John 9:29) and those who follow Him will be despised, too. They will be called mean spirited, and will be pushed aside and trodden down by the proud and ambitious. It matters not. If they have but drunk at the pure spring of humility (Phil. 2), their souls will be so refreshed that they will be full of joy at bearing ever so little of the beauty of their Lord.
What God Thinks of the Humble
Hear what God has to say of them. He hears them (Psa. 9:17); they enjoy His presence, (Isa. 57:15); He delivers them (Job 22:29); He exalts them (Luke 14:11; 18:14); He gives them more grace (James 4:6) while He resists the proud. Saints are exhorted to put on humility and be clothed with it (1 Peter 5:5), a beautiful word, meaning that on whatever side we are approached, humility is seen, to walk in humility, (Eph. 4:1-2), but to beware of false humility (Col. 2:18, 23), which is only pride in disguise.
Nothing perhaps shows more the transforming power of the grace of Christ than when a man, naturally proud and haughty, becomes really meek and lowly in spirit; and nothing tells more strongly of the way in which the letter of truth held apart from Christ corrupts, than when we see a humble quiet person after coming among Christians become vain and puffed up, a sight, alas! which is not rarer than the former.
We plead then, in closing, that our dear readers will seek to cultivate the two graces of which we have already spoken, unselfishness and humility, and thus get a long way on in becoming like Christ, putting away from them, as hateful things, the anti-Christian sin of selfishness and the Satanic sin of pride.
But who is sufficient for these things? Thank God, the answer is not far to seek, “Our sufficiency is of God,” the meek will He teach His way. May we look to Him there in all meekness to put upon us more of the grace of Christ, and fit us better to become humble followers of the Lord Jesus Christ.
“O may that mind in us be found,
That shone so bright in Thee,
The humble, meek, and lowly mind,
From pride and envy free.”

Be Thou in the Fear of the Lord All the Day Long: Proverbs 23:1

When you think, when you speak, when you read, when you write,
When you sing, when you walk, when you seek for delight;
To be kept from all wrong when at home or abroad,
Live always as under the eye of the Lord.
Whatever you read, though the page may allure,
Read nothing of which you are perfectly sure
Consternation at once would be seen in your look,
If God should say, solemnly: “Show Me that book.”
Whatever you think, never think what you feel
You would blush in the presence of God to reveal:
Whatever you say, in a whisper or clear,
Say nothing you would not like JESUS TO HEAR.
Whatever you write, though in haste, or with heed,
Write nothing you would not like JESUS TO READ:
Whatever you sing, in the midst of your glees,
Sing nothing that HIS LISTENING EAR COULD DISPLEASE.
Wherever you go, never go where you fear
Lest the Great God should ask you “How CAMEST THOU HERE?”
Turn away from each pleasure you’d shrink from pursuing
If God should look down and say: “WHAT ART THOU DOING?”

Christ the Center: Or, Why Christians Should Meet in His Name Alone, Part 2

Every Christian who owns the Lordship of Christ, and has visited the Romish places of worship, must have been deeply pained at the reverence paid to the name of the virgin. And is not human nature the same elsewhere? Is there not the same idolatrous tendency where any name is owned as the head of a sect? As that name is exalted, the name of Jesus is disowned, until at length it is a small matter to be a Christian, but a great one to belong to the sect. Surely this is wood, hay and stubble, that will not endure the coming day. In the days of the apostles, Jesus was the name exalted above every name. To exalt another, though it were a Paul, or a Cephas, was denounced by the Spirit of God as carnality and schism. Even to tolerate another name, or names, was virtually to lower the glorious Christ to the level of a mere man (1 Cor. 1:12; 3:4-5).
Is it not the same now? Jesus is worthy of the united worship of the millions of the redeemed who shall be gathered in heaven; therefore He is worthy of the united worship and praise of all Christians now on earth. Whatever others may do, whether they own that name alone, before the world or not, fellow believer, if thou desirest to do the will of God, the path is plain—give up every name and sect, and meet only in the name of Jesus, heaven’s exalted Lord. A question may now arise in the mind as to what order of church government is really according to the mind of God. This leads us to the second consideration.
Second. The sovereignty of the Spirit of God as the second reason why Christians should meet in the name of the Lord Jesus alone. Before Jesus left this world, while in the midst of His sorrowing disciples, He said, “I will pray the Father, and He shall give you another Comforter, that He may abide with you forever; even the Spirit of truth; whom the world cannot receive, because it seeth Him not, neither knoweth Him; but ye know Him; for He dwelleth with you, and shall be in you” (John 14:16-17).
The Lord Jesus solemnly promised that this Comforter or Guardian should teach us all things. Jesus says, “He shall testify of Me” (John 15:26). Observe, Jesus did not promise an influence; but the real, divine person of the Holy Ghost; as real a person as Jesus. And as really as Jesus had testified of the Father, so really should the Spirit testify of Jesus. And further, that He, the Holy Spirit, should guide us into all truth. “He shall glorify Me” (John 16:14). This promise God hath fulfilled. Jesus being glorified on high, God hath sent the Holy Ghost (Acts 2:4-38). Now from that moment, we search in vain in the New Testament for any church government except the sovereign guidance of the Holy Ghost. As really as the blessed Lord had been present with the disciples in the gospels, equally so is the Holy Ghost present with the church in the Acts. Pentecost was a marvelous display of the presence and power of the Holy Ghost. And again, “When they had prayed, the place was shaken where they were assembled together; and they were all filled with the Holy Ghost, and they spake the Word of God with boldness” (Acts 4:31). Yea, so real was the presence of the Holy Ghost, that Peter in the case of Ananias said, “Why hath Satan filled thine heart to lie to the Holy Ghost?” (Acts 5:3). And when the gospel was preached to the Gentiles, the Holy Ghost fell on them in like manner (Acts 11:15). Also at Antioch (Acts 13:52). And how marked the guidance of the Spirit to the Apostle Paul and his companions when “forbidden of the Holy Ghost to preach the Word in Asia,” and when they would have gone to Bithynia, “but the Spirit suffered them not” (Acts 16:6-7. See also Acts 19:2). If we now turn to 1 Corinthians 12, the government of the Spirit in the church is stated with the utmost clearness, “Now there are diversities of gifts, but the same Spirit.” “But the manifestation of the Spirit is given to every man to profit withal.”
This passage is often applied to the world, in violent opposition to that scripture which saith, “whom the world cannot receive, because it seeth Him not, neither knoweth Him” (John 14:17). But whatever variety of gift in the church, “all these worketh that one and the selfsame Spirit, dividing to every man severally as He will” (1 Cor. 12:11).
Now tell me what denomination thus owns the Spirit of God in our day. Nay, the moment any assembly of Christians do thus own the Spirit of God, that moment they cease to be a sect, or denomination; because the Holy Ghost would not honor any name but the name of Jesus. Now let us compare an assembly in the apostles’ time, with a denominational assembly now, and this will be plain. All the Christians in a neighborhood assembled together in the name of the Lord Jesus; the Spirit gave diversities of gifts; some were gifted to preach, others to teach, others to extort, and so on, with all the various manifestations of the Spirit. And He, the Spirit, was really present in their midst, dividing to every man severally as He would. They speak two or three—if anything is revealed to another that sitteth by, the first holds his peace—and this is the order of God; as we read (1 Cor. 14:29-33), “Let the prophets speak two or three, and let the other judge. If anything be revealed to another that sitteth by, let the first hold his peace. For ye may all prophesy one by one, that all may learn, and all may be comforted. And the spirits of the prophets are subject to the prophets. For God is not the author of confusion, but of peace, as in all churches of the saints.” When the sovereign guidance of the Spirit of God was owned, this was plainly the order.
Now let us enter an assembly belonging to any denomination of the present day. Tell me, Where is the Holy Spirit either expected or allowed to divide to every man severally as He will? This may not be intentional; the presidency of the Holy Ghost is forgotten. A man fills His place; and, whether led of or happy in the Spirit or not, he must occupy the time. This disowning of the personal presence and sovereign guidance of God the Holy Ghost is most sad every way. The diverse gifts are not exercised; the work of the ministry becomes a burden to the one man. But more than all, the Lord is disowned in the assembly for guidance in worship; and a human order, or rather, every kind of human disorder, takes the place.
It may sound well to call it liberty of conscience; But where is the liberty for the Spirit of God to use whom He will for the edification of the church of God? Is this a light matter? Was not the disowning the guidance and government of God by His people Israel, and the desire to have a man in God’s place, the first sad step in the downward path of that people? And what is the history of the prophets but that of a few men (in the midst of general departure from God) still finding and holding fast this blessed reality—the presence of God? How solemn the teaching in the book of Jeremiah: he sat alone, yet called by the name of the Lord God of Hosts—how sweet were the words of the Lord to him, “Let them return unto thee; but return not thou unto them.” (See Jer. 15:16-21).
(Continued and to be Continued).

The Coming and Reign of Our Lord Jesus Christ: The Appearing, Part 1

THE APPEARING
Having looked at the first part of the Christian’s hope— the Lord’s return in the air for His saints— and briefly traced the things coming on the earth, when God shall punish the inhabitants of the world for their iniquity, we now come to the second part, the glorious appearing, or manifestation of Christ. This wonderful event, fruitful with blessing for the believer, and oft presented to our hearts in Scripture as part of our hope, and as an incentive and encouragement to faithfulness and devotedness so long as we remain here below, will be an hour of woe and judgment for the poor world of that day.
Scripture teems with passages relating to our Lord’s second advent, and the attendant circumstances and surroundings; when, accompanied with His heavenly saints, He shall descend to this earth, Himself manifestly and personally dealing with and judging His enemies, gathering “out of His kingdom all things (margin, scandals) that offend and them which do iniquity” (Matt. 13:41), and taking the reins of government of the world into His own hand (Zech. 9:13; Isa. 40:10).
The judgment of the living will be a prominent feature of that day; but how precious for the Christian to know that he has been already judged in the Person of his Substitute, Christ Himself. “Verily, verily,” saith the Lord, “He that heareth My word, and believeth on Him that sent Me, hath everlasting life, and shall not come into condemnation (or judgment); but is passed from death unto life” (John 5:24). Here are our Lord’s own words telling us of our deliverance. Our judgment is passed; He drank the bitter cup in our stead; His precious blood, poured out on Calvary, has washed all our sins away, and thus we have the comforting testimony of John in the first Epistle (1 John 4:16-17), “God is love; and he that dwelleth in love dwelleth in God, and God in him. Herein is our love” (marg., love with us) “made perfect, that we may have boldness in the day of judgment: because as He is, so are we in this world.”
But some will reply, “Do you mean to say that the Christian will not be judged at all? I thought we should all stand together before God to know whether heaven would be our portion or not!” This arises from accepting the thoughts of men, instead of gathering all from the Word of God. The Christian will never be judged for his sins, because they were all borne and suffered for by Jesus on the tree, and put away forever (Heb. 10:12-17). God now expects the believer to walk in practical holiness, and have nothing more to say to sin at all. He is now called to follow Christ, and suffer for His sake. But “if any man sin, we have an advocate with the Father,” (the Christian is a child), “Jesus Christ the righteous” (1 John 2:1). Again, “If we confess our sins, He is faithful and just to forgive us our sins, and to cleanse us from all unrighteousness” (1 John 1:9).
If, however, we neglect confession to God, and go on in sin, as He will not condemn us with the world, He will judge us now, by weakness, sickness, or even putting to sleep, cutting off in His governmental ways, though saving in grace. This truth is taught us in Paul’s letters to the saints at Corinth, who, though real Christians, were allowing sin unjudged. “For this cause,” says the Apostle by the Spirit, “many are weak and sickly among you, and many sleep. For if we would judge ourselves, we should not be judged. But when we are judged, we are chastened of the Lord, that we should not be condemned with the world” (1 Cor. 11:31).
It is perfectly true that we “must all appear before the judgment seat (or bar) of Christ; that every one may receive the things done in his body, according to that he hath done, whether it be good or bad” (2 Cor. 5:10). This is a broad statement as to the fact; but other scriptures clearly show that the time and circumstances in which the Christian will appear there, are entirely different from those in which the ungodly will be judged.
The ungodly, whether quick or dead, will be arraigned to be judged according to their works— the former at the commencement, the latter at the close of the reign of Christ—and punished for their iniquity. The saints are already justified from all things, but will be manifested in the Presence of Christ in bodies changed and fashioned like unto His own glorious body. We shall not, then, stand before Him like the wicked, as guilty culprits before a stern and inexorable judge, but as justified persons to give account of ourselves (Rom. 14:12) and to have our works thoroughly tested, as the following scripture shows.
“Every man’s work shall be made manifest: for the day shall declare it, because it shall be revealed by fire; and the fire shall try every man’s work of what sort it is. If any man’s work abide which he hath built thereupon” (that is on Jesus Christ, the only foundation), “he shall receive a reward. If any man’s work shall be burned, he shall suffer loss: but he himself shall be saved; yet so as by fire” (1 Cor. 3:10-17). “Every one of us shall give account of himself to God” (Rom. 14:12). This manifestation will probably take place shortly after our being caught up to meet Him.
Being like Him at that wondrous moment, we shall see ourselves and all our works in the same light as He sees them. And how blessed at that day for those who have built good material, built according to the Word of God, to receive a reward from our gracious Saviour and Lord! to hear the “Well done, good and faithful servant; enter thou into the joy of thy Lord!” and to have praise of God (1 Cor. 4:5). As to our sins, of them He has said already, “Thy sins and thine iniquities I will remember no more.” “Blessed is the man to whom the Lord will not impute sin” (Rom. 4:8).
(Continued and to be Continued).

The Rest-Stone

“And He said, My presence shall go with thee, and I will give thee rest” (Ex. 33:14).
In India, where burdens are carried on men’s heads and backs, it is customary to provide resting places for them. Stones are set up along the hot dusty roads, just the right height for a man to rest his burden upon until he is refreshed and able to go on his way.
“Ah, Sahib,” said a native Christian to an English gentleman, “Christ is my rest-stone, Christ is all my hope.”
How many of my readers are finding their rest in Christ: turning to Him, and realizing His presence and His support in the difficult pathway they are treading?

Correspondence: Isa. 28:23-29; 1 John 3:6; 1 Cor. 15:58

Question 51: Explain Isaiah 28:23-29. H. E. B.
Answer: This lesson teaches us where the husbandman gets his wisdom. It is a picture of God’s dealings with His people Israel, and so for us also.
There is a time of plowing and harrowing, and of planting and sowing; and God doth instruct him. Here we get discipline and chastisement, and grace also. Then there is a time of reaping and threshing, when the results are reached. Jehovah sifts Israel, but will not destroy. This also cometh forth from Jehovah of hosts: He is wonderful in counsel, great in wisdom, (See Rom. 8:28, for us.)
Question 52: Would you apply 1 John 3:6 to unbelievers or to Christians? H. G.
Answer: John’s Epistle presents Christ as the Eternal Life, and the children of God as having Him as their life. It unfolds to us what is characteristic of Christ and of the children of God; and in contrast, what characterizes the children of the devil.
Sin is in us (1 John 1:8), and while told not to sin, provision is made “if any man sin” (1 John 2:1). But as children of God, we are identified with the divine life and character. These traits of character are looked at separately, enabling us to judge what we should avoid.
The children of the devil, are in darkness, practice sin, say they have not sinned, practice hatred and unrighteousness.
The children of God are in the light, they practice righteousness, love, and truth. All this is seen perfectly in Christ.
The true Christian feels sin in him, and learns from this epistle not to allow its action. Having eternal life, and the Holy Spirit, he can see how inconsistent it is to allow sin in practice.
Now look at 1 John 3:6, “Whoever abides in Him does not sin.” This is the character of Christ and of the children of God. “Whoever sins has not seen Him or known Him.” Here you get the unbeliever’s character. It is not some action, but the character of the man.
Question 53: What are to be the qualifications of a child of God in order to work for the Lord? Would the failures of the past disqualify him; or would his being young in the faith hinder him? How could the Apostle Paul give the exhortation to the Corinthians to be “steadfast, unmoveable, always abounding in the work of the Lord” (1 Cor. 15:58), in face of the severe rebukes he had to give them in previous chapters? J. H. J.
Answer: The first qualification needed to serve the Lord is full assurance of acceptance in the Lord, and fellowship with the Lord. Without this, souls are self-occupied.
Second, we need hearty willingness to do His will— true service is obedience. Service is to be the fruit of communion with the Lord. It is not quantity the Lord seeks, but quality. A stack of hay looks big, but after it goes through the fire it will be small enough (1 Cor. 3:12-13).
The Lord fits each believer for his special place and path, and each should own the Lord as His Master, and receive his orders from Him. The slave doing menial service to his earthly owner, serves the Lord Christ, if he does all for Him (Col. 3:22-26). The believer, enjoying his place in Christ, is also serving (Rom. 14:17-18). If failure in the past could disqualify us, then none of us could serve. Timothy was young, but he was to be an example to the believers (1 Tim. 4:12). Moses failed in his service, Aaron failed, Peter failed, but when he was restored, the Lord said to him publicly, “Feed My sheep” (John 21:15-18).
Remember, fruit makes no noise (Gal. 5:22-23); to talk much and to walk badly, is not true service.
The Apostle Paul does not discourage the saints; he rebukes their evils and encourages their true service. 1 Corinthians 15:58 encourages them, pointing out to them that as surely as Christ is risen, so surely their labor is not in vain in the Lord. This laboring takes in their whole life, sisters and brothers alike. (Compare 1 Thess. 1). Christ is our object, all the day and all the night.

Inspiration of the Scriptures: The New Testament, Part 2

THE NEW TESTAMENT.
We have had the most incontestable proofs of the Old Testament Scriptures being inspired, and authenticated by our Lord and His apostles. Our Lord honored, obeyed, and used the very words of the Old Testament; and with the apostles an appeal to their authority was final. As to the inspiration of the New Testament, we are told by an inspired Apostle, “which things [the things of God] also we speak, not in the words which man’s wisdom teacheth, but which the Holy Ghost teacheth” (1 Cor. 2:13), and at the same time he emphatically disallows all ability in the natural man, either to know, receive, or to communicate the things of God. So completely does the Spirit of God teach, that He alone is the source of the Scriptures of truth, that though Paul had been caught up into the third heaven, yet we never find him on this account asserting any competency for divine things apart from the Holy Spirit.
Although the New Testament Scriptures are equally inspired as the Old, and are interwoven with many hundreds of quotations from it, yet the instruments employed in giving us the sacred writings were somewhat different. Thus we find that the Old Testament prophets did not understand their own prophecies, and searched as to what they signified; yet they knew they were ministering to others rather than to themselves, even unto us who now have the ministry of the Holy Ghost come down from heaven. Their prophecies testified also to “the sufferings of Christ and the glories which should follow” so that the church on earth formed no part of their ministry. We know from other scriptures that the church was not revealed in the Old Testament, although now we can go back to it and find typical instruction concerning the church. We are emphatically taught, that the revelation of the church or assembly was “hid in God,” “kept secret since the world began,” and “not made known” till Saul of Tarsus was called by divine grace. We have, therefore, in the Old Testament after the call of Abraham, Israel and the heathen or Gentile nations; but in the New Testament we have Jews, Gentiles, and the church of God (1 Cor. 10:32. See also 1 Peter 1:10-14; Rom. 16:25; Eph. 3:3,5,9).
There is an important text in 2 Timothy 2:15, “Study to show thyself approved unto God, a workman that needeth not to be ashamed, rightly dividing the word of truth.” Now this rightly dividing the word of truth does not simply mean, as many say, giving to the saint and sinner each their portion, but it is cutting in a straight line the word of truth. In consequence of the accomplishment of eternal redemption, and the coming down of the Holy Ghost because of Christ being glorified, believers are brought into a totally different position and state to what could possibly have been known before. Then from the second chapter of Acts, when the Holy Spirit came down to indwell believers, and abide with us forever, we have the truth flowing out by the inspired apostles, and made known as could not have been made known before (1 Cor. 2:9-10). We judge, therefore, that we rightly divide, or cut in a straight line the word of truth, when we accept, as in contrast with God’s earthly people, our standing in Christ in the heavenlies, and know our union with Him by the Holy Spirit sent down by Him in ascension as Lord and Christ, and given to be the Head over all things to His assembly which is His body, the fullness of Him which filleth all in all (Eph. 1:19-23).
No doubt what is known as dispensational truth is also included in “rightly dividing [or cutting in a straight line] the word of truth.” Hence we find some scriptures which apply to God’s earthly people, the Jews; and others which especially belong to His heavenly people, the church — the body and bride of Christ. We have also instruction concerning millennial saints, and other Old Testament saints, the reign of Christ, and more.
The New Testament was written after the coming of the Holy Spirit, hence the intelligence of these inspired writers compared with the Old Testament prophets. Again, we do not find apostles saying, “Thus saith Jehovah,” because their relationship was not with God as Jehovah, but with the Father and the Son. Hence they wrote, “Blessed be the God and Father of our Lord Jesus Christ,” and the like.
When Paul wrote to the Galatians, he reminded them that he had received the gospel which he preached, from the Lord Himself, and not of man, nor by man; and so divinely-given did he know his ministry to be, that he could solemnly declare, “though we, or an angel from heaven, preach any other gospel unto you than that which we have preached unto you, let him be accursed.” How could he possibly use such language unless he had known it to be given him by the Lord? We may remember, perhaps, that when he was converted by a sight of the Lord Jesus in heaven, he was then told, “I have appeared unto thee for this purpose, to make thee a minister and a witness both of these things which thou hast seen, and of those things in the which I will appear unto thee” (Acts 26:16); so there is the plainest possible evidence that the Apostle Paul received his commission for the ministry of the Word immediately from the Lord Jesus Christ Himself.
The New Testament, like the Old, is also presented to us in three parts.
1. We have first the gospels, and Acts 1, giving us the coming into the world of the Only-begotten; His birth, life, death, resurrection, and ascension, and His bidding His disciples to wait for the gift of the Holy Spirit, and assuring them, by angelic ministry, that He shall so come in like manner as His disciples saw Him go into heaven.
2. From Acts 2, to the coming of our Lord for us, we have another portion of the New Testament chiefly occupied with the church of God on earth; its calling, endowments, ministry, and hope.
3. The book of Revelation, and other prophetic writings in the epistles and gospels, which give us the divine estimate of everything here, and God’s judgment of evil, and the translation and reign of saints with Christ, concluding with the new heaven and new earth, in which righteousness will dwell.
With regard to the four gospels, they are almost entirely occupied with our Lord’s own ways, ministry, and works. We are told that the “words” He spake, He received from the Father; so perfect was He as Man in dependence on the Father, that He said, “The Father which sent Me, He gave Me a commandment, what I should say and what I should speak.” Again, we read, “He whom God hath sent speaketh the words of God,” so that nothing could be more truly of, and from God, than the words which He spake. It is no marvel, then, that He should say, “Heaven and earth shall pass away, but My words shall not pass away.”
(Continued and to be Continued).

A Plea for "Sound" Reading

Observant persons have not failed to notice that there is a serious deterioration in the quality of literature so much in demand the past number of years. A few briefly stated facts may serve to substantiate as well as explain this statement.
In a leading daily newspaper of one of our large eastern cities, an editorial appeared lately from which the following is extracted, with a few words altered: “Yet no authors of the upper rank are left. No author’s writings today are worth intimate reading. Plays and novels win larger pay than they ever did in the past. The habit of reading good authors is dying out. Youth reads less than it did. Yet in life’s purest joys, those who cherish noble books, know there is naught better than the intimate and constant reading of the few greater authors.”
There is nothing perhaps, that will mold a man’s character in the noblest and best ways of life, like association with his fellowmen who exhibit the best virtues in their lives. The Christian will, of course, associate with those who are God’s. Next to this is the influence of literature, which will do more to character the young person in life than any other influence.
Thus according to our reading matter so will our thoughts be; and how careful the young Christian should be to choose only the noblest and best books, in order that his thoughts might be formed to the glory of God. We are all very careful in choosing friends to see that they are of the most worthy kind; and that their companionship will be for our mutual benefit. Should we not apply the same care to our choice of books? If we did, many that are unbecoming would be wisely laid aside.
Any person frequenting our many Libraries can readily see how eagerly youth takes to the lighter reading matter. “Card” records show the long list of readers the novels have, contrasted with the few records that show the more worthy books having been “out.” Almost invariably the larger crowd may be seen standing about the “Fiction shelves.” Perhaps for lack of parental restrictions, children are becoming alarmingly attracted more and more to this questionable class of books.
A few years ago the Canadian authorities made vain efforts to prevent the “American” Sunday newspaper from crossing the border on account of the bad influence of their “Comic” and “Magazine” sections. Their efforts were useless. The clamor of the people was for such reading. A sober-minded person can easily see that the Sunday newspaper’s influence is not for good in the homes where it is used. They are of the same general class as the ordinary novel. By this we do not mean that the newspaper is not very useful in its proper sphere. The monthly magazines that are an exceptionally worthy factor along educational lines, have of late years introduced an abundant supply of fiction; while the number of purely fictitious are increasing rapidly.
These observations, to some, may seem to savor of partiality. Some there are who cannot tolerate anything which differs with their individual creed; and they will, to a great extent, exclude all reading not upholding their particular ideas. Secular reading is good and necessary when kept in its proper place. Next to association with man and tuition, book reading is a splendid factor in supplying the mind with useful and necessary knowledge.
The writings spoken against in the foregoing paragraphs are not the so-called “dime novels.” They are usually downed by public sentiment and limited to a low class of readers. The writer has specially in view those writings that have no basis, or which are only works of the imaginative mind. When read they leave no lasting impressions upon the mind, except an exquisite sense of an airy nothingness. Written under the inspiration of imaginative minds, they readily appeal to the corresponding qualities in other persons, and are easily imposed upon the majority. They excite the sentimental qualities of the mind rather than the reasonable. The stronger qualities of the mind are not catered to, or cultivated by the ordinary fictitious work.
Whatever meritorious distinctions some of the fictitious works may have in delineating the social, political or historical times in which their scenes are set, all this is often obscured by that which can be condemned in the same volume.
At this point some will begin to make such inquiries as these: — “How can I know whether a book is good or bad before reading it?” or: “How am I to choose my reading matter?” While one cannot possibly choose subjects for another, yet general suggestions are appropriate; and many an elderly person can look with gratitude to the kindly advice given in youth. How many of us can recall the pious care with which our mothers would carefully choose our Sunday afternoon reading matter. How grateful we are now that the young mind was so trained.
To those who are well acquainted with books, the title will often convey an idea of the whole import of the contents of the book. Good authors are careful to select their titles accordingly. Then in all worthy books is the preface or introduction. This foreword is often as important as the text itself; and like a glow of light cast over the pages. In nearly all cases it gives a fair outline of what is found in the book. Again, while one cannot choose for another, it is best for young persons to give heed to the advice of elders and make use of their choice in reading matter.
Let us consider now for a moment a special class of fictitious writings, the so called “Religious novel.” Such works as “Quo Vadis,” “Ben Hur,” “The Dawn by Galilee,” “The White Christ” may be taken as fair examples of this class of books, whose popularity defies realization. All written by the more prominent novelists, and each title usually with the affix, “A tale of the Christ,” or some similar term.
And all who “live godly in Christ Jesus” will testify that the Scriptures declare:
“This world is in league with scorn and hate,
Tho’ clad in a garb of cultured state.
Conform to its ways, and smiles are thine,
But frowns are thy lot, should’st thou decline.”
The books so strongly protested against here are attractive because they pass as “religious” novels. They assume to do homage to the phraseology of religion: using Biblical subjects and characters to accomplish this end. Some of the most bold take upon them the liberty of using the character of the divine Person, our Lord and Saviour. It might well be said they degrade the Lord’s divine character down to a human level. The young Christian who lives “to serve the Lord Christ” will readily detect the ordinary light novel and shun it; but the deep subtlety of those clothed in a religious cloak are seldom found out until they commence their ruinous work upon the soul. When reading them, the lure and the charm carries the mind onward; while the soul is left hungering for its needed spiritual food.
Through their pages the “Old Serpent” uses his time-worn tactics to take away the sense of reverence due to the Lord.
(To be continued).

Scripture Study: Matthew 9

Matthew 9:2-8. This is man in his helplessness (paralytic), brought by the faith of other men. The Lord recognizes this and says to him, “Son, be of good cheer; thy sins be forgiven thee.” This is grace to the undeserving, but in the religious mind it was blasphemy. How blind they were to Who was there, even when the multitude saw the man carrying his bed! They were in fear and glorified God, who had given such power to men. Why did they not see that it was Emmanuel, Jehovah, the Saviour of His people, the Son of Man? Grace that forgives sins, is declared here.
Matthew 9:9. In the calling of Matthew, the publican, grace calls and fits him for the service of Christ.
Matthew 9:10-13. This grace is seen in eating with sinners, calling them to repentance and meeting their need, but the rich pharisee goes away empty.
Matthew 9:14-17. This new wine of grace could not be put into the old forms of Judaism. Man in the flesh could not receive it (Rom. 8:9). There must be a new birth, a new righteousness, a new joy. Christ is the bridegroom, and they could not fast while He was there, but His rejection is in view, and they will fast when He would be taken from them (John 16:20-22).
Matthew 9:18-26. The history of Israel is pictured. The ruler’s daughter is dying, and in fact dead. The Lord, at the ruler’s request, goes to heal her, but on the way individual faith gets the blessing. The woman is healed and made glad. Then, when the Lord comes as Son of Man, Israel will be raised again, that is, restored as a nation.
Matthew 9:27-31. Two blind men own Him as Son of David and they follow Him into the house. He questions them, and they confess their faith in His ability to open their eyes, and their eyes are opened. This is another picture of the Jewish remnant of faith. Then as they go forth, the dumb man has his mouth opened. The multitudes marvel and confess that this was new in Israel, but the Pharisees show their hatred, putting it down to Satan’s work. Such is religious man.
Matthew 9:35. The Lord carries on this service in cities and villages: teaching, preaching and healing— goodness and grace manifested in power.
Matthew 9:36-38. This good Shepherd of Israel is moved with compassion, and pities the flock, and engages the disciples in prayer that the Lord would send forth laborers into His harvest. May we enter into this.

Truths for Young Christians: Envy

ENVY.
Let us now consider briefly the examples of this dangerous sin, that have been recorded for our instruction. I say dangerous because we shall see that such is its character.
The First Example
is that of Cain. He, seeing that his brother’s offering was accepted (being with blood), while his was rejected, became envious of his brother, this led to anger, this to hatred, and this to murder, and in 1 John 3:12 this case is given as an express warning to us as Christians.
The next illustration we may take is in Genesis 26:14. The Philistines envied Isaac’s earthly prosperity, just as Cain envied Abel’s spiritual prosperity. (See Eccl. 4:4.) Their envy was shown by maliciousness (Gen. 26:15).
We next pass on to Laban’s sons, (Gen. 31:1,) who became envious of Jacob; Laban also became full of anger against him, though God did not permit him to show it (Gen. 31:2, 24). It is worthy of note that, though Isaac and Jacob were both envied for their riches, and we do not find that Abraham (although equally rich) ever was, a fact that says a great deal for his character. The next example is that of Joseph’s brethren in Genesis 37:11, and the result is, first they stripped him and threw him into a pit to perish, and next sold him into slavery for twenty pieces of silver, acts which we can only characterize as intense cruelty, springing solely from envy.
Envy in a Child of God
Joshua is one of the last we should have expected to find this evil in, but the seed is alas! in all our hearts; and we actually find Joshua (Num. 11), trying to hinder God’s work, led on by this fearful and dangerous spirit. It is, however, only just to add that it is possible that the envy was not for his own sake but for Moses’, whose servant he was. We have only, however, to go on to the very next chapter to find an undoubted instance of envy, no less an one than Aaron, the high priest, and in Miriam also. They did not like the growing nearness of Moses to God, and the difference of the way in which the Lord spoke to him and them; and envy led them to despise God’s servant. The Lord, however, did not leave Moses to fight his own battles, for Miriam became leprous, white as snow. The sin of Korah which follows closely in Numbers 16. was entirely prompted by envy (Psa. 106:16,), and led to still more awful consequences. Envy in this case led Korah, Dathan, and Abiram into fearful lying against and reviling of Moses and Aaron (Num. 16:13, 14), and brought upon them a most appalling death (Num. 16:32), so swift was God to visit their sin upon them.
Envy Leads to Murder
Let us now pass on to Saul in 1 Samuel 18:8-11. Envy here seems to possess Saul so fearfully that it obtains a complete mastery over him, leading him three times to attempt to murder David. To one who does not know how rapidly and fatally the poison of envy works, it seems almost incredible that for such a trivial reason (ver. 8) Saul could have sought to kill the very one who had just delivered Israel. Yet I am sure that there is not one of us who knows anything of his own heart, but can trace the seeds of great crimes in the feelings prompted by envy.
In Ezekiel 35:11 we find in the case of Edom that envy leads to hatred. In the case of Daniel 6:3, 4 it is, I think, clear, that envy prompted the presidents and princes to their cruel course; which cannot be called anything but wicked and unscrupulous. We now pass on to the most fearful thing envy ever accomplished, in Mark 15:19. Jesus, the son of God, was delivered up to Pilate, from the wretched miserable feeling of envy, that had eaten away all that was even human in the hearts of God’s professed servants, the chief priests. Here envy led them to crucify Christ.
In Acts 13:45, we find the same horrible sin, leading the Jews through hatred of the success of the gospel to lying and blaspheming; and in 17:5, a similar company led away by the same feelings were guilty of rioting and violence!
(Continued and to be Continued).

Christ the Center: Or, Why Christians Should Meet in His Name Alone, Part 3

Such is the solemn yet blessed place of all, in these days, who have been led to own the real presence of the Holy Ghost in the assembly. The Lord’s words have indeed been found to be sweeter than man’s. O, that all God’s dear children, in every denomination, knew the blessedness of unfeigned subjection of heart to the sovereign guidance of the Holy Ghost. Where there is this, not in mere form but in reality, He does testify of Christ in such a manner, that no human wisdom can even imitate. Often the hymns by one, the prayers by others, and the reading of the Word, as led by the Holy Ghost, so manifest the divine guidance, and give such a sense of the presence of God, as can only be enjoyed where the Spirit of God is thus owned. I can not then go where He is not owned whom the Father hath sent to guide us and guard us, and abide with us unto the end. It is no matter what may be substituted— whether the Pope, or the Emperor, or the Bishop, or the Conference, or the minister— God is right, and man is wrong. It is not a question of opinion, but of owning or displacing the Holy Ghost as the sovereign guide and ruler of the assembly. I have found the reality of His very presence; and for this I must be separate from every community— Greek, Romish or Protestant, all alike— where He is not thus owned.
Third. I now state the third reason why Christians should meet in the name of Christ alone— the unity of the church; or more correctly, the unity of the one body. I am not aware that there is such a passage of Scripture as one Church, but there is “one body” (Eph. 4:4). The word translated church simply means assembly. It is so used to describe a crowd of heathen in Acts 19:32, 39. The church of God is the assembly of God; comprising every saved person in every place. So the assembly in a particular place would embrace all saved people there. But those, only, who are gathered to the Lord’s name are the representation of the church or assembly. What will characterize those so gathered is their recognition of Christ as the Head of the body the church guiding and guarding, by the Spirit, each one in all things, as in the assemblies of God in the days of the apostles. Therefore no company so gathered or not, can properly be called the assembly of God, unless all Christians in that place are in that way gathered.
Take the following illustration: suppose the president sends out a commander to the army in the Phillippines, and that for a time the army puts itself entirely under his command. It could then be properly called the army of the United States. But if that army were to set aside the commander, and appoint another of its own, or if the army should divide into separate parts, each division appointing its own commander, each soldier might be still an American soldier, but could that divided army be correctly called the regular army of the United States? Having set aside the authority of the president’s appointed commander, Would not every division be in a state of mutiny? And would it not be disloyalty to join the ranks of any such mutinous division?
Now apply this to the church or assembly of God. For a time the authority of the Holy Ghost, sent down from heaven, was owned, just as the American army for a time owned the authority of the president’s appointee. Then the sovereign authority of the Spirit of God was set aside, and the authority of the Pope of Rome was put in the place of God the Holy Ghost. Can the church of Rome, then, be called the true church of God? Impossible! She has mutinied against God’s commander, the Holy Ghost. To join her ranks is disloyalty to Christ. But if I must declare the whole counsel of God, am I not compelled to adopt the same conclusion respecting every division of the professing church?
Take the Greek church: Has it not set aside the command of the Holy Ghost? And though it put so high a person as the emperor of all the Russians in the place of the Holy Ghost, Yet would it not be mutiny to join its ranks? Take again what is called the church of England. Are we not compelled to acknowledge that the sovereign command of the blessed Spirit is entirely set aside? As in Russia, so in England, the head of the world’s government is made the head of the church; and instead of the Holy Ghost being allowed to divide, severally, as He will, a prime minister, whatever his principles may be, can appoint a minister over a given town or village, and according to this system no other person ought dare to name the name of Christ in that so-called parish. Only in this mail I have a letter from a Christian lady in such a town. A young woman had just been to request her to see her dying husband. She went, and put Jesus before the dying man. He appeared very thankful, and wished to see her again. She left him three gospel tracts: as she came out the vicar met her, and in the most authoritative manner, bid her only attend to the bodily wants of the dying man; that his soul’s affairs were his (the vicar’s) business alone. When he left he took all the three tracts with him. O, when I think of the eternal interests of that poor man dying this day, his young, weeping, anxious wife by his side, and then think of the delusion that I know that vicar will put before him, my heart turns sick at the fearful results in thousands of such instances, all springing from the solemn fact that the church of England disowns the sovereign guidance of the Spirit of God.
(Continued and to be Continued).

The Lord Is My Portion

They tell me of the glory of yonder home of light,
The many, many mansions, of pure and true delight.
The walls of purest jasper, the gates of jewels rare.
The street of gold transparent, the saints will walk when there.
The song which ransomed sinners will sing in sweetest tone,
As one and all they gather around the eternal throne.
The greetings there of loved ones, that long since went before,
Embraced and gladly welcomed on yonder peaceful shore.
They speak to me of heaven, where God’s redeemed shall dwell, The Father’s house where pleasures are more than tongue can tell.
They speak to me of freedom from sickness, pain and tears, From sorrow, care, temptations, from death and all its fears.
Not all these put together, with countless joys beside,
Could satisfy the longings of one for whom Christ died.
‘Tis the Babe of Bethlehem’s manger, the One of humble birth, Despised on earth, a stranger, yet none could tell His worth.
The mighty God, Creator of earth and skies above,
Who, in each word and action, made known the Father’s love.
The One with love so boundless, for sinners such as we,
Laid down His life for sinners on Calvary’s cruel tree.
‘Tis He, the Lord of glory, my heart so longs to see,
‘Tis where He is, His presence, makes heaven heaven to me.
Lord, hasten that blest moment, when with Thyself on high, My heart with joyous rapture will swell the jubilee,
Thyself, my joy eternal-my head upon Thy breast-
My satisfying Portion, my Home, my Joy, my Rest.

The Coming and Reign of Our Lord Jesus Christ: The Appearing, Part 2

THE APPEARING.
During the short interval when the apostasy will ripen, and the beast and the false prophet are in power on the earth, the heavenly saints are viewed in the Revelation as twenty-four elders surrounding the throne of God in heaven. Just at the close of that period, following upon the downfall of Babylon on the earth, we have the description of the marriage of the Lamb in the glory of God. This is the moment when Christ shall present the church to Himself glorious, and without spot, for the joy and satisfaction of His own heart of love (Eph. 5:27). It will also be the consummation of her joy, when, united as the Bride to Christ, the heavenly Bridegroom, she becomes the Lamb’s wife. And all heaven will be jubilant with joy (Rev. 19:6-9). “And to her was granted that she should be arrayed in fine linen, clean and white (margin, bright): for the fine linen is the righteousness (or righteousnesses) of saints” (Rev. 19:8). From this we learn that the bridal robe of the church, on the day of the espousals of the Lamb, is the righteousnesses of the saints, wrought by them in the power of the Holy Ghost, whilst living on the earth, then woven together as one beautiful garment, to adorn her when Christ shall present her to Himself, glorious, “not having spot, or wrinkle, or any such thing” (Eph. 5:27).
Then, the heaven being opened, Christ is seen in the vision ready to come forth in power to judge and make war, and take the kingdom. He is seated on a white horse (symbolic of conquering power), and is variously called, Faithful and True, The Word of God, King of kings and Lord of lords; also having a name written that no man knew but He Himself. His clothing is a vesture dipped in blood—figurative of the blood of His enemies about to be shed for their wickedness in opposing Him. Following Him are seen the armies which are in heaven on white horses, like unto their Leader, clothed in fine linen, white and clean. These are the saints here, not angels, as some have thought. For if we turn to Revelation 17:14, we are told that those “that are with Him are called, and chosen, and faithful;” and angels are never spoken of as called, though it is perfectly true that angels will also accompany Him at His revelation from heaven (2 Thess. 1:7).
An angel standing in the sun cries with a loud voice to all the fowls that fly in the midst of heaven, “Come and gather yourselves together unto the supper of the great God” (or the great supper of God) (Rev. 19:17) to eat the flesh of the mighty of the earth. The hour of His judgment is come. The beast and the kings of the earth and their armies are gathered together to make war against the Lord and His army. And the beast is taken, the great leader of Western Europe, and with him the false prophet (antichrist), which wrought miracles before him (Comp. 2 Thess. 2:8, 9; Rev. 13:11-14), and both are cast alive into the lake of fire burning with brimstone, and the rest are slain. Thus suddenly the Lord will judge the leaders of the apostasy, and the boasted military power of the great nations of the West is destroyed. Europe goes on arming; immense standing armies are the order of the day. Men little think what Satan is leading them on to. Nation fights against nation, until the beast comes to the forefront, when the ten kings, who have one mind, give their power and strength to him (Rev. 17:13). He leads one great league against the Christ of God, when they receive the cup of judgment at His hand, and all the fowls are filled with their flesh.
We have several other descriptions in the Word besides this of the Lord’s coming in judgment upon His enemies, not only upon those of the West, but also many others. He doth both judge and make war (Rev. 19:11). Having come in warrior judgment upon the Western Powers directly opposed to Him, He then summarily deals with those to the north and east of the pleasant land, Palestine (Rev. 19:19-21; Zech. 14:3,12:15). “For,” saith the Lord, “My determination is to gather the nations, that I may assemble the kingdoms, to pour upon them Mine indignation, even all My fierce anger: for all the earth shall be devoured with the fire of My jealousy” (Zeph. 3:8). “Behold, evil shall go forth from nation to nation, and a great whirlwind shall be raised up from the coasts of the earth. And the slain of the Lord shall be at that day from one end of the earth even unto the other end of the earth” (Jer. 25:32, 33).
In Jude 14 we find a prophecy of Enoch, “Behold, the Lord cometh with ten thousands of His saints, to execute judgment upon all, and to convince all that are ungodly among them of all their ungodly deeds which they have ungodly committed, and of all their hard speeches which ungodly sinners have spoken against Him.”
In 2 Thessalonians 1:7-10 “... the Lord Jesus shall be revealed from heaven with His mighty angels, in flaming fire taking vengeance on them that know not God, and that obey not the gospel of our Lord Jesus Christ: who shall be punished with everlasting destruction from the presence of the Lord, and from the glory of His power.”
“Behold,” again saith the Lord, “I will make Jerusalem a cup of trembling unto all the people round about, when they shall be in the siege both against Judah, and against Jerusalem. And in that day will I make Jerusalem a burdensome stone for all people: all that burden themselves with it shall be cut in pieces, though all the people of the earth be gathered together against it. In that day, saith the Lord I will smite every horse with astonishment, and his rider with madness; and I will open Mine eyes upon the house of Judah, and will smite every horse of the people with blindness” (Zech. 12:2-4).
(Continued and to be Continued)

Truthfulness

“So they gave it to me: then I cast it into the fire, and there came out this calf.” Ex. 32:24.
Henry Venn Elliott, the pious Brighton minister, writes thus in late life: “If there is one point more than another in morality concerning which I have been especially watchful in my own words, and earnest in teaching my children, it has been strict truth, even to the banishment of ordinary exaggerations.” If a child had made some trifling mistake, and said, “I am so very sorry,” he would say, “Keep your sorrow, my child, for a greater occasion.” He used to refer to Adam’s self-justification, “The woman gave unto me”; to Aaron’s, “There came out this calf”; to Saul’s, “The people took of the spoil”; as compared with David’s earnest ingenuous “I have sinned against the Lord.”
May our young readers ponder well the thought given by Mr. Elliott, not forgetting the solemn words spoken by our Lord in Matthew 12:37, “By thy words thou shalt be justified, and by thy words thou shalt be condemned.”
“Set a watch, O Lord, before my mouth; keep the door of my lips” (Psa. 141:3).

Correspondence: 2 Cor. 12:16; Matt. 12:40; Heb. 5:7

Question 54: Kindly explain 2 Corinthians 12:16. J. H. M.
Answer: There were many things said against Paul. (See 2 Cor. 10:2, 10; 11:4,12-15, 23-26; 12:11)
In 2 Corinthians 12:16 he is going over what others said about him. He did not burden them himself, but, being crafty, he caught them with guile. 2 Corinthians 12:17-18 show that neither he nor any he had sent, had done this. In 2 Corinthians 4:1-2, he flatly contradicts such an allegation. “We have renounced the hidden things of dishonesty, not walking in craftiness, nor handling the Word of God deceitfully; but by manifestation of the truth, commending ourselves to every man’s conscience in the sight of God.”
Question 55: Please explain how the time is reckoned during which the Lord Jesus remained in the grave. Matthew 12:40. Someone has said the Lord must have been crucified on Wednesday, to make three days and three nights; but according to Mark 15:42, it was on Friday, the preparation day. M. W. S.
Answer: “And the evening and the morning were the first day” (Gen. 1:5). From sunset to sunset is the Jewish day. The Lord Jesus was crucified and buried on the sixth day of the week; this is counted one. The seventh day, on which the disciples rested, is another. Mary went and viewed the grave at night when the sabbath was past (Matt. 28:1, read “dusk” instead of “dawn”); the sepulcher was still closed; but early in the morning she pays another visit, and finds it open and empty this is the third day.
Luke 23:56 and 24:1 show that only the Sabbath intervened between buying and bringing the spices by the women.
You will find a sample of this kind of reckoning in 1 Kings 6:37-38.
Question 56: Does Hebrews 5:7 refer to Christ or to Melchizedec?
Answer: It refers to Christ. “Thou art My Son, today have I begotten Thee” (Heb. 5:5), is His personal glory. “Thou art a priest forever after the order of Melchizedec” is His official glory.
It is His history here on earth that makes us feel how truly able He is to take part in the sorrows of redeemed men. Here below He went into all the anguish of death, in dependence on God, praying to Him who was able to save Him out of death. Being here only to obey and to suffer, He did not save Himself. He submitted to God’s will, obeyed Him implicitly, depended on Him for all He needed.
He took death on Himself, and felt its weight upon His soul. He suffered the consequences of what He had undertaken to do, and felt what it was to be under God’s hand in judgment.
His godly fear was in understanding aright man’s sinful condition and what it deserved from God. But He was obedient unto death, perfect in it all. He learned obedience by the things He suffered, then perfected—glorified as man— He became author of eternal salvation to all them that obey Him.
Hebrews 5:7 is the Lord Jesus in the garden of Gethsemane.

Inspiration of the Scriptures: The New Testament, Part 3

THE NEW TESTAMENT.
In reference to the inspiration of the New Testament, we must keep in mind that the Holy Spirit had come and indwelt believers before any part of it was written. Those whom He employed to write it, no longer, as in olden time, wrote what they did not understand, and greatly desired to know; but those who wrote the New Testament, though by the same Spirit, had intelligence of accomplished redemption; they enjoyed communion with the Father and the Son, and new relationships as to what they wrote, for they were filled with joy and with the Holy Spirit. We, therefore, as we have before observed, never find them saying, “Thus saith Jehovah;” but they speak of the Father and the Son, especially in the epistles, as addressing the children of God.
There are, however, certain true marks of the distinguishing activities of the Holy Spirit. He is the Glorifier and Testifier of the Son of God, and takes of the things of Christ, and shows unto us. He always leads into the path of obedience to the Father’s will, and subjection to Christ as Lord of all. Now, without going further as to the operations of the Holy Spirit, by whose power, as come down from heaven, the gospel is preached, it is clear, that every part of the New Testament abounds with proofs of His ministrations by the writers. Moreover, the Scriptures speak to us continually of holiness, truth, righteousness, the grace of God, and of His faithfulness to His own Word; and all taught of Him learn that “the things of God knoweth no man, but the Spirit of God;” that He is “given unto us,” that “we might know the things that are freely given to us of God,” for “the Spirit searcheth all things, yea the deep things of God” (John 14-16; 1 Cor. 2:10, 12). God hath then not given unto us “the spirit of the world,” or “the spirit of fear; but of power, and of love, and of a sound mind.”
Thus while we find it is said that, as to the Old Testament Scriptures, holy men of God were “moved by the Holy Spirit” to give them to us; the New Testament Scriptures were written by the same Spirit through those instruments who knew the Lord Jesus as the Accomplisher of their eternal redemption, and were intelligent by the indwelling Spirit concerning what they wrote.
With regard to the Apostle Paul’s ministry, he was especially a minister of the church or assembly; to him was revealed “the mystery,” as we have seen, and only his writings give us the revelation of the assembly, or “one body,” and its administration. He taught also its practical power on the life and walk of saints now, both individually and collectively, by the same Spirit, as well as the ministration of the affections of Christ in nourishing and cherishing every member of His body. Besides this, and the instructions he received of the Lord from heaven at his conversion, he received also an “abundance of revelations.” For example, as to the Lord’s Supper, when it was instituted by our Lord, the nation of Israel had not been judicially set aside, and the assembly set up, we therefore find it was taken in hope of the kingdom; hence our Lord added, “I will not drink henceforth of this fruit of the vine, until that day when I drink it new with you in My Father’s kingdom” (Matt. 26:29). It is quite true, looking at drinking wine as a type of earthly joy, He has not since had joy in, or with, His earthly people; nor will He till He comes in great power and glory, and brings them into their promised blessing in the land as a repentant people. When, therefore, Israel was actually given up for a time, and this marvelous mystery of the church or assembly began to be built according to God’s eternal purpose, as a people on earth united by one indwelling Spirit to Christ in heaven as the limbs of our natural bodies are united to our head, then the Lord’s Supper needed a special revelation. And what was it? Hear what the Apostle says, “I have received of the Lord, that which also I delivered unto you, that the Lord Jesus, the same night in which He was betrayed, took bread; and when He had given thanks, He brake it, and said, Take eat; this is My body which is broken for you; this do in remembrance of Me. After the same manner also, He took the cup, after supper, saying, This cup is the new testament in My blood; this do ye, as oft as ye drink it, in remembrance of Me. For as oft as ye eat this bread, and drink this cup, ye do show the Lord’s death till He come” (1 Cor. 11:23-26). Now this was a most important revelation, and the Apostle declares that he had it from the glorified Lord, and for us; so that we have His own mind about it now, that instead of watching for events, and waiting, for the kingdom on earth, we are to be carrying out His wish and looking up, not knowing whether the next moment we may hear the shout, and in the twinkling of an eye be taken away from the earth— changed, translated, meet the Lord in the air, and enter with Him on our heavenly and eternal inheritance.
It is also since the glorification of Jesus, and His having sent the Holy Spirit, that we read in Scripture of “the Lord’s table.” Hence it is only after the formation of “one body” by “one spirit,” that we are taught that, in breaking and eating of the “one bread” (one loaf), we express “one body,” “for we are all partakers of that one bread” (or loaf). We are also enjoined to be using diligence to “keep the unity of the Spirit in the bond of peace.” These truths were specially committed to Paul as a minister of the church, or assembly. (See 1 Cor. 10:16-17,21; 12:13; Col. 1:24-26.)
Again, we find as to His coming another special revelation, not given before; for while Old Testament Scriptures speak of the Lord coming to reign with His saints, His coming for us at the rapture was not known till the Apostle wrote to the Thessalonian saints. He said, “This we say unto you by the word of the Lord.” What is it? That the Lord Himself will descend from heaven with a shout, the dead saints be raised, the living changed, and all caught up to meet the Lord in the air, and so be forever with the Lord. It is clear, then, that the Apostle had revealed to him, by the Lord Himself, many things to communicate to us for our instruction and blessing. Peter, in his Second Epistle, most touchingly refers to Paul’s writings being inspired, and therefore Scripture. He says, “Even as our beloved brother Paul also, according to the wisdom given unto him, hath written unto you; as also in all his epistles, speaking in them of these things; in which are some things hard to be understood, which they that are unlearned, and unstable wrest, as they do also the other scriptures, unto their own destruction” (2 Peter 3:15-16).
So careful was the Apostle Paul in his writings to minister what he was commanded of the Lord, that when, on one or two occasions, he gave his own judgment merely, as a servant of the Lord who had the Spirit of God, he would say, “I have no commandment of the Lord; yet I give my judgment as one that hath obtained mercy of the Lord to be faithful” (1 Cor. 7:25,40). But in the same letter, he says, in another part, “If any man think himself to be a prophet or spiritual, let him acknowledge that the things that I write unto you are the commandments of the Lord. But if any man be ignorant, let him be ignorant.” So absolutely was his administration to them the Word of God, that he says, “What! came the Word of God out from you? or came it unto you only?” (1 Cor. 14:36-38).
(Continued and to be continued).

A Plea for "Sound" Reading

“The fear of the Lord is the beginning of wisdom, and the knowledge of the holy is understanding” (Prov. 9:10). But the world has no “fear of God,” and while the religious man of the world will give the Lord his patronage, he fails to reverence His name, or fear Him with that fear begotten in the soul by the Lord Himself. The writings that are of a detrimental character to the truths of Scripture, Mary’s lament may be first applied; but may it not with equal force be written over the pages of the so-called “Religious” novel: “They have taken away my LORD.” This is serious, and if the young Christian will exercise a little diligence, he cannot fail to notice that the divine life within him resents the intrusion of all literature that intermingles the Lord’s name with its contents in an unbecoming way. The place of honor and respect, the glory and reverence due to Jesus Christ as Lord, should be zealously guarded by the young Christian.
The young Christian who is “begotten of God” has “received the Spirit from above,” and has within the finer, delicate instincts of the divine life; which will, if heeded, save him from many of the delusions of the attractive literature of the world.
The voices of the majority prevail to uphold this class of books as good reading. Some, yes, many will say, “But — recommended the books to me and I know they would not ask me to read anything that would not be good for me!” Yes, but — may or may not be a Christian. Refined, amiable, cultured he may be. Living an upright, respectable and religious life. None such will care to go with the true Christian whose enlightened conscience necessitates his parting company with the world. The Spirit taught and spiritual Christian will have to pursue his lonely way unaccompanied by all such. Now those of us who are Christians, must part company with popular favor and public opinion. Scripture testifies that that which is “highly esteemed among men is abomination in the sight of God” (Luke 16:15).
These books, taking so rapidly amongst the reading class are influenced by the “spirit of the age,” which is never in accord with the mind of God. Their titles and themes, appeal at once to the natural religious mind of man.
God has been pleased to give us in the past half century, a very great many valuable books by His “teachers,” and they are they of whom it can truly be said, “I have taught thee in the way of wisdom, I have led thee in the right paths” (Prov. 4:11-13). Such “instruction” we should be diligent to “take fast hold of” and “keep,” and we shall prove indeed to be “life” to our souls. An aged brother has said, “Reading is not everything; it is study that is necessary,” and we may add, to allow the truths of the Word to take hold of the mind from whence they pass on to be realized and acted upon by the soul. How grateful the young Christian should be whose elders are maintaining the truth in the spirit of a contrite and steadfast soul. They are able to direct the unestablished to the writings that honor the Word of God. The word sound as applied to the quality of doctrine, occurs a complete number (seven) of times in the two Epistles of Timothy and Titus. Do not fail to look for them, because they will guard your soul from the inroads of false teachings. Notice very carefully that in 1 Timothy 6:3 it speaks of “wholesome words,” and “doctrine which is according to godliness.” Five times in the New Testament we find “fables” spoken against. Four times in the books referred to above, and once in 2 Peter 1:16.
As regards reading matter, like so many other pursuits, most people do not like the Bible introduced where its presence would be embarrassing. Alas, this is true of many Christians! While they are not abashed to freely chat of the popular works of fiction. The Bible of all other books is supreme. The young person who chooses it as his special reading matter, can never by any means make a mistake. Its preservation from time immemorial renders it the grandest of books, even looked at from a literary standpoint only. But deeper far than this is the fact that it is the Revelation of God to and for us. To it, then, let us turn to discern therein the mind of the Lord as to what or what not we should read. The Christian who is zealous for his Lord’s interests, will plainly see portrayed in many verses of the Scriptures, what should be the nature of his reading matter. Every word of it is “pure.” Every “jot and title” of it is God inspired. Its precepts, doctrines and teachings present the principles of the noblest and highest morality possible. Its pages unfold in a divine way the glories and beauties of the “Man after God’s own heart” our Lord and our Saviour. For the “babe” in Christ it is the “sincere milk.” For the older Christian it is “strong meat.” When the unnumbered hosts of earth’s books are swept into oblivion, the Bible will be the only everlasting and enduring Book. Young Timothy, whom the Apostle Paul had found at Lystra(Acts 16:1), had known the Scriptures from a child; and they were able to make him “wise” unto salvation “through faith which is in Christ Jesus.” Certainly, it is profitable to study well the Scriptures from earliest childhood; they should regulate the life in order that the man of God may be perfect. The Apostle had instructed Timothy in all manner of doctrine, and in a fatherly way exhorted him to “hold fast the form of sound words” (2 Tim. 1:13). He was to “continue in the things” which he had heard from the Apostle. He was to “study” to show himself an “approved” workman, that would have no need to be ashamed, and to be “approved” he must “shun profane and vain babblings” (2 Tim. 2:16), words that were no profit to the hearers.
How grateful we should be to God since He has preserved the Bible unsullied to this present hour — a time when its teachings were never more urgently needed by the saints of God. The young Christian who fails to make it his daily study, and to allow it to influence his life, will do so to the sorrowful injury of his own soul.
“Read the Bible to be wise, believe it to be safe, and practice it to be holy. Read it slowly, frequently, and prayerfully.”
(Continued)

Scripture Study: Matthew 10

Matthew 10:1. The Lord now, in compassion upon His people, calls His twelve disciples, and gives them authority and power over unclean spirits, to cast them out, and to heal all manner of disease and sickness. This is another proof to them that He was Emmanuel, who could not only heal and deliver, but could also confer on others the same power.
Matthew 10:2-4. The names of the twelve are given; Matthew does not forget that he had been a publican; grace teaches him to acknowledge it, as Paul did in 1 Timothy 1:15.
Matthew 10:5-6. This mission is to the lost sheep of the house of Israel only; Gentiles and Samaritans were as yet left out, but a Gentile centurion and a Canaanitish woman, exercising faith in the Lord, were not shut out. Needy ones always found a refuge in the Saviour, but as yet the message was sent to Israel only. (See John 12:20-24.)
Matthew 10:7. It is the gospel of the kingdom, that is, the announcement that the King is to appear.
The gospel of the grace of God is to all, and could not be preached till atonement was accomplished (Acts 20:21-24): “How am I straitened till it be accomplished!” (Luke 12:50). His heart of love desired to flow out unhinderedly to all.
Matthew 10:8. These are the signs of a deliverer, coming to meet man’s deep need, confirming the Word as His message (Heb. 2:3-4). What mercy and grace are expressed in it: “Heal the sick, raise the dead, cleanse the lepers, cast out demons: freely ye have received, freely give.” This power was given them; it was “the powers of the world to come” (Heb. 6:5), that is, the millennial kingdom will be characterized by this power; Satan will be bound and man delivered.
But we see how the disciples through lack of faith failed to exercise this power as they should (Matt. 17:19-21).
Matthew 10:9-10. They were not to make provision for the way, they were to depend entirely on the One who sent them, Emmanuel was there. Miracles might prove this to the world. His care over His own proved it to their hearts, they “lacked nothing.”
This was during the time of His presence on earth; see the difference when the Lord is away (Luke 22:35-37).
Matthew 10:11-15. These were as ambassadors, and serious indeed it was for those who did not receive them; it would be worse for them than for the land of Sodom and Gomorrah in the day of judgment.
From Matthew 10:16, the teaching is more general, and looks on to the time when the Son of Man will come (Matt 10:23).
The rejection of Christ and consequent scattering of the Jews and destruction of Jerusalem, His atonement and glorification, the Holy Spirit sent down, the church gathered and caught up and the Jews gathered back to their own land (though not mentioned here), have all come in before the coming of the Son of Man.
Matthew 10:16. The Lord sent these servants forth as “sheep in the midst of wolves.” They were to be “wise as serpents and harmless as doves.” Like the Lord Himself in this, wisdom from above it would surely be.
Matthew 10:17. “Beware of men.” Alas! men are opposed to God and to His people, and under the power of Satan; so they persecute the saints of God.
Matthew 10:18. The Lord had an object in allowing His servants to be brought before “men” for His sake, as a testimony that they had rejected Him.
Matthew 10:19. They could therefore face their trials with the calm, quiet assurance that it would be given them what to say at the right time; “the Spirit of their Father” would speak in them.
Matthew 10:21. Their brothers, fathers, children, would rise against them and cause them to be put to death; and they would be “hated of all men for My (His) name’s sake.” All this shows the terrible enmity of man’s heart against the Lord.
Matt 10:22. “But he that endureth to the end, the same shall be saved.” Notice, the end is the coming of the Son of Man. This is the endurance of the saints at the time of the great tribulation, when Satan tries to get them to worship the beast (Rev. 14:9-12).
We cannot rightly apply this verse to Christians. (See for our guidance, 1 Cor. 1:8, 9; Phil. 1:6; 1 Thess. 5:23-24).
Matthew 10:23. During the time when the Jews are back into their own land (but not now), this testimony will be carried from city to city of the land of Israel, and before they are finished, the Son of Man will have come. What a deliverance it will be for them!
Matthew 10:24-25. The disciples and servants were to be as their Master, that is, rejected and spoken against. It was their privilege; it is ours also (Phil. 1:29).
Matthew 10:26-27. Fear them not, proclaim the truth. They are to make known the things which the Lord has communicated to them. He will charge Himself with their care.
Matthew 10:28. The enemy may be allowed to kill their body; but, if so, it was as a testimony for the truth. So they were to fear Him who is able to destroy both soul and body in hell. (Gehenna—figurative of the place of eternal torments.)
Matthew 10:29-31. The comparatively worthless sparrow cannot fall to the ground without your Father, and He has numbered the very hairs of your head: “Fear not therefore, ye are of more value than many sparrows.”
Matthew 10:32-33. “Whosoever therefore shall confess Me before men, him will I confess also before My Father which is in Heaven. But whosoever shall deny Me before men, him will I also deny before My Father which is in Heaven.” Those who deny Him, deny His person and His work. Peter denied that he knew the Lord with oaths and curses; this was certainly very bad, but Peter did not deny that the Lord Jesus was the Son of God. He was a weak believer, but not a blasphemer; afterward he could say, “Ye have denied the holy One and the Just, and desired a murderer to be granted unto you” (Acts 3:13-15).
Matthew 10:34-36. Here again the state of man’s heart is seen as enmity against God and all that is His, and is stronger than all the ties of nature. The love of Christ in the believer is also to prove stronger than all human love.
Matthew 10:37-39. The Lord challenges their hearts to find a worthy object in Himself, dearer than all earthly ties, dearer than life itself. To put self first, is to fall and lose much spiritually.
Matthew 10:40-42. In receiving these witnesses, they received Him, and His Father also, who sent Him. And receiving them (in spite of an opposing world), even to the giving a cup of cold water in the name of a disciple, would bring them a sure reward. God takes knowledge of how each one acts toward His Son, and judges or rewards accordingly.

Truths for Young Christians: The Sin that Envy Leads to

THE SINS THAT ENVY LEADS TO.
Let us now just sum up from the few examples that we have selected, the crimes which are actually recorded in the Word, as having been committed through the sin of envy. We have seen that through envy, Christ was crucified—Abel was murdered—Joseph and David almost murdered; that it led at different times to hatred—wicked and unscrupulous conduct—lying and blaspheming—rioting and violence— hindering God’s work—despising God’s servant—lying and reviling—maliciousness—anger—and intense cruelty.
Surely, now that we have laid some part of the horrible form of this vice bare from Scripture, our readers must shudder to think that the root of all these crimes lurks in their hearts. James does not hesitate to say that envy is a root of every evil work (James 3: 16). It is worse than wrath or anger, none can stand before envy (Prov. 27:4). It hinders growth in grace (1 Peter 2:1,2), is a proof of carnal mindedness, (1 Cor. 3:1-2), it is one of the works of the flesh (Gal. 5:21), and one to which our spirits are especially liable (James 4:5), being produced by the prosperity and good deeds of others (Eccl. 4:4) and also by arguments and disputes (1 Tim. 6;4). Now to be forewarned is to be forearmed. Never let us give place to the devil, in allowing envy to sprout and germinate in our hearts; but let us ever check the first risings of an envious spirit.
A Cure for Envy
Seek to rejoice in the prosperity of others, seek to be unselfish; for, after all, envy is only a form of selfishness. Seek the good of others, not your own. Consider the Apostle and High Priest of our profession—Christ Jesus—who was not envious like Adam (Phil. 2), but emptied Himself (lit.), and ended a life of self-abnegation on the cross.
Ask yourself the question, Shall I allow for a moment in my heart the feeling of envy, a feeling which prompted the crucifixion of my Lord?
There is no saying to what length even a child of God may not be led, who once willingly allows this feeling. It grows so very rapidly that from only beginning to be envious of the success, prosperity, and position of another, we may soon begin to hate him, and then to plot against him.
As with pride, so it is with envy: its most horrible and deadly form is when it conceals itself under a cover of zeal for the Lord, and under this or some other religious subterfuge, seeks the evil of another. O! what unmaskings of all such actions will take place at the judgment seat of Christ.
Seek, beloved reader, to be pure from this vice at least, after the fearful warnings the Word of God has given us (remembering especially that it is one of the five sins that hinder our love of the Word of God itself (1 Peter 2). Real occupation with Christ’s glory and interests instead of our own, effectually though unceremoniously, checks not only this but many other sins. It is only the self-seeker who is envious. The servant who can truly say, like his Master, “I seek not my own glory,” is surely delivered from a spirit of envy.
May the Lord preserve us from this sin, which is alas! by no means uncommon amongst young as well as old believers.
(Continued)

Christ the Center: Or, Why Christians Should Meet in His Name Alone, Part 4

Do you think it possible that the Spirit of God could appoint such a man; a man who will enter the cottages of the poor and take their gospel tracts and put them in the fire? Could Satan desire more sad departure from the Spirit’s rule? Surely then I cannot be loyal to Christ, and join or sanction such a mutinous system. No; so far from the Church of England being the true church of God, it is only a piece of the world governed by the world—in fact the very opposite of the church of God, which is gathered out of the world, and governed by the Spirit of God.
As I have before proved, every other division of the professing church fails to recognize the personal government of the Holy Ghost, and sets up a government of its own; therefore, no division of the professing church can be called the true assembly or church of God any more than a division of the American army which failed to recognize the commander, and set up a commander of its own, could be called the true army of the United States.
I am fully aware that the personal guidance of the blessed Spirit of God has been so long forgotten that it is most difficult to make even Christians understand what is meant.
Take another illustration: a certain senator is announced to preside over a public meeting of the inhabitants of any given town. The meeting assembles, the senator comes, he stands on the platform, but nobody recognizes him; he speaks, but still no one knows him. Message after message is sent to his house begging him to come; they then desire his influence, and not knowing his personal presence, they appoint some one else to preside. Such is a precise picture of the divisions of the present day. However we may have grieved and disowned the Spirit, still that precious promise is fulfilled, “And He [the Father] shall give you another Comforter, that He may abide with you forever.”(John 14:16) Yes, as the senator was present though not known, when the letters were sent to his house, even so in the assembly of Christians, the Holy Ghost has come, is present, at the very time prayer is being offered, in ignorance, for Him to come from heaven. Yea, to hear many Christians pray, one would almost think that they were praying for an influence. Would it not be shocking to speak of God the Father as an influence? Would it not be most revolting to say the life of God the Son, on earth, was only an allegory or an influence? And is not the Holy Ghost as real a person now on earth, as Jesus was when on earth, and now is in heaven? What a commander is to an army, or a president is to a meeting, such is the Holy Ghost to the assembly of God: commanding, directing, using whom He will. Where He is not thus owned, no assembly even of Christians can be called God’s assembly. And hence from all such assemblies I must separate if I would be loyal to God.
(Continued)

An Appeal in These Last Moments

“Be ye steadfast, unmoveable, always abounding in the work of the Lord, forasmuch as ye know that your labor is not in vain in the Lord” (1 Cor. 15:58).
If there ever was a day when this word of exhortation and encouragement was needed, it is today, and especially for the young Christian.
Every evil doctrine is on the increase, and the advocates of such are showing increased energy in spreading them, and often with considerable self-sacrifice. On the other hand, those who are the Lord’s, are to a great extent folding their hands, taking their leisure, and giving Christ up to His enemies. May this not be the case with any of us who are readers of “The Young Christian,” but may each of us “be... steadfast, unmoveable.” Let us allow nothing to turn us out of the way, but be unmoveable from the path of faithfulness and response of heart to the Lord for all He has done for us. He has saved our souls at the great expense of giving Himself for us, and what more could He give than His life? He is not satisfied with doing this, but He is coming again for us, as the previous verses show, at any moment, to take us to be forever with Himself, and this is the reason for this word of exhortation.
Is that blessed hope so before our hearts, do we have it so as a present reality, that we are abounding in the work of the Lord? Let each of us ask ourselves the question, What am I doing for my Lord? What am I doing for the good of His people? What am I doing for the lost and perishing all around me? What about that unsaved classmate? that unsaved fellow workman? or that saved friend who needs instruction and encouragement in the things of God? Often just a word to a young Christian friend, or a decided stand for the Lord, in all lowliness and meekness, gives encouragement to the other for faithfulness of walk. If we feel unable to speak, we have the printed matter, that can be procured at a very low price, and sent to them each month. Twenty-five “sowers” can be sent to our unsaved acquaintances, at their different addresses, for less than is often spent on trivial things. “The Young Christian” contains simple truths that most Christians need; and if we were to pick out as many of our Christian friends as we could afford to send this paper to, only a little done in this way by each reader, would cover quite a large number; and then the little we do, if watered by prayer, with the Lord’s blessing upon it, Who can tell the extent of the blessing and the reward in that day so near?
May we be assured and encouraged by the last sentence, “Your labor is not in vain in the Lord.”

The Harvest Is Great

Use me, O God, in Thy great harvest field
That stretcheth far and wide, like a wide sea.
The gatherers are so few, I fear
The precious weald will suffer loss!
O, find a place for me; a place
Where best the strength I have will tell-
It may be one the other toilers shun;
Be it a narrow sphere or wide, ‘tis well,
So that Thy work be only done.

The Coming and Reign of Our Lord Jesus Christ: The Appearing, Part 3

THE APPEARING.
In Isaiah 2:12-21: “For the day of the Lord of hosts shall be upon every one that is proud and lofty, and upon every one that is lifted up; and he shall be brought low.... And the idols He shall utterly abolish. And they shall go into the holes of the rocks, and into the caves of the earth, for fear of the Lord, and for the glory of His majesty, when He ariseth to shake terribly the earth. In that day a man shall cast his idols of silver, and his idols of gold, which they make each one for himself to worship, to the moles and to the bats; to go into the clefts of the rocks, and into the tops of the ragged rocks, for fear of the Lord, and for the glory of His majesty, when He ariseth to shake terribly the earth.”
In Isaiah 13: “I have commanded My sanctified ones, I have also called My mighty ones for Mine anger, even them that rejoice in My highness. The noise of a multitude in the mountains, like as of a great people; a tumultuous noise of the kingdoms of nations gathered together: the Lord of hosts mustereth the host of the battle. They come from a far country, from the end of heaven, even the Lord, and the weapons of His indignation, to destroy the whole land. Howl ye; for the day of the Lord is at hand; it shall come as a destruction from the Almighty,” ... “And I will punish the world for their evil, and the wicked for their iniquity,” (Isa. 13:3-11).
“Behold, the Lord will come with fire, and with His chariots like a whirlwind, to render His anger with fury, and His rebuke with flames of fire. For by fire and by His sword will the Lord plead with all flesh: and the slain of the Lord shall be many” (Isa. 66:15-16).
After the judgment of the Western powers under the beast, the terrible foe of the Jews, the king of the North, “The Assyrian” will be smitten by the Lord. Used by God as the rod of His wrath to chastise the apostate Jews, when they bow to the idol and the beast, he exalts himself against God, and is then himself judged (Isa. 10:5-12). He comes as a destroying storm upon the land (Isa. 28:2), attacks antichrist in Jerusalem, at the head of a powerful army, takes the city, and overthrows many countries (Dan. 11:40-41). The king of the South (Egypt) pushing at antichrist, the willful king, at the same time (Dan. 11:40), is also conquered by him (the Assyrian); but tidings out of the East and out of the North troubling him, he returns with great fury to destroy and utterly take away many of the Jews— plants the tabernacles of his palace between the seas close to Jerusalem, and is then smitten by the Lord (Dan. 11:41-45).
A graphic description of his last march to Jerusalem is given in Isaiah 10, where he is seen to shake his hand against the city; but just as he thinks to possess it a second time, “Behold,” saith the prophet, “the Lord, the Lord of hosts, shall lop the bough with terror.”(Isa. 10:33)
A striking type of the Assyrian’s fall is seen in the destruction by the angel of the Lord of the mighty army of Sennacherib (Isa. 37:36).
This closes the judgments of God on Israel and the nations; the Deliverer, the Lord, comes out of Zion, and delivers His faithful remnant (Rom. 11:26). “Therefore thus saith the Lord God of hosts, O My people that dwellest in Zion, be not afraid of the Assyrian: he shall smite thee with a rod, and shall lift up his staff against thee, after the manner of Egypt. For yet a very little while, and the indignation shall cease, and Mine anger in their destruction” (Isa. 10:24-25). “The consumption decreed shall overflow with righteousness” (Isa. 10:22). Isa 11-12 teem with millennial blessing.
Another very distinct scripture (Mic. 5:5-7), has reference to this same period. After speaking of One who shall come forth from Bethlehem to be ruler in Israel (Mic. 5:2), the prophet continues, “And this man shall be the peace, when the Assyrian shall come into our land: and when he shall tread in our palaces, then shall we raise against him seven shepherds, and eight principal men” (Marg., princes of men.) “And they shall waste the land of Assyria with the sword, and the land of Nimrod in the entrances thereof: thus shall He deliver us from the Assyrian, when he cometh into our land, and when he treadeth within our borders, And the remnant of Jacob shall be in the midst of many people as a dew from the Lord.”
(Continued)

Correspondence: Psa. 138:2; Acts 2:16-17; Rom. 11:26; The Lord's Table

Question 57: Please explain Psalm 138:2: “Thou hast magnified Thy Word above all Thy name.” L. S.
Answer: The absolute authority of the Word of God is seen in this verse. Jehovah has made His Word great above all His renown. We must therefore receive and honor the written Word above all that we have heard of Him: above everything else.
Question 58: Is there any authority in the Word of God for women preaching? Does Acts 2:16-17, give any? I. E. K.
Answer: 1 Corinthians 14:34-36 and 1 Timothy 2:12, distinctly forbid women speaking in public, or taking a place as teacher or preacher to men. We never find a woman speaking publicly in Scripture. Mary carried a message to the brethren (John 20:17-18). Philip had four daughters that prophesied, but it must have been privately. They were forbidden to be speakers in the assembly. The quotation from Joel’s prophecy will be fulfilled in the day of Israel’s restoration. The Apostle said, “This is that which was spoken by the prophet Joel,” that is, it was of the same kind. It was not drunkenness, as they supposed.
Question 59: Please give a few scriptures to prove that Question 45 is answered right, as I have heard another interpretation. I. H. (Ques. 45. What does “ALL ISRAEL shall be saved” mean? Rom. 11:26.)
Answer: Romans 11:26. Just now Jews and Gentiles are all alike needy sinners, are all concluded in unbelief (Rom. 11:32, John 3:36).
Ezekiel 37:11-12. “The whole house of Israel” is the uniting of the two and the ten tribes together into one stick, one kingdom. “I will open your graves” in this passage and in Daniel 12:2, is taking the Israelites out from among the heathen nations and bringing them into their own land (Ezek. 37:21).
Zechariah 12:9-13:1. This is the way the Lord brings every one of them who will be saved, to repentance, and the fountain is opened for them; but in Zechariah 13:8-9, we see two-thirds of them cut off and die; in Ezekiel 20, it is the ten tribes who are brought into the wilderness and are purged there, so that the rebels do not enter into the land at all (Ezek. 20:34-38). (Read also Matt. 13:41-42) The antichrist is himself a Jew (Dan. 12:37), and is cast into the lake of fire (Rev. 19:20).
But in the millennium, when the covenant is established with Israel (the ten tribes), and with Judah (the two tribes), then each of them shall know the Lord, from the least of them unto the greatest of them (Heb. 8:8-12; Ezek. 36:25-29).
Question 60: How do we really know that we are at the Lord’s table? There have been so many divisions; how do we know that we are with the ones who have remained faithful to His name and are still meeting on the ground of the one body? Are there any scriptures to prove the same? L. M. S.
Answer: The truth that “there is one body,” that we are united by one Spirit to Christ our head, and to all believers as members one of another, is the blessed truth that brings us together. Every denomination claims to have the Lord’s table, and they have professedly (1 Cor. 1: 2), and many believers in them enjoy individual fellowship with the Lord in taking the Lord’s supper, but it is with those actually gathered to Christ’s name (Matt. 18:20), that His presence is promised, “there am I in the midst of them.” To be gathered to His name, is to be gathered in holiness and truth; His name means that (Rev. 3:7).
The Holy Spirit is also the Spirit of truth: He is the gatherer and can gather only to that name. The truth, where it is owned, will have us together as one in separation from evil. The two wave loaves baken with leaven (Lev. 23:17), is typical teaching, that evil, that is the flesh, is in us all, and we need forbearance with each other on this account (Eph. 4:2), but the leaven is not working when it is baked. There are three kinds of evil, which, if allowed, would deprive us of the presence of Christ in the midst.
First: Immorality, or unrighteous living (1 Cor. 5). Second: Bad doctrine that touches the foundation of our faith in Christ and His finished work (2 John 9-10). Third: Divisions (1 Cor. 1:10; Rom. 6:17-18; Eph. 4:1-4).
The first two are easy to understand. The last asks our submission to the truth that we are one, for without this unity of the Spirit, discipline could not be carried out, nor divisions kept out. The unity of the Spirit implies identification with all those with whom we break bread, and judges all those who, though right in their individual walk and doctrine, are associated with loose or independent meetings of Christians, and who are therefore not gathered in the Spirit’s unity.
The start of each meeting of Christians generally tells what principles it is gathered on. If not gathered by the Spirit, it is man’s will that brought them together; they cannot in truth claim Matthew 18:20. A wrong thing will not grow right. It is of no use for any of us to say, we are right, for a right thing often goes wrong. What we need is subject hearts to the truth, hearts that desire to keep Christ’s word and not to deny His name (Rev. 3:8). Going on, subject to, and maintaining, the truth of the unity of the body, in separation from evil, the Lord will be with us according to His Word.
Courtesy of BibleTruthPublishers.com. Most likely this text has not been proofread. Any suggestions for spelling or punctuation corrections would be warmly received. Please email them to: BTPmail@bibletruthpublishers.com.